You are on page 1of 279

2 0 1 9

SECONDARY 3
Normal Academic Exam Paper
Science Chemistry NA
1 Assumption English SA1
2 Bedok View Sec SA1
3 Canberra Sec SA1
4 Jurong West Sec SA1
5 Bartley Sec SA2
6 Beatty Sec SA2
7 Canberra Sec SA2
8 East Spring Sec SA2
9 Hillgrove Sec SA2
10 Hougang Sec SA2
11 Ken Ridge Sec SA2
12 Kranji Sec SA2

www.KiasuExamPaper.com
1
www.KiasuExamPaper.com
2
1

Name: ……………………………………………….. ( ) Class: ……...

ASSUMPTION ENGLISH SCHOOL


MID-YEAR EXAMINATION 2019

SCIENCE (CHEMISTRY)
5105 / 03
5107 / 03

ASSUMPTION ENGLISH SCHOOL ASSUMPTION ENGLISH SCHOOL ASSUMPTION ENGLISH SCHOOL ASSUMPTION ENGLISH SCHOOL
ASSUMPTION ENGLISH SCHOOL ASSUMPTION ENGLISH SCHOOL ASSUMPTION ENGLISH SCHOOL ASSUMPTION ENGLISH SCHOOL
ASSUMPTION ENGLISH SCHOOL ASSUMPTION ENGLISH SCHOOL ASSUMPTION ENGLISH SCHOOL ASSUMPTION ENGLISH SCHOOL
ASSUMPTION ENGLISH SCHOOL ASSUMPTION ENGLISH SCHOOL ASSUMPTION ENGLISH SCHOOL ASSUMPTION ENGLISH SCHOOL
ASSUMPTION ENGLISH SCHOOL ASSUMPTION ENGLISH SCHOOL ASSUMPTION ENGLISH SCHOOL ASSUMPTION ENGLISH SCHOOL

LEVEL: Sec 3 Normal Academic DATE : 14 May 2019


Sec 3 Normal Technical

CLASSES: Sec 3/3, 3/4, 3/5 SBB DURATION: 1 hour 15 minutes


(Papers 3 & 4)
Additional Materials provided: 1 sheet of OAS paper

INSTRUCTIONS TO CANDIDATES
Do not open this booklet until you are told to do so.

Write your NAME, INDEX NUMBER and CLASS at the top of this page and on the
OAS paper. Shade your index number on the OAS paper.

There are 20 questions in this paper. Answer all questions. For each question, there
are four possible answers A, B, C and D. Choose the correct answer and record your
choice in soft or 2B pencil on the OAS paper provided. DO NOT fold or bend the
OAS paper.

At the end of the examination, hand in your OAS paper and Question Papers
separately.

INFORMATION FOR CANDIDATES


You are advised to spend no longer than 30 minutes on Paper 3.
You may proceed to answer Paper 4 as soon as you have completed Paper 3.

A copy of the Periodic Table is printed on the last page of Paper 4.

This question paper consists of 8 printed pages including this page.


[Turn over

www.KiasuExamPaper.com
3
2

Multiple Choice Questions (20 marks)


There are twenty questions in this section. Answer ALL questions. For each question,
there are four possible answers, A, B, C and D. Choose the one you consider correct and
record your choice on the OAS in soft pencil.

1 The melting and boiling points of four substances are given below in the table.

Which substance is in the solid state at 10 °C?

melting point / ºC boiling point / ºC


A í110 í50
B í4 25
C 0 100
D 58 203

2 The graph below shows part of the cooling curve of a liquid substance.
temperature / °C

Q
R

time / min

Which row correctly identifies the states of the substance at regions P, Q and R?

P Q R
A liquid and gas liquid liquid and solid
B liquid liquid and solid solid
C gas liquid solid
D liquid liquid and gas gas

5105/5107/03/3NA/MYE/19

www.KiasuExamPaper.com
4
3

3 Which letter (A, B, C or D) represents the process of condensation?

Liquid D
A
B C
Solid Gas

4 A balloon filled with helium gas is placed in a basin of hot water. It is observed that
the balloon increases in size.

Which statement best describes this phenomenon?

A The helium gas particles increase in size, and spaces between them
decrease.
B The helium gas particles decrease in size, and spaces between them
decrease.
C The helium gas particles remain the same size, and spaces between the
particles increase.
D The helium gas particles remain the same size, and spaces between the
particles decrease.

5 The diagram shows a burette filled with dilute hydrochloric acid.

25 cm3

26 cm3

27 cm3

What is the volume of acid used if it started off at 0 cm3 in the burette?

A 25.70 cm3
B 25.75 cm3
C 25.80 cm3
D 25.85 cm3

5105/5107/03/3NA/MYE/19 [Turn over

www.KiasuExamPaper.com
5
4

6 A gas, X, is denser than air and insoluble in water.

Which method is not used to collect the gas?

A X B

C D

X
X
water

7 A student makes some crystals.

What does the student check to test the purity of the crystals?

A the melting point of the crystals


B the shape of the crystals
C the size of the crystals
D the solubility of the crystals in water

8 Mixture 1 contains sand and water.

Mixture 2 contains salt and water.

Which method of separation could be used to obtain each of the required


products from each mixture?

mixture 1 mixture 2
to obtain sand to obtain water to obtain salt to obtain water
A filtration distillation crystallisation filtration
B filtration filtration crystallisation distillation
C crystallisation distillation filtration filtration
D crystallisation filtration filtration distillation

5105/5107/03/3NA/MYE/19

www.KiasuExamPaper.com
6
5

9 Which diagram best represents a mixture of two compounds?

A B C D

10 Which examples of elements, compounds and mixtures are correct?

element compound mixture

A aluminium water apple juice

B copper sea water oxygen gas

C water carbon dioxide urine

D platinum milk carbonated drink

11 The diagram shows an outline of the Periodic Table.

W
X
Z
Y

Which pair of elements has been placed in the correct category?

metal non-metal

A W Z

B X Y

C X Z

D Z Y

5105/5107/03/3NA/MYE/19 [Turn over

www.KiasuExamPaper.com
7
6

12 How many atoms are present in a formula unit of aluminium sulfate, Al 2 (SO 4 ) 3 ?

A 4
B 7
C 17
D 18

13 Atoms of all elements have equal numbers of ………………….

A neutrons and electrons


B protons and electrons
C protons and neutrons
D protons, neutrons and electrons

14 The diagram shows the structure of an atom of an element.

Which group and period of the Periodic Table does this atom belong to?

group period
A II 3
B II 5
C V 2
D V 3

5105/5107/03/3NA/MYE/19

www.KiasuExamPaper.com
8
7

15 Which option shows the correct relative charges of the particles in an atom?

electron neutron proton


A -1 0 +1
B 0 +1 -1
C +1 -1 0
D -1 +1 0

16 Two particles X and Y have the composition shown in the table below.

particle number of protons number of neutrons number of electrons


X 17 20 18
Y 17 21 18

Which statement is true for particles X and Y?

A Both particles are positive ions.


B Both particles are metallic atoms.
C Both particles are isotopes of the same element.
D Both particles have the same melting and boiling points.

17 Which row shows the general properties of an ionic compound?

conduct electricity
melting point soluble in water
solid liquid
A high no no yes
B high yes no yes
C low no no no
D low yes yes yes

5105/5107/03/3NA/MYE/19 [Turn over

www.KiasuExamPaper.com
9
8

18 An atom X has an electronic configuration of 2, 8, 2.

What ion will atom X form?

A X2+
B X2-
C X6+
D X6-

19 How does a barium atom form a bond with a sulfur atom?

A by giving two electrons to the sulfur atom


B by sharing one pair of electrons
C by sharing two pairs of electrons
D by taking two electrons from the sulfur atom

20 The chemical formula of ammonium ion and carbonate ion is NH 4 + and CO 3 2-


respectively.

What is the formula for ammonium carbonate?

A NH 4 CO 3
B (NH 4 ) 2 CO 3
C NH 4 (CO 3 ) 2
D (NH 4 ) 3 (CO 3 ) 2

- End of Paper 3 -

5105/5107/03/3NA/MYE/19

www.KiasuExamPaper.com
10
Name: ……………………………………………….. ( ) Class: ……...

ASSUMPTION ENGLISH SCHOOL


MID-YEAR EXAMINATION 2019

SCIENCE (CHEMISTRY)
5105 / 04
5107 / 04

ASSUMPTION ENGLISH SCHOOL ASSUMPTION ENGLISH SCHOOL ASSUMPTION ENGLISH SCHOOL ASSUMPTION ENGLISH SCHOOL
ASSUMPTION ENGLISH SCHOOL ASSUMPTION ENGLISH SCHOOL ASSUMPTION ENGLISH SCHOOL ASSUMPTION ENGLISH SCHOOL
ASSUMPTION ENGLISH SCHOOL ASSUMPTION ENGLISH SCHOOL ASSUMPTION ENGLISH SCHOOL ASSUMPTION ENGLISH SCHOOL
ASSUMPTION ENGLISH SCHOOL ASSUMPTION ENGLISH SCHOOL ASSUMPTION ENGLISH SCHOOL ASSUMPTION ENGLISH SCHOOL
ASSUMPTION ENGLISH SCHOOL ASSUMPTION ENGLISH SCHOOL ASSUMPTION ENGLISH SCHOOL ASSUMPTION ENGLISH SCHOOL

LEVEL: Sec 3 Normal Academic DATE : 14 May 2019


Sec 3 Normal Technical

CLASSES: Sec 3/3, 3/4, 3/5 SBB DURATION: 1 hour 15 minutes


(Papers 3 & 4)

Additional Materials provided: NIL

INSTRUCTIONS TO CANDIDATES
Do not open this booklet until you are told to do so.

Write your NAME, INDEX NUMBER and CLASS at the top of this page.
Write in dark blue or black pen on both sides of the paper.
You may use a soft pencil for any diagrams or graphs.
Do not use staples, paper clips, highlighters, glue or correction fluid.

Answer all questions in Section A and any two questions in Section B.


In calculations, you should show all the steps in your For Examiner’s Use
working, giving your answer at each stage.
Paper 3
You are advised to spend no longer than 30 minutes on 20
Paper 3.
You may proceed to answer Paper 4 as soon as you have
Section A
14
completed Paper 3.
A copy of the Periodic Table is printed on the last page of Section B
16
Paper 4.
Total
50
At the end of the examination, hand in your OAS paper and Question Papers separately.
The number of marks is given in brackets [ ] at the end of each question or part question.

This question paper consists of 11 printed pages including this page.


[Turn over

www.KiasuExamPaper.com
11
2

Section A [14 marks]


Answer all the questions in the spaces provided.

1 The proton (atomic) numbers of some elements are shown in the table.

element magnesium potassium fluorine neon


proton number 12 19 9 10

(a) (i) Draw the electronic structure of neon.

[1]

(ii) Using the electronic structure in part (a)(i), explain why neon is an
unreactive gas.

………………………………………………………………………………

……………………………………………………………………………… [1]

(iii) Draw the arrangement of the particles of neon gas in the box given.

[1]

(b) (i) Write the electronic configuration for potassium ion.

……………………………………………………………………………… [1]

(ii) Is potassium fluoride, a compound formed by potassium and fluorine,


an ionic or covalent compound? Give a reason for your answer.

………………………………………………………………………………

……………………………………………………………………………… [1]

5105/5107/04/3NA/MYE/19

www.KiasuExamPaper.com
12
3

2 (a) A food inspector used chromatography to test a new food colouring X, for
the presence of red, blue and green dyes. The result is shown below.

pencil line

X red blue green


(i) What physical property are the food colourings separated based on?

……………………………………………………………………………… [1]

(ii) Why must the start line be drawn in pencil?

………………………………………………………………………………

……………………………………………………………………………… [1]

(iii) On the diagram above, sketch the chromatogram for food colouring
X which only contains the dyes red and blue. [1]

(b) A student placed a drop of brown dye, A, and a drop of green dye, B, on a
piece of filter paper. The end of the filter paper was then dipped into a
beaker of water. The results are shown below.

A B

State two conclusions that can be made from this results.

……………………………………………………………………………………..

……………………………………………………………………………………..

…………………………………………………………………………………….. [2]
5105/5107/04/3NA/MYE/19 [Turn over

www.KiasuExamPaper.com
13
4

3 20.80 cm3 of ethanoic acid, CH 3 COOH, is added into a conical flask. 1.25 g of
magnesium carbonate, MgCO 3 , is then added into the flask. It is observed that
35 cm3 of carbon dioxide gas is evolved and the temperature of the mixture rose
to 42.3°C.

(a) State all suitable apparatus needed to measure the physical quantities
mentioned above.

(i) 20.80 cm3 of ethanol ……………………………………………

(ii) rose to 42.3°C …………………………………………… [2]

(b) State the composition of elements in the compounds used in the


experiment. The first one has been done for you.

compound chemical formula composition of elements

2 x hydrogen atoms
water H2O 1 x oxygen atom

ethanoic acid CH 3 COOH

magnesium MgCO 3
carbonate
[2]

5105/5107/04/3NA/MYE/19

www.KiasuExamPaper.com
14
5

Section B [16 marks]

Answer any two questions from this section in the spaces provided.

4 A sample of solid X is heated and the thermometer readings are recorded as


shown in the table below.

time / min 0 2 4 6 8 10 12 14
temperature / °C 20 50 80 110 110 110 130 150

(a) Plot a graph of temperature against time, marking each point with a cross
(×). The first two points have been done for you. Draw the heating curve of
solid X.

[2]

5105/5107/04/3NA/MYE/19 [Turn over

www.KiasuExamPaper.com
15
6

(b) From the graph, what is the melting point of solid X?

…………………………………………………………………………………….. [1]

(c) Using the graph, determine how long it takes for solid X to be heated from
20 qC to 92 qC.

…………………………………………………………………………………….. [1]

(d) Describe the changes in arrangement and movement of the particles in X


at when it is heated from 20 qC to 92 qC.

……………………………………………………………………………………..

……………………………………………………………………………………..

……………………………………………………………………………………..

…………………………………………………………………………………….. [2]

(e) (i) Is solid X a pure substance or a mixture? Explain your answer.

………………………………………………………………………………

……………………………………………………………………………… [1]

(ii) Complete the table below about compounds and mixtures.

compounds mixtures

can be separated by can be separated by


.………………… …………………
methods. methods.
[1]

5105/5107/04/3NA/MYE/19

www.KiasuExamPaper.com
16
7

5 The table shows some of the properties of the first six members of the alkane
homologous series.

name boiling point / qC physical state at 20 qC


methane  164 gas
ethane  89 gas
propane  42 gas
butane 0
pentane 36 liquid
hexane 69 liquid

(a) Write, in the last column, the physical state of butane at 20 qC. [1]

(b) A mixture of water, pentane and hexane can be separated using the
apparatus shown.

(i) Name this method of separation.

……………………………………………………………………………… [1]

(ii) Which of the three chemicals will collect in the beaker first?
Explain your answer.

………………………………………………………………………………

……………………………………………………………………………… [2]

5105/5107/04/3NA/MYE/19 [Turn over

www.KiasuExamPaper.com
17
8

(c) Methane has a chemical formula of CH 4 . It is made up of one carbon atom


and four hydrogen atoms.

Carbon has 15 different known isotopes. Two of the isotopes are listed in
the table below.

atom symbol number of number of number of


protons electrons neutrons

ଵଶ
carbon-12 ଺‫ܥ‬

ଵସ
carbon-14 ଺‫ܥ‬

(i) Complete the table. [2]

(ii) What is meant by the term ‘isotope’?

………………………………………………………………………………

……………………………………………………………………………… [1]

(iii) Do the two isotopes have similar chemical properties?


Explain your answer.

………………………………………………………………………………

……………………………………………………………………………… [1]

5105/5107/04/3NA/MYE/19

www.KiasuExamPaper.com
18
9

6 The table below shows a list of substances, containing common elements,


compounds and mixtures.

sulfur milk oxygen


air carbon dioxide seawater
sodium chloride magnesium hydrogen

(a) From the list, write down one substance that

(i) does not have a fixed boiling point, and ……………………………. [1]

(ii) is a group VI element. ……………………………. [1]

(b) (i) Draw a ‘dot-and-cross’ diagram to show the electron arrangement of


the product formed when an atom of magnesium reacts with an atom
of oxygen. Show valence electrons only.

[2]

(ii) Explain why magnesium oxide has a high melting point.

………………………………………………………………………………

………………………………………………………………………………

………………………………………………………………………………

……………………………………………………………………………… [2]

5105/5107/04/3NA/MYE/19 [Turn over

www.KiasuExamPaper.com
19
10

(iii) The diagram shows how solid magnesium oxide can be tested for
electrical conductivity.
battery

light bulb

solid magnesium
oxide

crocodile clip

State the result you would expect to see. Explain your answer.

………………………………………………………………………………

………………………………………………………………………………

……………………………………………………………………………… [2]

- End of Paper 4 -

5105/5107/04/3NA/MYE/19

www.KiasuExamPaper.com
20
11

5105/5107/04/3NA/MYE/19 [Turn over

www.KiasuExamPaper.com
21
www.KiasuExamPaper.com
22
ASSUMPTION ENGLISH SCHOOL
Sec 3 Normal Academic Science (Chemistry) 5105, 5107 Marking Scheme
Mid Year Examinations 2019

Paper 3 (20m)

Q1 Q2 Q3 Q4 Q5 Q6 Q7 Q8 Q9 Q10

D B D C B B A B B A

Q11 Q12 Q13 Q14 Q15 Q16 Q17 Q18 Q19 Q20

C C B C A C B A A B

Paper 4 Section A (16m)

1 ai 1

valencce shell
It has a full valence sh
s heelll / complete
com
comp ple
lete
te valence
val
a ence shell / noble
noble gass cconfiguration
onfi
onfiguratio
fi
aii 1
andd does not
nott need
need to
to form
orm bond.
for bo
b on
nd
d.

aiii
aiiiiii 1

bi 2.8.8
2.8.8
2.
2.8 8 1
Io
oni
niic.
Ionic.c
c.
bii 1
Potassium
Potatassiuum (metal)
iu (me
(m and fluorine (non-metal) forms an ionic bond.
etal) an

2 ai They
ey have different solubilities in the solvent. 1
cil lead is insoluble in solvent and does not affect the accuracy of the
Pencil
aii 1
chromatography results.

aiii 1

www.KiasuExamPaper.com
23
1. Dye A is a mixture / Dye B is pure.
2. Dye B is a component / is a part of Dye A.
b 2
Award 1m for each conclusion.

3 ai Burette 1
aii thermometer 1
compound chemical formula composition of elements
2 x carbon atoms
4 x hydrogen atoms
ethanoic acid CH 3 COOH
2 x oxygen atoms
b 2

1 x magn
magnesium
nes
esiu
ium atom
iu a
magnesium MgCO 3
carbonate 1 x ca
carbon
arb
rbon
naatom
tom
3 x ox
oxygen atoms
en ato
oms
m

www.KiasuExamPaper.com
24
Paper 4 Section B (20 m)

4 a 2

Award
Award
A d 1m m for
for correct
fo correcct points
p in
po ints
s plotted
plo
lott
tted
ed
A
Aw war
Awarda d 1m m ffor
orr graph
o ph
p h drawn
dra
awn
w
b 11
1 10 oC
110 1
c 4.8
4 8 min (accept
4. ((a
acc
cceeppt +/-
+ - of
+/ of 0.2 min)
min
in)) 1
o
At 2
At 200C C,, the
he p
he particles
a ticles
ar es are
are cclosely
losely packed in an orderly arrangement and
1
vvibrate
viib ate about
brrate
at ab
a bout their
bou
ou fixed
theiir fi
fixe
xed
xe d positions.
positi
d
At 9922oC,
C tthe
he p
particles
article are less closely packed in a disorderly arrangement
1
and
nd slide
slid
sl e over o
ide
id one another.
Purere subst
substance
ei 1
It hass a fixed melting point.
Compound - Chemical
eii Mixture - Physical 1
Award 1m only when both answers are correct

5 a Gas 1
bi Fractional distillation 1
Pentane. 1
bii
It has a lowest boiling point. 1
atom symbol number of number of number of
ci 2
protons electrons neutrons

www.KiasuExamPaper.com
25
carbon- 6 6 6
12

carbon- 6 6 8
14
Award 1m for every 3 correct answers.
Atoms of the same element, with the same number of protons, different
cii 1
number of neutrons.
Yes.
ciii
They have the same number of electrons / electronic configuration. 1

6 ai Milk OR seawater OR air 1


aii Oxygen OR sulfur 1

bi 2

Award 1m for o each h co


ccorrect
orrrrec
ect io iion.
on.
n
he oppositely
The opposi s teely
ly charged
charged d ionss are a e held
ar held together
tog
oget
ether by strong
str
trongg electrostatic
elec
electr
trostati 1
bii forc
forces
ces
es of at attraction
ttrra
accti
t on
on innag giant
iia
ant
nt iionic
onicc structure.
strruc
uctuture
re.
energy
A lott of ene erg gy requ
re
required
equ
quirire
ed d tto
o over
overcome
errco
come
me thethe
he strong g fo
forc
forces.
rces
rc e . 1
T
Th
Thee light
liigh
ght bulb
bu
ulb b will noot light
not lilig
ghht u up p/N o vi
No vvisible
isi
s blee ch
chanange
ange.
ge
change. 1
biii
bi
biiii T
Th
There
ere areare
ar e noo mobile
mobile io ions
ons ns iinn a sso
solid
olid io
ionic
oni
nicc co
com
compound
mpoun to act as charge 1
ccarriers
caarrrie
iersr to
to conduct
cond uct electricity.
duc elec
el ecctr tr
tricity.
tr

www.KiasuExamPaper.com
26
www.KiasuExamPaper.com
27
BEDOK VIEW SECONDARY SCHOOL
MID-YEAR EXAMINATION 2019

CANDIDATE
NAME

REGISTER
CLASS
NUMBER

SCIENCE (CHEMISTRY) 5105


Secondary 3 Normal Academic 13 May 2019
1 hour 15 minutes
Additional Materials: Multiple Choice Answer Sheet

READ THESE INSTRUCTIONS FIRST

Write your index number and name on all the work you hand in. For Examiner’s use
Write in dark blue or black pen on both sides of the paper.
Section A / 20
You may use a soft pencil for any diagrams, graphs or rough working.
Do not use staples, paper clips, highlighters, glue or correction fluid. 21 / 8
22 / 2
Information for Candidates
23 / 8
Section A 24 / 6
There are twenty questions in this section. Answer all questions. For each
25 / 6
question there are four possible answers A, B, C and D.
Choose the one you consider correct and record your choice in soft pencil Total / 50
on the separate Answer Sheet.
% / Grade /
Section B
Answer all questions.
Write your answers in the spaces provided.

The use of an approved calculator is expected, where appropriate.


In calculations, you should show all the steps in your working, giving
your answer at each stage.

The number of marks is given in brackets [ ] at the end of each


question or part question.

A copy of the Periodic Table is printed on page 13.

Setter(s): Mrs KK Wai Leng Parent’s / Guardian’s Signature: ………………….…………...

This document consists of 13 printed pages.


Do not turn over the page until you are told to do so.

www.KiasuExamPaper.com
28
2

Section A
Answer all questions in this section on the Multiple Choice Answer Sheet.

1 The S.I. unit for mass is ………….

A kg B ton

C g D mg

2 Which apparatus is most suitable to measure 10.55 cm3 of liquid accurately?

A measuring cylinder

B burette

C beaker

D pipette

3 Ammonia and hydrogen chloride cannot be collected by downward displacement of


water.
They are collected instead by the methods shown below.

ammonia

hydrogen chloride

What deductions could you make about the properties of the two gases?

ammonia hydrogen chloride

density solubility in water density solubility in water

A denser than air insoluble less dense than air insoluble

B denser than air insoluble less dense than air soluble

C less dense than air insoluble denser than air insoluble

D less dense than air soluble denser than air soluble

BVSS

www.KiasuExamPaper.com
29
3

4 The chromatogram below is obtained from four different dyes.

Which dye will boil at a fixed temperature?

A B C D

5 Seawater contains dissolved sodium chloride.

Which of the following methods is used to obtain pure water from seawater?

A Filtration B evaporation to dryness

C Crystallisation D simple distillation

6 A student tries to separate a mixture of ethanol and water by fractional distillation using
the apparatus shown.

thermometer
water out

condenser

water in
receiver

liquid

heat

Which error has the student made?

A The condenser is at the wrong angle.

B The thermometer is in the wrong position.

C The top of the receiver should be open.

D The water enters the condenser in the wrong place.

BVSS [Turn over


www.KiasuExamPaper.com
30
4

7 Substance W has a melting point of 20 oC and a boiling point of 59 oC. At which


temperature would substance W be a solid?
A B C D

- 50 - 25 0 25 50 75 100 125 150


temperature (oC)

8 The diagram shows the arrangement of the particles in substance X at two different
temperatures.
– 150 oC – 50 oC

What could be the melting and boiling points of substance X?

melting point /oC boiling point /oC

A – 161 – 65

B – 161 – 49

C – 148 – 65

D – 148 – 49

9 The figure below shows a graph obtained by heating solid Z in air.

temperature /oC

time /mins

Which statement is true about the particles of Z at the fourth minute?

A All the particles are closely packed.

B All the particles move freely in all directions.

C Some particles are closely packed and some particles slide past each other.

D Some particles slide past each other while some particles move freely in all
directions.
BVSS

www.KiasuExamPaper.com
31
5

10 Which list contains three elements?

A sulfur, sodium, magnesium oxide

B carbon dioxide, carbon, air

C silver, sodium, sulfur

D seawater, salt, silver

11` Which substance is likely to be a pure compound?

A a brown solution.

B an oily liquid which gives two fractions when distilled.

C blue crystals which melt over a range of 55 oC to 60 oC.

D colourless crystals which melts completely at 57 oC.

12 What is the total number of atoms in one molecule of butanoic acid, C 4 H 8 O 2 ?

A 3 B 14 C 16 D 34

13 The diagrams below show the relative masses and charges of particles Q, R and S.

1
relative charge
relative mass

1 0
Q R S
0 -1
Q R S

What are the identities of particles Q, R and S?

Q R S
A electron proton neutron

B electron neutron proton

C proton electron neutron

D proton neutron electron

BVSS [Turn over


www.KiasuExamPaper.com
32
6

14 Which equation is correct for all atoms?

A number of electrons = number of protons

B number of electrons = number of neutrons

C number of neutrons = number of protons + number of electrons

D number of electrons = number of protons + number of neutrons

15 The diagram represents an atom.

key
proton
neutron
electron

What is the nucleon number of this atom?

A 3 B 4

C 7 D 10

16 The diagrams show the nuclei of four different atoms.

Q R S T
key:
10 p 20 p 12 p 14 p p = proton
10 n 20 n 14 n 14 n n = neutron

Which two atoms belong to the same period in the periodic Table?

A Q and R B Q and T

C R and S D S and T

17 The element T has an electronic configuration of 2.6. Which ion will it form?

A T
B T
C T2
D T2

BVSS

www.KiasuExamPaper.com
33
7

18 Which ion has the correct set of sub-atomic particles?

number of
ion
protons electrons neutrons
A 2 -
1H 1 1 2

B 18 2-
8O 10 12 8

C 27 3+
13Al 13 10 14

D 40 2+
20Ca 20 20 20

19 Which of the following takes place during the formation of an ionic bond?

A atoms share electrons

B atoms share protons

C atoms gain and lose electrons

D atoms gain and lose protons

20 Which set of properties are those of ionic compounds?

electrical conductivity electrical conductivity electrical conductivity


of solid of molten compound of aqueous solution
A good good good

B poor good good

C poor poor good

D poor poor high

BVSS [Turn over


www.KiasuExamPaper.com
34
8

Section B
Answer all questions in the spaces provided

21 Fig. 21.1 shows the nuclei of seven elements, P, Q, R, S, T, U and V. For


examiner’s
They are not the symbols of elements. use

P Q R S
3 protons 17 protons 11 protons 17 protons
5 neutrons 20 neutrons 12 neutrons 18 neutrons

T U V
12 protons 10 protons 8 protons
12 neutrons 10 neutrons 8 neutrons

Fig. 21.1

(a) Write down the letter(s) P, Q, R, S, T, U and/or V which represent the


following:

(i) The nuclei of atoms of Group I elements;

…………………………………………………………………………... [1]

(ii) The nucleus of an atom which forms an ion with a 2+ charge;

…………………………………………………………………………... [1]

(iii) Two nuclei from isotopes of the same element;

…………………………………………………………………………... [1]

(iv) The nucleus of an atom of a noble gas.

…………………………………………………………………………... [1]

(b) Elements T and V form an ionic compound with high melting point.

(i) Draw a ‘dot-and-cross’ diagram to show the bonding in the compound


formed by T and V. You need only to show the valence electrons.

[2]

BVSS

www.KiasuExamPaper.com
35
9

(ii) Explain why the compound formed between T and V has a high melting For
examiner’s
point. use

…………………………………………………………………………...

…………………………………………………………………………...

…………………………………………………………………………...

…………………………………………………………………………... [2]

[Total:8]

22 Complete Table 22.1.

Table 22.1

name and formula name of elements present total number of


of substance atoms present

zinc nitrate
Zn(NO 3 ) 2

urea
CO(NH 2 ) 2

[2]

BVSS [Turn over


www.KiasuExamPaper.com
36
10

23 (a) Explain the following observations in terms of kinetic particle theory. For
examiner’s
use
(i) When ice is heated, the temperature stays constant at 0 oC until all the
ice has melted.

…………………………………………………………………………...

………………………………………………………………………….. [1]

(ii) When 1 cm3 of water is heated, the volume of steam obtained is more
than a thousand times larger.

…………………………………………………………………………...

…………………………………………………………………………... [2]

(b) A powder consists of a mixture of three substances is given in Table 23.1.

Table 23.1

solubility of action of hot dilute sulfuric acid


substance
substance in water on substance
copper insoluble does not react
copper(II) oxide insoluble dissolves to give a blue solution
nickel oxide insoluble dissolves to give a green solution

Suggest how a dry sample of copper powder can be obtained from this
mixture.

…………………………………………………………………………………..

…………………………………………………………………………………..

…………………………………………………………………………………..

…………………………………………………………………………………..

………………………………………………………………………………….. [3]

(c) Give two reasons for considering that copper is an element.

1 ………………………………………………………………………………

………………………………………………………………………………

2 ………………………………………………………………………………

……………………………………………………………………………… [2]

[Total:8]

BVSS

www.KiasuExamPaper.com
37
11

24 Chemists use paper chromatography in Fig. 24.1 to compare the food dyes present For
examiner’s
in five different jellies with two approved food dyes, Red No. 3 and Red No. 40. use

movement
of solvent

starting line
Red Red Jelly Jelly Jelly Jelly Jelly
No. 3 No. 40 A B C D E
Fig. 24.1

(a) Which of the jellies contain a mixture of food dyes?

………………………………………………………………………………….. [1]

(b) (i) Which of the jellies contain either one of the approved food dyes?

…………………………………………………………………………... [1]

(ii) Which jellies in (b)(i) is considered safe to consume? Explain your


answer.

…………………………………………………………………………...

…………………………………………………………………………... [2]

(c) Suggest why there is no spot produced by Jelly E.

………………………………………………………………………………….. [1]

(d) Explain why the starting line is not drawn by a pen.

………………………………………………………………………………….. [1]

[Total:6]

BVSS [Turn over


www.KiasuExamPaper.com
38
12

25 A student plans to carry out an experiment to study how temperature affects the For
examiner’s
speed of reaction between zinc and dilute hydrochloric acid. use

The steps are as follows:


(i) Weigh 0.5 g of zinc powder.
(ii) Measure 25 cm3 of dilute hydrochloric acid into a conical flask quickly.
(iii) Measure the time taken for zinc to react completely.
(iv) Rinse the conical flask.
(v) Repeat steps (i) to (iv) using dilute hydrochloric acid warmed to 40 oC
and 50 oC.

(a) Name a suitable apparatus to carry out the following steps.

step procedure name of apparatus

(i) Weigh 0.5 g of zinc powder.

(ii) Measure 25 cm3 of dilute


hydrochloric acid into a conical
flask quickly.

(iii) Measure the time taken for zinc


to react completely.

(iv) Measure temperature of dilute


hydrochloric acid.

[4]

(b) Describe how the particles in zinc powder and dilute hydrochloric acid differ
in their arrangement and movement.

zinc powder dilute hydrochloric acid

arrangement

movement

[2]

[Total:6]

End of Paper

BVSS

www.KiasuExamPaper.com
39
13
The Periodic Table of Elements

BVSS [Turn over


www.KiasuExamPaper.com
40
www.KiasuExamPaper.com
41
Bedok View Secondary School
Mid-Year Examination 2019
Mark Scheme for Sec 3NA Sci(Chemistry)

Section A
1 2 3 4 5 6 7 8 9 10
A B D A D C A A C C

11 12 13 14 15 16 17 18 19 20
D B D A C D C C C B

Section B

Qn Answers R
Rem
Re mark
ark
rks
Remarks Marks
21ai P and R CAO
CA
AO B1
ii T CAO
CAO
O B1
iii Q and S CAO
CA
AO B1
B
iv U CAO
CAO B1
bi Corr
Correct
rrec
ectt sstructure
tructtur
uree B2
of bboth
oth
oth T an and
dV
[1]

T V Correct
Corr charge
of both T and V
[1]

bii P: It h as a g
as
has giian
ant lla
giant attttic
ic e s
lattice stru
ruct
ru ctur
ctu
ct ure
ur
u
structure. e.
B2
R: A lo ot of
lot of energy
ene
nergygy [1] 1]] is needed
ne
eeded to t overcome the strong
electrostatic
trossta
tatitic
tati
tic forces
fo
forc
ces
ces
es of of attraction
attrac between the oppositely-
charged ed io
ions.
onsns.. [1]
[1
1]
O: Thus the compoundcom
om has a high melting point.
T=8
22 B2
name and name of elements total number of
formula of present atoms present
substance
zinc nitrate Zinc, nitrogen 2 box correct
9
Zn(NO 3 ) 2 Oxygen [1m]
urea Carbon, oxygen, all correct [2m]
8
CO(NH 2 ) 2 nitrogen, hydrogen
T=2

www.KiasuExamPaper.com
42
P: During melting, heat is taken in by particles
23ai B1
R: to overcome the strong forces of attraction [1]
between water molecules or particles.
O: thus temperature stays constant during melting.
P: In liquid water, the molecules/particles are close
ii B2
together [1], (R): occupying a small volume.
P: When liquid (water) change to gas (steam), the
molecules/particles are spread far apart [1] and (R):
can occupy a large volume.
O: thus the volume of steam obtained is much larger
than the same volume of water when heated.
Dissolve powder (containing the three substances) in
b B3
hot dilute sulfuric acid. [1]
Filter mixture to obtain insoluble copper as residue. [1]
Wash residue with distilled water and dry it between
filter papers [1]
c copper cannot be broken down into simper B2
substances by chemical means.[1]
copper is made up of only one type of atom.[1] [1]
T=8
T
24a B and D CAO
AO
B1
bi B, C and D (or B and
d C)) B1

ii C and D [1]] CAO


O
B2
C and D contain approved
n appr
apppr
p oved dd dyes.
y
or B cont
contains
tai
ains an nu unknown
nkno
nk wn dye
nown dye other
dy otth
o heer than
th
ha
hann the
an he approved
the ap
pproved d
dyes.
dyess. [1]
[[1
1]
c
Jelly
Je y E is insoluble
inso
solu
so blle in the solvent.
lubl
lu sollv
so veen
nt. B1
d Ink
Ink inn pen
pen
en wi
w
willlll d
dissolve
isso
is s lv
lve
ve in
in tthe
he
h e ssolvent
olvent and
olven and interfere
inte
in terfere with
te
B1
the
th
he re
rresults.
resu
e ltts.ts.
s.
T=6
25a (i)) mass
maas
sss balance
bal
alan
ance
a nc
cee/e electroni
electronic
lectroni
nic
c balance
ba
b / beam balance
1m each B4
(ii) m
measuring
mea asu
asu
s ri
ring
inng cylinder
g cylin
cylinde er (Reject:
( eject: burette,
(Re
(R b pipette)
(iii) stopwatch
topw digital
watch / dig
d igita
igit
ital
it al stopwatch
stopw
(iv) thermometer
ermomo et
om ee
ete
er
b zinc powder dilute
OWTTE B2
hydrochloric acid
particles are particles are
closely packed in closely packed in [1]
arrangement orderly / regular a disorderly
manner manner
particles vibrate particles slide
in fixed position past each other [1]
movement or move freely NOTE: answers
throughout the to compare
liquid difference for
marks to be
awarded
T=6
Note: CAO: Correct Answer Only; OWTTE: Other Word to that effect.

www.KiasuExamPaper.com
43
www.KiasuExamPaper.com
44
CANBERRA SECONDARY SCHOOL

2019 Semestral Assessment 1


Secondary Three Normal Academic

SCIENCE (CHEMISTRY) 13 May 2019


5105/03 1 hour 15 minutes
5105/04 1035h – 1150h

Name: ______________________________________ ( ) Class: ________

READ THESE INSTRUCTIONS FIRST

Do not turn over the paper until you are told to do so.
Write in dark blue or black ink pen on both sides of the paper.
Do not use staples, paper clips, glue or correction fluid.

Answer ALL questions in Paper 3. Indicate your answers on the OTAS provided.
You are advised to spend no more than 30 minutes on Paper 3.

Answer ALL questions in Paper 4 Section A and any TWO questions in Section B.
Write all your answers on the spaces provided.

At the end of the examination, hand in your OTAS and question paper.

The number of marks is given in brackets [ ] at the end of each question or part question.

You may use a calculator for this examination.

A copy of the Periodic Table is printed on Page 15.

FOR MARKER’S USE

Marks Max
Section
Awarded Marks

Paper 3 20
Section A 14
Paper 4
Section B 16
Total 50

This question paper consists of 15 printed pages including the cover page.

Setter: Ms Goh Shu Hui

www.KiasuExamPaper.com
45
2

Paper 3 (20 marks)

Answer all questions in this section.

1 Which apparatus is most suitable to measure 35.10 cm3 of liquid accurately?

A beaker
B burette
C measuring cylinder
D pipette

2 Ammonia gas and hydrogen chloride gas cannot be collected by displacement of


water. They are instead collected by the methods shown below.

hydrogen chloride

ammonia

What deductions can be made about the properties of the two gases?

ammonia hydrogen chloride


density solubility in water density solubility in water
less dense than
A denser than air insoluble insoluble
air
less dense than
B denser than air soluble soluble
air
less dense than
C insoluble denser than air insoluble
air
less dense than
D soluble denser than air soluble
air

____________________________________________________________________________________________________
Canberra Secondary School Chemistry 5105/03, 5105/04
2019 Semestral Assessment 1 Secondary 3 Normal (Academic)
www.KiasuExamPaper.com
46
3

3 The diagram shows some changes in state.

process 1 process 2

solid liquid gas

process 3 process 4

What are the name of the processes?

process 1 process 2 process 3 process 4


A freezing boiling melting evaporation
B melting evaporation freezing condensation
C melting sublimation freezing evaporation
D sublimation evaporation melting condensation

4 Which of the following statements best describes the particles in a solid as the solid is
melting?

A The particles gain energy and vibrate faster.


B The particles gain energy and vibrate slower.
C The particles lose energy and vibrate faster.
D The particles lose energy and vibrate slower.

5 Which substance has particles that are arranged in a closely packed and disorderly
manner at 78 °C?

melting point / °C boiling point / °C


A -185 8
B -63 67
C 0 81
D 98 214

________________________________________________________________________________________________
Canberra Secondary School Chemistry 5105/03, 5105/04
2019 Semestral Assessment 1 Secondary 3 Normal (Academic)
www.KiasuExamPaper.com
47
4

6 The table shows some information about the solubilities of 3 solids, X, Y and Z.

solid solubility in water solubility in alcohol


X insoluble insoluble
Y soluble insoluble
Z insoluble soluble

Some of the following steps could be carried out to obtain pure Z from a mixture of the
three solids.

1 add alcohol
2 add water
3 evaporate the filtrate
4 filter

In what order should the steps be carried out to obtain pure solid Z?

A 1, 4, 3
B 1, 3, 2, 4
C 2, 4, 3
D 2, 1, 4, 3

7 A student found some crystals in the laboratory.

Which of the following methods is suitable to determine the purity of the crystals?

A Measure the size of the crystals.


B Measure the mass of the crystals.
C Measure the solubility of the crystals.
D Measure the melting point of the crystals.

8 A white powder is formed when zinc is completely burned in air.

Which row correctly describes the white powder, zinc and air?

white powder zinc air


A compound element mixture
B compound mixture element
C element compound mixture
D mixture element compound

9 Which of the following shows the correct relative charges of the subatomic particles in
an atom?

proton neutron electron


A +1 0 -1
B +1 -1 0
C 0 -1 +1
D -1 0 +1

____________________________________________________________________________________________________
Canberra Secondary School Chemistry 5105/03, 5105/04
2019 Semestral Assessment 1 Secondary 3 Normal (Academic)
www.KiasuExamPaper.com
48
5

10 The diagram shows the structure of an atom of an element.

4p key
5n z = electron
p = proton
n = neutron

What is the nucleon number of this element and to which Group does it belong to?

nucleon number Group


A 4 IV
B 5 II
C 9 II
D 9 IV

11 An atom of element X has 50 neutrons and 40 protons.Which of the following shows


the correct chemical representation for element X?

A ସ଴
ଽ଴‫܆‬

B ହ଴
ସ଴‫܆‬

C ଽ଴
ସ଴‫܆‬

D ଽ଴
ହ଴‫܆‬

12 The electronic structures of four atoms are shown below.

Which atom is chemically unreactive?

A B

C D

________________________________________________________________________________________________
Canberra Secondary School Chemistry 5105/03, 5105/04
2019 Semestral Assessment 1 Secondary 3 Normal (Academic)
www.KiasuExamPaper.com
49
6

13 The element T has an electronic configuration of 2, 8, 6. Which ion will it form?

A T
B T2
C T
D T2

14 Two particles P and Q have the following composition as shown in the table shown
below.

particle number of protons number of neutrons number of electrons


P 13 14 10
Q 20 20 18

Which statement describes particles P and Q?

A P and Q are both positive ions.


B P and Q are both negative ions.
C P and Q are both non-metal atoms.
D P and Q are isotopes of the same element.

15 Which one of the following pairs of elements will form an ionic compound?

A chlorine and oxygen


B copper and zinc
C neon and fluorine
D potassium and bromine

16 Which of the following is most likely to be an ionic compound?

melting point / °C boiling point / °C electrical conductivity in solid state


A -205 -183 poor
B -153 50 good
C 1057 2050 poor
D 1798 2978 good

17 The structural formula of ethanol is shown below.

How many electrons are shared in one molecule of ethanol?

A 8
B 14
C 16
D 26
____________________________________________________________________________________________________
Canberra Secondary School Chemistry 5105/03, 5105/04
2019 Semestral Assessment 1 Secondary 3 Normal (Academic)
www.KiasuExamPaper.com
50
7

18 The electronic configuration of elements X and Y are shown below.

particle electronic configuration


X 2, 8, 1
Y 2, 6

X and Y will react with each other to form a compound.

What is its chemical formula?

A XY
B XY 2
C X2Y
D YX 2

19 The equation represents the reaction between silver nitrate and magnesium chloride.

xAgNO 3 + MgCl 2 Æ yAgCl + Mg(NO 3 ) 2

What are the values of x and y?

x y
A 1 1
B 2 2
C 1 2
D 2 1

20 A molecule CH 3 CO 2 X has a relative molecular mass of 82.

What is the identity of element X?

A antimony
B lead
C sodium
D vanadium

End of Paper 3

________________________________________________________________________________________________
Canberra Secondary School Chemistry 5105/03, 5105/04
2019 Semestral Assessment 1 Secondary 3 Normal (Academic)
www.KiasuExamPaper.com
51
8

Paper 4
Section A (14 marks)

Answer all the questions in this section.


Write your answers in the spaces provided.

A1 The diagrams A, B, C, D, E and F represent the particles in different substances.

A B C

D E F

(a) State the diagram which best represent

(i) a mixture of an element and a compound; …………………………… [1]

(ii) a pure compound; …………………………… [1]

(iii) a mixture of elements. …………………………… [1]

(b) A is a diatomic gas while E is a monatomic gas. Suggest a possible identity


of E.

……………………………………………………………………………………… [1]

A2 Magnesium oxide is used to make refractory linings for blast furnaces because of
its high melting point.

(a) (i) Write the symbols for magnesium ion and oxide ion.

magnesium ion …………………………….

oxide ion ……………………………. [1]

(ii) Hence, deduce the formula for magnesium oxide.

……………………………………………………………………………….. [1]

____________________________________________________________________________________________________
Canberra Secondary School Chemistry 5105/03, 5105/04
2019 Semestral Assessment 1 Secondary 3 Normal (Academic)
www.KiasuExamPaper.com
52
9

(b) Draw the ‘dot and cross’ diagram to show the bonding in magnesium oxide.

[2]

(c) In terms of structure and bonding, explain why magnesium oxide has a high
melting point.

………………………………………………………………………………………

………………………………………………………………………………………

……………………………………………………………………………………… [2]

(d) Molten magnesium oxide is able to conduct electricity. Explain why.

………………………………………………………………………………………

……………………………………………………………………………………… [1]

A3 Solid calcium hydroxide, Ca(OH) 2 , reacts with dilute hydrochloric acid, HCl, to
form aqueous calcium chloride, CaCl 2 , and water.

(a) Balance the chemical equation for the reaction and fill in the missing state
symbols.

……. Ca(OH) 2 (s) + ……. HCl ( ) Æ ……. CaCl 2 (aq) + ……. H 2 O ( ) [2]

(b) Calculate the relative formula mass, M r , of calcium hydroxide.

relative formula mass = ………………………………… [1]

End of Section A

________________________________________________________________________________________________
Canberra Secondary School Chemistry 5105/03, 5105/04
2019 Semestral Assessment 1 Secondary 3 Normal (Academic)
www.KiasuExamPaper.com
53
10

Paper 4
Section B (16 marks)

Answer any TWO questions in this section.


Write your answers in the spaces provided.

B1 The ‘dot and cross’ diagram shows the arrangement of all the electrons in a
molecule of compound Z.

1p

key
6p x and z = electron
1p 1p p = proton

1p

(a) Using the Periodic Table, write the chemical formula of Z. ………………… [1]

(b) What type of bonding is present in Z?

……………………………………………………………………………………… [1]

(c) The heating curve of Z is given below.

temperature / qC

time / min

R S
-162

P Q
-182

(i) Based on the heating curve, deduce the boiling point of Z.

boiling point = ………………………….. qC [1]

____________________________________________________________________________________________________
Canberra Secondary School Chemistry 5105/03, 5105/04
2019 Semestral Assessment 1 Secondary 3 Normal (Academic)
www.KiasuExamPaper.com
54
11

(ii) Explain why Z has a low boiling point.

………………………………………………………………………………..

………………………………………………………………………………..

……………………………………………………………………………….. [1]

(iii) Deduce the physical state(s) of Z at the following regions.

PQ …………………………………………………….

QR ……………………………………………………. [2]

(d) Carbon dioxide, CO 2 , has the same type of bonding as Z. Draw a ‘dot and
cross’ diagram to show the arrangement of all the electrons in a molecule of
carbon dioxide.

[2]

________________________________________________________________________________________________
Canberra Secondary School Chemistry 5105/03, 5105/04
2019 Semestral Assessment 1 Secondary 3 Normal (Academic)
www.KiasuExamPaper.com
55
12

B2 Eucalyptol is one of the active ingredients found in eucalyptus oil which is used as
an essential oil for aromatherapy. Chromatography was done on four different
brands of essential oils, A, B, C and D, using alcohol as the solvent. The results
are shown in the chromatogram below.

start line
eucalyptol A B C D

(a) State the essential oil(s) containing eucalyptol as one of the ingredients.

……………………………………………………………………………………… [1]

(b) State and explain whether essential oil D is a pure substance or a mixture.

………………………………………………………………………………………

……………………………………………………………………………………… [2]

(c) Explain why the start line must be drawn using a pencil instead of a pen.

………………………………………………………………………………………

……………………………………………………………………………………… [1]

____________________________________________________________________________________________________
Canberra Secondary School Chemistry 5105/03, 5105/04
2019 Semestral Assessment 1 Secondary 3 Normal (Academic)
www.KiasuExamPaper.com
56
13

(d) Eucalyptol has a melting and boiling point of 5 °C and 176 °C respectively.
It is separated from eucalyptus oil using the following separation technique
shown in the diagram below.

thermometer
water out
X condenser

fractionating
column

water in
conical
round-bottom flask
flask
Y
eucalyptus oil
boiling
chips
heat

(i) Name the separation technique used to obtain eucalyptol.

……………………………………………………………………………….. [1]

(ii) State the temperature observed on the thermometer when eucalyptol


is being collected in the conical flask.

temperature = …………………………. °C [1]

(iii) In the boxes below, draw diagrams to show how the particles are
arranged at positions X and Y.

position X Y

diagram of
particles

[2]

________________________________________________________________________________________________
Canberra Secondary School Chemistry 5105/03, 5105/04
2019 Semestral Assessment 1 Secondary 3 Normal (Academic)
www.KiasuExamPaper.com
57
14

B3 The table gives information about four atoms, R, S, T and U.

atom proton number nucleon number electronic configuration

R 38 2, 8, 6

S 16 32

T 19 38

U 30 2, 8, 5

(a) Complete the table above. [2]

(b) Which atom has 22 neutrons?

……………………………………………………………………………………… [1]

(c) Using the Periodic Table and the table above, name the element that atom
T represents.

……………………………………………………………………………………… [1]

(d) Another atom Q has 7 electrons.

Which atom (R, S, T or U) is in the same group as atom Q?

……………………………………………………………………………………… [1]

(e) (i) Define the term ‘isotopes’.

………………………………………………………………………………..

……………………………………………………………………………….. [1]

(ii) Which two atoms (R, S, T or U) are isotopes?

……………………………………………………………………………….. [1]

(f) T forms ionic compounds to achieve stable electronic configuration.

Describe, in terms of electrons, how atom T becomes an ion.

………………………………………………………………………………………

……………………………………………………………………………………… [1]

End of Paper 4

____________________________________________________________________________________________________
Canberra Secondary School Chemistry 5105/03, 5105/04
2019 Semestral Assessment 1 Secondary 3 Normal (Academic)
www.KiasuExamPaper.com
58
15

________________________________________________________________________________________________
Canberra Secondary School Chemistry 5105/03, 5105/04
2019 Semestral Assessment 1 Secondary 3 Normal (Academic)
www.KiasuExamPaper.com
59
www.KiasuExamPaper.com
60
Answer Scheme

CANBERRA SECONDARY SCHOOL

2019 Semestral Assessment 1


Secondary Three Normal Academic

SCIENCE (CHEMISTRY) 13 May 2019


5105/03 1 hour 15 minutes
5105/04 1035h – 1150h

Name: ______________________________________ ( ) Class: ________

READ THESE INSTRUCTIONS FIRST

Do not turn over the paper until you are told to do so.
o.
Write in dark blue or black ink pen on both sides
es of the pappaper.
perr.
orrectio
io fluid.
ion
io
Do not use staples, paper clips, glue or correction d..

Answer ALL questions in Paper


er 3. Indicate
Indica
cate
cate yourr an
answers
n errs on
answer on the
thee OTAS
OTA provided.
T S pr
prov
ovided.
ov
You are advised to spend no mo
ore
re than
more tha
han 30 m inutes
es o
es
minutes n Paper 3
on 3..

Answer ALL questions


estions iin
nPPaper
Pa p r 4 Section
ape S ctio
Se on A and any
dany TWO
an WO questions
TW
T ques ns in Section
estion Sectio B.
Se
Write all your ans
answers
nsswe
wers on tthe
n th sspaces
he sppac e provided.
aces pro
rovviided.
d.
d.

At the end
he end off tthe
en examination,
he exa
ami
m na
atio , hand
tion nd in
nd in your
yyourr OTAS
ou TAS and
OTA nd question
an estion paper.
ques
ques

The number
er of
mbe
be marks
of mar
arks
ark is gi
given
giv
ve
en in
nbbrackets
rackets [ ] at tthe
ack he end of each question or part question.

You may use


se a calculator
callcu
c lator for
lato or this
ffo hiss examination.
th exa
xami
m nati

A copy of the Periodic


c Table
Tabl
Table
bl printed on Page 15.
e is pri

FOR MARKER’S USE

Marks Max
Section
Awarded Marks

Paper 3 20
Section A 14
Paper 4
Section B 16
Total 50

This question paper consists of 15 printed pages including the cover page.

Setter: Ms Goh Shu Hui

www.KiasuExamPaper.com
61
2

Paper 3 (20 marks)

Answer all questions in this section.

1 Which apparatus is most suitable to measure 35.10 cm3 of liquid accurately?

A beaker
B burette
C measuring cylinder
D pipette

2 Ammonia gas and hydrogen chloride gas cannot be collected by displacement of


water. They are instead collected by the methods shown below.

hydrogen chloride

ammonia
am
mmo
moni
na

ctions can
What deductions can be
ca b made
mad
ad
ade about
ab
abou
out the
th
he properties
prrop
p operrttiies of the
e two
tw ga
gases?
ase
ses?
s?

am
mmo
moni
na
ammonia hydroge
hy
hydrogen chloride
density
dens
density
ns y solubility
so
s olu
olu
lubi
b liity
ty iin
n wa
w
water
ate
ter de
density solubility in water
le dense
less d than
A denser
de
dens
nser than
than
ha
an airr insoluble
insollublle insoluble
air
less dense than
B denser
de
dens
n er than
tha
an air
aiir
a soluble
solu
solubl
be soluble
air
les
less
lesss dense
den
nse
se tthan
han
C insoluble
ins denser than air insoluble
airr
a
ai
de
ens
nse
less dense e th
than
D soluble denser than air soluble
airr
ai

____________________________________________________________________________________________________
Canberra Secondary School Chemistry 5105/03, 5105/04
2019 Semestral Assessment 1 Secondary 3 Normal (Academic)
www.KiasuExamPaper.com
62
3

3 The diagram shows some changes in state.

process 1 process 2

solid liquid gas

process 3 process 4

What are the name of the processes?

process 1 process 2 process 3 process 4


A freezing boiling melting evaporation
B melting evaporation freezing condensation
C melting sublimation freezingg evaporation
D sublimation evaporation melting
elting condensation
co

4 Which of the following statements best describes


escribes the particles
par
a ticl
cles
les in a so
soli
solid
lilid
daas
s the solid is
melting?

A The particles gain energy andan


and vibrate
vibratate
at e faster.
f stter
fa er.
B The particles gain energy
en
ne rgy and
erg
erg nd vibrate
an
a v brate
vi e slower.
s ower.
sl
C The particles
cles lose energy
en gyy and
energy and vibrate
nd vibbratee faster.
fas
a ter.
D The particles
particle llose
es loose energy
en
ne gy and
errg nd vibrate
an viibr ate slower.
brat slow
ower.
ow
owe

5 Which
hich substance
Wh su
ubsta
an ce has
nc h s particles
ha partic les that
iccle tthat are
hat e arranged
arre arr ed in a closely
rrange closel packed and disorderly
manner
ma
ann e at
nner
nner at 78 °C
°C?
C?

melting
me
elting
gp point
pooiin
nt / °C
°C boiling
oiling point / °C
bo
A -185
-18
18
18
855 8
B --63
6
63 67
C 0 81
D 98 214

________________________________________________________________________________________________
Canberra Secondary School Chemistry 5105/03, 5105/04
2019 Semestral Assessment 1 Secondary 3 Normal (Academic)
www.KiasuExamPaper.com
63
4

6 The table shows some information about the solubilities of 3 solids, X, Y and Z.

solid solubility in water solubility in alcohol


X insoluble insoluble
Y soluble insoluble
Z insoluble soluble

Some of the following steps could be carried out to obtain pure Z from a mixture of the
three solids.

1 add alcohol
2 add water
3 evaporate the filtrate
4 filter

In what order should the steps be carried out to obtain pure solid
olid Z?

A 1, 4, 3
B 1, 3, 2, 4
C 2, 4, 3
D 2, 1, 4, 3

7 ystals in tthe
A student found some crystals he
he labor
raattor
o y.
laboratory.

owing met
Which of the following methods
etth
e ho
o
ods is
is suitable
suitititab
su able to
ab o determine
d termine the
de th
he purity
purity
y off the
th
he cr
crys
crystals?
y tals?

A Measu
su
ure the
Measure he
he size
siz
ize of the hee crystals.
cryyssttal
a s.
B Measure
Mea
e su r the
sure e mass
maasss of of the he crystals.
the crry
ystal allss..
C Measure
Measu e the
urre e solubility
sol
olub
ubilililitity o
off tthe
hee crystals.
cryst
rry
yst
sta allss..
D Measure
Measure
re
e the
the
e melting
melting p point
on
oi ntt of
of the
tth
he cr
he ccrystals.
rystalslss.

8 A whit
itte po
white p wder is
powder s fformed
orrm
o meed wh
when zzinc
when incc is
in i co
ccompletely
mpl burned in air.

h row
Which ro
ow correctly
corr
corrrec
ectl
tly describes
desc
scri
scribe
ribe
bes the white
w powder, zinc and air?

e po
white powd
wder
wd
powder zinc air
A compound
co
omp
mpound element mixture
B compound
comp mixture element
C element compound mixture
D mixture element compound

9 Which of the following shows the correct relative charges of the subatomic particles in
an atom?

proton neutron electron


A +1 0 -1
B +1 -1 0
C 0 -1 +1
D -1 0 +1

____________________________________________________________________________________________________
Canberra Secondary School Chemistry 5105/03, 5105/04
2019 Semestral Assessment 1 Secondary 3 Normal (Academic)
www.KiasuExamPaper.com
64
5

10 The diagram shows the structure of an atom of an element.

4p key
5n z = electron
p = proton
n = neutron

What is the nucleon number of this element and to which Group does it belong to?

nucleon number Group


A 4 IV
B 5 II
C 9 II
D 9 IV

11 An atom of element X has 50 neutrons s and 40 prot


protons.Which
ton
nss.. W
Whhic
ich of the
the following
fol
olllowing shows
the correct chemical representation
on for el
element
elem
emen X?
nt X ?

A ସ଴
ଽ଴‫܆‬

B ହ଴
ସ଴‫܆‬

C ଽ଴
଴‫܆‬
ସ଴

D ଽ଴
ହ଴‫܆‬
ହ଴

12 The
e electronic
e ec
ele tr
tron
onic
ic structures
strructure
res of
re of four
fou
our atoms are
are shown
sh below.
b

ich atom
Which ato
at om
m iss chemically
ch
c hem
miicca
allllyy unr
nrrea
eact
ctiv
ct ive?
iv
unreactive? ?

A B

C D

________________________________________________________________________________________________
Canberra Secondary School Chemistry 5105/03, 5105/04
2019 Semestral Assessment 1 Secondary 3 Normal (Academic)
www.KiasuExamPaper.com
65
6

13 The element T has an electronic configuration of 2, 8, 6. Which ion will it form?

A T
B T2
C T
D T2

14 Two particles P and Q have the following composition as shown in the table shown
below.

particle number of protons number of neutrons number of electrons


P 13 14 10
Q 20 20 18

Which statement describes particles P and Q?

A P and Q are both positive ions.


B P and Q are both negative ions.
C P and Q are both non-metal atoms.
D P and Q are isotopes of the same
me element.

15 Which one of the following


ng pairs
rs o
off el
elem
elements
e en
nttss w
will
ill fo
form
orrm
maan
n io
ionic
oniic comp
compound?
mpou
mpound?
ou

A chlorine and oxygen e


en
B pper and
copper d zinc
ziinc
n
C neon and
a d fluorine
an flluo
uorriin
ne
e
D potassium
po
pota
t ss
ssiu
ium and
an
and bromine
brrom
b omiin
ne

16 Wh
Which
i h of
hic of the
the following
he fol
o lo
owi
w ng
n iss m
mo
most
ost
st likely
lik
i ely to b
be
e an iionic
o ic co
on compound?

me
m eltting
melting g point
po
poiin
nt / °°C
C boilililin
bo
boiling ing
ing point / °C electrical conductivity in solid state
A -20
-205
20
2
205
05 -183
-18 poor
B --153
15
153 50 good
C 105757 2050 poor
D 1798
179
7988 2978 good

17 The structural formula of ethanol is shown below.

How many electrons are shared in one molecule of ethanol?

A 8
B 14
C 16
D 26
____________________________________________________________________________________________________
Canberra Secondary School Chemistry 5105/03, 5105/04
2019 Semestral Assessment 1 Secondary 3 Normal (Academic)
www.KiasuExamPaper.com
66
7

18 The electronic configuration of elements X and Y are shown below.

particle electronic configuration


X 2, 8, 1
Y 2, 6

X and Y will react with each other to form a compound.

What is its chemical formula?

A XY
B XY 2
C X2Y
D YX 2

19 The equation represents the reaction between silver nitrate


rate and
d ma
magnesium
agn
gnesi chloride.

xAgNO 3 + MgCl 2 Æ yAgCl


AgCl + Mg
Mg(NO
g(N
(NO
O3)2

What are the values of x and y?

x y
A 1 1
B 2 2
C 1 2
D 2 1

20 A mo
m
molecule
le
ecu
ule CH
H3C
CO
O 2 X has a relative
rela
relati
tivve
e molecular
mol
olec
ecular m
mass
asss of
as o 82.
82

What
at iss the
at th
he identity
iid
de
en
ntiity
t of e
el
le
em
men
nt X?
element X?

A an
a nti
timoony
antimonyny
B le
ead
ad
lead
C sodi
diu
diium
sodiumum
D anadium
vanadiumum
m

End of Paper 3

________________________________________________________________________________________________
Canberra Secondary School Chemistry 5105/03, 5105/04
2019 Semestral Assessment 1 Secondary 3 Normal (Academic)
www.KiasuExamPaper.com
67
8

Paper 4
Section A (14 marks)

Answer all the questions in this section.


Write your answers in the spaces provided.

A1 The diagrams A, B, C, D, E and F represent the particles in different substances.

A B C

D E F

(a) m whic
State the diagram ich be
which best
st repre
ese
ent
represent

(i) a mixture off a


an
n ele
le
eme
ment an
element and a co
and ccompound;
ompound; C [1]

(ii) a pu
pure
ure
e co
ccompound;
omp
mpou
un
nd
d; F [1]

((iii)
(iii) a mixture
m xt
mixu urre of e
elements.
le
eme
menttss.. B [1]

(b)
(b
b) A is
is a dia
ia
atomi
to
om c g
diatomic ga
as while
gas whiille E is
wh i am onat
on ato
at omic g
monatomic gas. Suggest a possible identity
of EE..
helium
h
he lium / n
li neon argon
eon / a
eo ar
rgo
g n/k krypton
kr
ryp on / x
ypto xenon / radon
………………………………………………………………………………………
……
… ……………
……
… ………… …………… ……………
…… [1]

A2 Magnesium m oxide
esium oxid
ox ide
ide is used
use to make refractory linings for blast furnaces because of
its high mel
melting
ltin point.
ting poi

(a) (i) Write the symbols for magnesium ion and oxide ion.

magnesium ion Mg2+

oxide ion O2- [1]

(ii) Hence, deduce the formula for magnesium oxide.


MgO
……………………………………………………………………………….. [1]

____________________________________________________________________________________________________
Canberra Secondary School Chemistry 5105/03, 5105/04
2019 Semestral Assessment 1 Secondary 3 Normal (Academic)
www.KiasuExamPaper.com
68
9

(b) Draw the ‘dot and cross’ diagram to show the bonding in magnesium oxide.

Mg O

key
y – electron of Mg
x – electron of O

1 m – correct transfer of electrons [2]


1m – correct arrangement of electrons
(A: no key; gained electrons of O are
e paired ttogether;
oget
ogethe
et herrr;; oxid
he oxide before
magnesium)

(c) In terms of structure and bonding,


nding,
g, explain
g, nwwhy magnesium
hy ma
hy m agn
gnesiuum oxide
ide has a high
oxid
ox id
melting point.
Magnesium oxide e is an iio
ionic
oni
n c co
compound.
omp
m ou und.
nd
n d. A large
larg
larg
la ge am
amou
amount
ount
ou nt of
of energy
………………………………………………………………………………………
…………… … ……………… …… ………… ………… ………… …… ……… ………… ……… … …
is required
ed [1] to
o overcome
ov
o verco
come
co e the
h strong
the strronng electros
electrostatic
staatic forc
fo
forces
orc
r essooff attr
a
at
attraction
tttrractio
………………………………………………………………………………………
……………………
…… ………
… … ……………………… ……… ………
… …… ………… …… ……………
between the
the oppositely
th opposite
op charged
tely c haarged ions
iio
on
nss [1].
[1
[1
………………………………………………………………………………………
……
… ………
… ……………… … … …… ……… ……… ………
…………
…… …… …………… ………… …… [2]

(d)
(d
(d) M olten
Moltennm aggne
nesium oxide
magnesium oxi
xide
de is
is able
ab
able
le to
to conduct
cond
duc
uctt electricity.
electric
el Explain why.
M
Moolten m
Molten ag
gne
n sium
magnesium um
um oxide
oxixide contains
con
o taain
ins mobile
mo
m obi
b le iions that can carry charges/
………………………………………………………………………………………
…… ……… …………… ……
… ………
……………… ………
… ……
ions
/ have ioons tthat
ns ha
h are
at ar ffree
re fr eee tto
ommove
moove to carry charges.
………………………………………………………………………………………

…… ……
… ………
……………………… ………
…… ………… [1]

A3 Solid calcium
m hy
hydr
dro
dr oxide Ca(OH) 2 , reacts with dilute hydrochloric acid, HCl, to
hydroxide,
form aqueous
ueou
ouss calcium
calciu chloride, CaCl 2 , and water.

(a) n the chemical equation for the reaction and fill in the missing state
Balance
symbols.

……. Ca(OH) 2 (s) + 2 HCl ( aq ) Æ ……. CaCl 2 (aq) + 2 H 2 O ( l ) [2]


1 m – correctly balanced equation; 1 m – correct state symbols

(b) Calculate the relative formula mass, M r , of calcium hydroxide.

M r of Ca(OH) 2 = 40 + 2(16 + 1) = 74
0 m – no working
relative formula mass = 74 [1]

End of Section A

________________________________________________________________________________________________
Canberra Secondary School Chemistry 5105/03, 5105/04
2019 Semestral Assessment 1 Secondary 3 Normal (Academic)
www.KiasuExamPaper.com
69
10

Paper 4
Section B (16 marks)

Answer any TWO questions in this section.


Write your answers in the spaces provided.

B1 The ‘dot and cross’ diagram shows the arrangement of all the electrons in a
molecule of compound Z.

1p

key
y
6p x and z = el
ele
electron
ectro
1p 1p p = pr
proton
prot
oton
on

1p

(a) Using the


e Periodic
Pe
erriiod
o ic Table,
Table,
b write
wriite th
the
he chemical
che
hemi
mica
ic l formula
form
mul
ula
la of Z
Z.. CH 4 [1]

(b) Wh
W at type
What typ
y e off bonding
bondi
on
nd
diing
ng is pres
ese
esen
nt in
present nZ ?
Z?
c
covale
lent
lent b
covalent on nding
bonding
………………………………………………………………………………………
… ……… ……………………… ……
… ………………… ………
………… [1]

(c)) The heating


he
heatin
ng curve
cu
curv
rve of
of Z is given
iv en below.
given bel
e ow

em
emp
mpper
p urre / qC
temperature
te eratur
er
eratur
u C

time / min

R S
-162

P Q
-182

(i) Based on the heating curve, deduce the boiling point of Z.


-162
boiling point = ………………………….. qC [1]

____________________________________________________________________________________________________
Canberra Secondary School Chemistry 5105/03, 5105/04
2019 Semestral Assessment 1 Secondary 3 Normal (Academic)
www.KiasuExamPaper.com
70
11

(ii) Explain why Z has a low boiling point.


Only a small amount of energy is required to overcome the weak
………………………………………………………………………………..
intermolecular forces of attraction between the molecules.
………………………………………………………………………………..

……………………………………………………………………………….. [1]

(iii) Deduce the physical state(s) of Z at the following regions.

PQ solid and liquid

QR liquid [2]

(d) Carbon dioxide, CO 2 , has the same type of bonding as Z. D Draw a ‘dot and
cross’ diagram to show the arrangement of all the electrons
trons in a molecule of
carbon dioxide.

C O
O

key
ke
k y
electron
x = elecctrron off C
on o
y = el
e
electron
leec
c
ctr
tron
tr o of
of O
[2]
1m–c correct
coo
orr
rrrect sharing
ec sh
sharinng of
of electrons
electro
1m–c correct
coorr
rre
rrec
ec arrangement
ct ar
rrang
ra
ang ement of electrons
gem
-1
1 – showing
sh
howin only
ng onnly valence electrons
y valen
(A:
A: no key)
(A key
eyy)

________________________________________________________________________________________________
Canberra Secondary School Chemistry 5105/03, 5105/04
2019 Semestral Assessment 1 Secondary 3 Normal (Academic)
www.KiasuExamPaper.com
71
12

B2 Eucalyptol is one of the active ingredients found in eucalyptus oil which is used as
an essential oil for aromatherapy. Chromatography was done on four different
brands of essential oils, A, B, C and D, using alcohol as the solvent. The results
are shown in the chromatogram below.

start line
eucalyptol A B C D

(a) State the essential oil(s) containing eucalyptol


calyptol ass one
one of
of the
th ingredients.
ingredi
A and C
………………………………………………………………………………………
……………… ……… …… …
…… …………………… …… ………… [1]

(b) State and explain whether


hetherr essential
e sent
es nttiia
ntia al oil
oil D is
oi is a pu
p
pure
ure
re ssubstance
ubst
ubstan
st ance
an ce or a mixture.
mixture
Pure substance.e. [1]] It
It contains
con tains only
onta
on
nta only
on y one
nly e component.
ne compo
po nent. / Itt shows
one shows
………………………………………………………………………………………
……… ………… …… … … …………… ………… ………… ………
… …… ………… ………
……………
only one
ne spot on
n tthe
he
h ec chromatogram
hro
hrom
hr om
mat
atog
ogra ram [1].
………………………………………………………………………………………
…………… …………
…… … ………… ………… ………… …………… …………… ………
…… ……… [2]

(c) E
Ex plai
plaiin wh
a
Explain whyhy thhe st
the sstart
ta
arrt lliline
in
neem usst b
must be
ed raw
rawn
drawn n using
gap enciil in
en
pencil instead of a pen.
P
Pe
Pen n in
inkk coonttai
a ns a m
contains mi
mixtureixt
xtu
urre of
of dy
dyes
e which
dyes whic
ch wi
w
wil
will
ill sep
separate out with the
………………………………………………………………………………………
………… ……………………… ……… ……… …… …
………
……
… …………
………………
solvent
so
s olv e t and
lven and interfere
an inte
terf
te erre with
rfe with thee results.
res
esul
ults
t .
………………………………………………………………………………………
………… … …… ……… ………… …… …………… …………
…… … … [1]
(R:
((R affect,
R: af
a fe
ec smudge,
ctt, sm
smu uddge, spread)
sp pre
ead
ad)

____________________________________________________________________________________________________
Canberra Secondary School Chemistry 5105/03, 5105/04
2019 Semestral Assessment 1 Secondary 3 Normal (Academic)
www.KiasuExamPaper.com
72
13

(d) Eucalyptol has a melting and boiling point of 5 °C and 176 °C respectively.
It is separated from eucalyptus oil using the following separation technique
shown in the diagram below.

thermometer
water out
X condenser

fractionating
column

n
water in
conical
round-bottom flask
flask
Y
eucalyptus oil
boilin
boiling
ng
chipss
heat
he
ea
att

(i) Name
me tthe
me h sep
he separation
epa
eparrat atio
ion
on technique
tecch
te hniq
qu
ue
e used
used
sse
ed to obtain
obt
btai
bt ain
ain eucalyptol.
eu
uca
caly
lypt
ly pto
pt ol.
fractional
frac
cti
t on al distillation
onal disti
tiillllatio
on
………………………………………………………………………………..

……………………… ……… …………… ……… …… ………
… ………… ……………
…… [1]

((ii)
ii) State
St
S tat the
ate th temperature
he temp per
ea ure observed
attur obse
ob r ed on
serv on the
the thermometer
t erm
th when eucalyptol
iss being
be
eiing
ng co
collected
ollle
ec
cte
ted in the
he con
he conical
on
nical
iccal fflask.
lask.
la
176
temperature
tem = …………………………. °C [1]

(iii)
(iii
(iiiii) In
n the
the boxes
th bo
oxe
xess below,
be draw diagrams to show how the particles are
arranged
arrang
ngged at
at positi
positions X and Y.

positi
position X Y

diagram of
particles

[2]

(R: sketchy particles, incomplete circles, circles of different


sizes, box less than ¾ filled)

________________________________________________________________________________________________
Canberra Secondary School Chemistry 5105/03, 5105/04
2019 Semestral Assessment 1 Secondary 3 Normal (Academic)
www.KiasuExamPaper.com
73
14

B3 The table gives information about four atoms, R, S, T and U.

atom proton number nucleon number electronic configuration

R 16 38 2, 8, 6

S 16 32 2, 8, 6

T 19 38 2, 8, 8, 1

U 15 30 2, 8, 5

(a) Complete the table above. [2]

(b) Which atom has 22 neutrons?


R
……………
………
…………………………………………………………………………………………………
…………… [1]

(c) abo
bo
ove
ve, name
Using the Periodic Table and the table above, name
name the
e element
ele
leme
men
ment that atom
T represents.
potassium
………………………………………………………………………………………
……… …………………
… …… ………
… …… ……… ………
………… ………………… … … [1]

(d) Anotherr atom Q has


has 7 electrons.
ha elec
electr
tron
onss..
on

Whhicch atom
Which om ((R,
om R, S,
S, T or
or U)
U) is
is in the sam
ssame
saame
me group
group a
as
s at
atom
om Q?
Q?
U
………………………………………………………………………………………
… …… ……………
……………… ………… ……
…………… ……………………… ………
…… ……… [1]

(e)
(e
e) (i)
((ii)
i) Define
De n the
effiine e term
errm ‘isotopes’.
te ‘i‘iso
s topes’..
Isotopes
Isotop
Is ope
op s are
es re atoms
are atoms off tthe
at he same element with the same number
he
………………………………………………………………………………..

…… ……… …………………… …………
……
of p
of protons
rrotons but
oto bu different
ut diff
d
di f erent number of neutrons.
iff
………………………………………………………………………………..
… ……… ………
… …… [1]

(ii) Which
Whic
Wh ich
ic h two atoms are isotopes?
R and S
……………………………………………………………………………….. [1]

(f) T forms ionic compounds to achieve a stable electronic configuration.

Describe, in terms of electrons, how atom T becomes an ion.


T loses 1 valence electron to become T+.
………………………………………………………………………………………

……………………………………………………………………………………… [1]

End of Paper 4

____________________________________________________________________________________________________
Canberra Secondary School Chemistry 5105/03, 5105/04
2019 Semestral Assessment 1 Secondary 3 Normal (Academic)
www.KiasuExamPaper.com
74
15

________________________________________________________________________________________________
Canberra Secondary School Chemistry 5105/03, 5105/04
2019 Semestral Assessment 1 Secondary 3 Normal (Academic)
www.KiasuExamPaper.com
75
www.KiasuExamPaper.com
76
Class Index Number

Name:

Jurong West Secondary School


Mid-Year Examinations 2019 20

SCIENCE (CHEMISTRY) 5105/5107/03


Secondary Three Normal (Academic) 14 May 2019
Paper 3 0800 - 0915
Paper 3 and 4: 1 hour 15 minutes
Candidates answer on the Question Paper.

READ THESE INSTRUCTIONS FIRST

Write your name, class and index number on all the work you hand in.
Write in dark blue or black pen.
You may use an HB pencil for any diagrams, graphs, tables or rough working.
Do not use staples, paper clips, glue or correction fluid.

The use of an approved scientific calculator is expected, where appropriate.


You may lose marks if you do not show your working or if you do not use appropriate units.

There are twenty questions. Answer all questions. For each question there are four possible
answers A, B, C and D. Choose the one you consider correct and record your choice in soft
pencil on the separate Answer Sheet. Read the instructions on the Answer Sheet very
carefully. Each correct answer will score one mark. A mark will not be deducted for a wrong
answer. Any rough working should be done in this question paper.

You are advised to spend no more than 30 minutes on Paper 3.


You may proceed to answer Paper 4 as soon as you have complete Paper 3.

A copy of the Periodic Table is printed on page 8.

After checking of answer script


Checked by Signature Date
Student

This document consists of _8_ printed pages.

Setter: Ms Kek Poh Khee www.KiasuExamPaper.com


77
2

1 Which change occurs when a liquid changes state to a gas?

A The particles lose energy.


B The particles move further apart.
C The particles become bigger in size.
D The particles have stronger forces of attraction.

2 The graph shows the cooling curve of a substance.

At Y, the particles are sliding over one another.

Which of the following can the substance be?

Melting point/oC Boiling point/oC


A 24 89
B -35 15
C -20 175
D -89 -30

3 Which of the following represents condensation?

A C

B D

JWSS Mid-Year Examinations 2019 Science (Chemistry) 5105/5107/03 Secondary 3NA

www.KiasuExamPaper.com
78
3

4 A student wishes to accurately add 24.15 cm3 of acid to a fixed


volume of solution of 25.0 cm3.

Which apparatus should the student use to measure these volumes?

Acid Solution
A Burette Pipette
B Pipette Burette
C Measuring cylinder Pipette
D Burette Measuring Cylinder

5 A gas, X, is more dense than air and insoluble in water.

Which method cannot be used to collect the gas?

A C

B D

6 Which property must be present in order for different substances to be


separated by paper chromatography?

A Substances must be coloured.


B Substances must be in liquid state at room temperature.
C Substances must have different boiling points.
D Substances must have different solubilities in the same solvent.

JWSS Mid-Year Examinations 2019 Science (Chemistry) 5105/5107/03 Secondary 3NA

www.KiasuExamPaper.com
79
4

7 The diagram below shows a set-up used to separate a mixture of ethanol


(boiling point = 78oC) and water (boiling point = 100oC).

Heat

Which of the following best describes the purpose of apparatus X?

A To measure the temperature of the vapour.


B To allow both ethanol and water to boil together.
C To condense the ethanol vapour and allows it to re-enter the round-
bottom flask.
D To prevent water from escaping the round-bottom flask before its boiling
point.

8 A mixture consists of two solids.

The following procedures were performed to separate the mixture.


Step 1: Excess water is added to a beaker containing the mixture.
Step 2: The mixture is filtered.
A green filtrate and a white residue is obtained.

Which of the following shows the colour and solubility in water of the two
solids present in the mixture?

Solid 1 Solid 2
A Green Insoluble White Soluble
B Green Soluble White Insoluble
C Green Insoluble White Insoluble
D Green Soluble White Soluble

JWSS Mid-Year Examinations 2019 Science (Chemistry) 5105/5107/03 Secondary 3NA

www.KiasuExamPaper.com
80
5

9 Samples of four food colourings W, X, Y and Z were tested using paper


chromatography.

The chromatograms were compared with S 1 , and S 2 which are artificial


colourings.

W X Y Z S1 S2

Which of the following statements is false?

A W contains S 1 and S 2 .
B X is the artificial colouring, S 1 .
C Y is a pure substance.
D Z contains S 1 .

10 Which group of substances below consists of an element, a compound and a


mixture?

A air, water, sodium chloride


B water, salt solution, calcium oxide
C aluminium, copper, zinc oxide
D air, potassium bromide, oxygen

11 The descriptions of two substances are given below:

M is a solid which melts on heating to form a yellow liquid that cannot be


broken into a simpler substances.

N is a liquid which forms carbon dioxide and water when burnt in air.

Which of the following is correct regarding substances M and N?

M N
A Element Compound
B Compound Compound
C Compound Mixture
D Element Mixture

JWSS Mid-Year Examinations 2019 Science (Chemistry) 5105/5107/03 Secondary 3NA

www.KiasuExamPaper.com
81
6

12 Three statements regarding a compound are made.

I A compound has a fixed composition by mass and can be


represented by a chemical formula.
II A compound has properties different from its elements.
III A compound can be separated by physical means.

Which statements are correct?

A I and II only
B I and III only
C II and III only
D I, II and III

13 Which of the following is correct regarding the relative mass of the sub-atomic
particles of an atom?

Proton Neutron Electron


A +1 0 -1
B -1 1 +1
C 1
1 1
1840
D 1 1
1
1840

14 Which of the following shows the number of sub-atomic particles in a


positively-charged ion?

Protons Neutrons Electrons


A 2 2 2
B 3 3 2
C 2 3 3
D 3 3 3

15 Element Z has a proton number of 9 and a nucleon number of 20.


What ion will Z form?

A Z+
B Z-
C Z2+
D Z2-

16 What is the chemical formula of zinc nitrate?


JWSS Mid-Year Examinations 2019 Science (Chemistry) 5105/5107/03 Secondary 3NA

www.KiasuExamPaper.com
82
7

A ZnN
B Zn 3 N 2
C Zn(NO 3 ) 2
D Zn 3 (NO 3 ) 2

17 Element X has an electronic configuration of 2,6.


Element Y has an electronic configuration of 2,8,1.
What is likely to form if X and Y combine?

A A covalent compound Y 2 X.
B A covalent compound YX 2 .
C A ionic compound Y 2 X.
D A ionic compound YX 2 .

18 Which of the following will form a simple covalent compound with oxygen?

A Aluminium
B Lithium
C Neon
D Nitrogen

19 Which compound has 2 electrons being shared?

A LiF
B F2
C O2
D H2O

20 The table below gives data about four substances.


Which set of properties describes a simple covalent compound?

Able to conduct electricity in


melting point/ oC
solid state liquid state
A 53 no no
B 1500 no no
C 1265 no yes
D 805 yes yes

End of Paper

JWSS Mid-Year Examinations 2019 Science (Chemistry) 5105/5107/03 Secondary 3NA

www.KiasuExamPaper.com
83
8

JWSS Mid-Year Examinations 2019 Science (Chemistry) 5105/5107/03 Secondary 3NA

www.KiasuExamPaper.com
84
Class Index Number

Name:

Jurong West Secondary School


Mid-Year Examinations 2019 30

SCIENCE (CHEMISTRY) 5105/5107/04


Secondary Three Normal (Academic) 14 May 2019
Paper 4 0800 - 0915
Paper 3 and 4: 1 hour 15 minutes
Candidates answer on the Question Paper.

READ THESE INSTRUCTIONS FIRST

Write your name, class and index number on all the work you hand in.
Write in dark blue or black pen.
You may use an HB pencil for any diagrams, graphs, tables or rough working.
Do not use staples, paper clips, glue or correction fluid.

The use of an approved scientific calculator is expected, where appropriate.


You may lose marks if you do not show your working or if you do not use appropriate units.

Section A
Answer all questions in the spaces provided.

Section B
Answer all questions in the spaces provided.

The number of marks is given in brackets [ ] at the end of each question or part question.
A copy of the Periodic Table is printed on page 8.

After checking of answer script


Checked by Signature Date
Student

This document consists of _8_ printed pages.

Setter: Ms Kek Poh Khee www.KiasuExamPaper.com


85
2

Section A (14 marks)

Answer all the questions in this section in the spaces provided.

1 The diagrams (A to F) in the figure below shows the particles in different


substances.

A B C

D E F

Fill in the table below by stating the diagram(s) that matches with the
description.

Description Diagram (A to F)
A pure element

Mixture of two compounds

Mixture of an element and a


compound
[3]

2 The table below shows the melting and boiling points of nitrogen and
hydrogen.

Melting point/oC Boiling point/oC


Nitrogen -210 -196
Hydrogen -259 -253

(a) State the movement and arrangement of the particles in nitrogen at


-220oC.

Movement: ………...……………………………………………………

Arrangement: ...……………………………………………………………
[2]

JWSS Mid-Year Examinations 2019 Science (Chemistry) 5105/5107/04 Secondary 3NA

www.KiasuExamPaper.com
86
3

(b) When cooled from -100oC to -256oC, state whether nitrogen or hydrogen
will condense first. Explain your answer.

…………………………………………………………………………………..
[1]

(c) State the electronic configuration of nitrogen atom.

…………………………………………………………………………………..
[1]

(d) Nitrogen and hydrogen reacts to form ammonia, NH 3 , during the Haber
process.

Draw the dot-and-cross diagram of ammonia.

[2]

3 Compound Z contains elements X and Y.

The dot-and-cross diagram below shows the arrangement of all the electrons
in a molecule of compound Z.

Y X Y

(a) State the type of bonding present in Z.

…………………………………….……………………………………………
[1]
(b) Use the Periodic Table to identify elements X and Y.

X: …………..…………………………..

Y: …………..…………………………..
[1]

JWSS Mid-Year Examinations 2019 Science (Chemistry) 5105/5107/04 Secondary 3NA

www.KiasuExamPaper.com
87
4

(c) Element X has an isotope with 7 neutrons.

(i) Define the term isotopes.

……………….…………………………………………………………….

……………….…………………………………………………………….
[1]

(ii) Complete the table below for the isotope of X.

Number of Number of Number of


Mass number
neutrons protons electrons

[2]

JWSS Mid-Year Examinations 2019 Science (Chemistry) 5105/5107/04 Secondary 3NA

www.KiasuExamPaper.com
88
5

Section B (16 marks)

Answer all the questions in this section in the spaces provided.

4 Fig. 4.1. shows the heating curve of a mixture containing substance Y, in liquid
state, contaminated with some impurities.
Temperature / oC

200

179
172

30

Time / min
Fig. 4.1.

(a) Based on the heating curve, suggest why the mixture is impure.

……………………………………………………………………………………….
[1]

(b) Tick(3) the box which shows the boiling point of pure substance Y.

Tick (3)
171oC
174oC
180oC
199oC
[1]

(c) The mixture can be purified using the method shown in Fig. 4.2. below.

Condenser

Boiling chips

Fig. 4.2
JWSS Mid-Year Examinations 2019 Science (Chemistry) 5105/5107/04 Secondary 3NA

www.KiasuExamPaper.com
89
6

(i) Name the method shown in Fig. 4.2.

…....………………….…………………………………………………………
[1]

(ii) Draw arrows on Fig. 4.2. to indicate the direction of flow of water in the
condenser by drawing arrows. Label ‘water in’ and ‘water out’.
[1]

(iii) State the purpose of the boiling chips.

…....……….……………………………………………………………………
[1]

(d) The table below shows some properties of three solids.

Solids Colour Soluble in water


A Brown No
B Blue Yes
C White Yes

(i) Suggest a method to separate A and water.

…....……….……………………………………………………………………
[1]

(ii) A student performed crystallisation to separate a mixture containing


B and water.

He heated the mixture until it is saturated.


He then left the saturated solution to cool.

Describe how he can test whether the solution he has obtained is


saturated.

…....……….……………………………………………………………………

…....……….……………………………………………………………………
[1]

(iii) When separating solution of C, the student has forgotten to test for
saturation. He continued heating until all the solvent has evaporated.

Instead of a white solid, he has obtained black solid.

Explain why.

…....……….……………………………………………………………………
[1]

JWSS Mid-Year Examinations 2019 Science (Chemistry) 5105/5107/04 Secondary 3NA

www.KiasuExamPaper.com
90
7

5 Magnesium chloride is formed when magnesium reacts with hydrochloric acid.

(a) Complete the table below for the formula and number of electrons for
magnesium and chloride ions.

Name of ion Formula of ion Number of electrons


Magnesium ion

Chloride ion
[2]

(b) Draw the dot-and-cross diagram for magnesium chloride.


Show only valence electrons.

[3]

(c) (i) Magnesium chloride has high melting and boiling points.
Explain why.

………..........………………………………………………………………….

………..........………………………………………………………………….

………..........………………………………………………………………….
[2]

(ii) State another property of magnesium chloride.

………..........………………………………………………………………….
[1]

End of Paper

JWSS Mid-Year Examinations 2019 Science (Chemistry) 5105/5107/04 Secondary 3NA

www.KiasuExamPaper.com
91
8

JWSS Mid-Year Examinations 2019 Science (Chemistry) 5105/5107/04 Secondary 3NA

www.KiasuExamPaper.com
92
www.KiasuExamPaper.com
93
2019 MYE 3N(A) Paper 3 MS

1 2 3 4 5
B C C A C
6 7 8 9 10
D D B D D
11 12 13 14 15
A A D B B
16 17 18 19 20
C C D B A

Setter: Ms Kek Poh Khee www.KiasuExamPaper.com


94
5105/5107/04 MYE 2019 Answers

Qn Answer Marks / Remarks


A1
A pure element A, E [1]

Mixture of two F [1]


compounds
Mixture of an element C [1]
and a compound

[Total: 3]

A2 (a) vibrating about fixed positions [1]

closely packed in orderly manner/arrangement [1]


(b) Nitrogen. It has a higher boiling point. [[1]]
[1
(c) 2,5 [1
[1]
1]
(d)

H N H

H
x pair
airr off el
3 pa electrons
e ectron shared
ns sshhaarred
ed [1]
(For each
( or e
(F ach pair
ac r: 1 e- fr
pair: ffrom
om H, 1 e- fr
m H, from
fro
om N)

x 1 lone
ne
eppair
aiir off e
a electrons
lectro
le ons o
onn nitro
nitrogen [1]

Maximum 1 mark
if no dot-and-cross
to differentiate
electrons from
nitrogen/hydrogen
atoms
[Total: 6]

Setter: Ms Kek Poh Khee


www.KiasuExamPaper.com
95
2

A3 (a) Covalent bonding [1]


(b) X: Carbon [1] Both correct
Y: Oxygen
(c) (i) Isotopes are atoms of the same element with [1]
the same number of protons but different
number of neutrons.
(ii) Number Number Number [1] correct number
Mass
of of of of p & e
number
neutrons protons electrons
7 6 6 13 [1] mass number
Allow e.c.f. for mass
number = total
number of p + n
[Total: 5]

B4 (a) It does not have a fixed boiling point. / [1]


[1]
It boils over a range of temperature. / OW
OWTTE
It boils from 172oC to 179oC. /
Temperature is not constant during boiling.
ring boiling
g.

(b) Tiicckk (3)


T
Tick (3) [1]]
[1
1oC
171 3
174 4oC
180
18 80oC
199
19 99oC
(c) (i) Simple
S
Simp
ple
eddistillation.
issttiilllattio
ion.
n. / Distillation
Distillation
Di on
o n [1]
((ii)
(i
iii)) [1]
Correct labelling of
waateer ou
w
water oout
ut ‘water in’ and
‘water out’

water in

(iii) To ensure smooth boiling / prevent bumping. [1]

(d) (i) Filtration. [1]


(ii) Insert a clean, dry glass rod into the solution. [1]
Remove the glass rod.
If crystals form on the glass rod, solution is
saturated.
(iii) Solid C is unstable upon strong heating/ has [1]
decomposed. OWTTE

[Total: 8]

JWSS Mid-Year Examinations 2019 Science (Chemistry) 5105/5107/04 Secondary 3NA

www.KiasuExamPaper.com
96
3

B5 (a) Name of ion Formula of ion Number of


electrons
Magnesium ion Mg2+ 10 [2]

Chloride ion Cl- 18 Any 2 correct [1]

(b) x correct Mg2+ ion [1]


x correct Cl- ion [1]
x two chloride ions [1]

(c) (i) Large amount of energy is required [1]


to overcome the strong electrostatic forces of [1]]
attraction between oppositely-charged ions.
(ii) Soluble in water / [1]]
[1
Insoluble in organic solvent /
Cannot conduct electricity in solid,
lid, but ccan
an
conduct in aqueous and molten
olten state..
[Total: 8]
[T

JWSS Mid-Year Examinations 2019 Science (Chemistry) 5105/5107/04 Secondary 3NA

www.KiasuExamPaper.com
97
www.KiasuExamPaper.com
98
2019 Sec 3 NA Science Chemistry SA2
Bartley Sec 2

Section A (20 marks)

Answer all the questions in this section. Write your answers in the separate Multiple Choice Answer Sheet

1 The graph below shows a heating curve of a substance.

Which letter represents a mixture of solid and liquid states of the substance?

temperature/ oC

D
C

time/ min
0

2 What is the best explanation to why solids have fixed shapes?

A The particles are closely-packed together.


B The particles have a fixed shape and size.
C The particles have little spaces between them.
D The particles vibrate about a fixed position.

3 Which set of substances is correctly classified as elements, compounds and mixtures?

elements compounds mixtures


A argon air water
B boron carbon dioxide air
C carbon dioxide milk sea-water
D water hydrogen milk

4 How many elements are found in a molecule of ethanoic acid, CH 3 COOH?

A 3
B 4
C 6
D 8
BSS/ 2019/ End-of-Year Examination/ 3NA Sc(Chem)

www.KiasuExamPaper.com
99
3

5 An unknown element X has the electronic configuration 2, 8, 3.

What is the charge of the ion formed by an atom of element X?

A 3-
B 3+
C 5-
D 5+

6 Cl-35 and Cl-37 are isotopes of chlorine.

Which statement about the isotopes of chlorine is incorrect?

A They exist as gases at room conditions.


B They have the same neutron number.
C They have the same number of electrons in each atom.
D They have the same number of protons in each atom.

7 Which statement best describes a covalent bond?

A It is formed by donating and accepting a pair of electrons.


B It is formed by donating and accepting an electron.
C It is formed by sharing a pair of electrons.
D It is formed by sharing an electron.

8 Element X forms the ion X2+ while element Z forms the ion Z-.

What is the chemical formula of the compound formed when element X reacts with element Z?

A XZ
B XZ 2
C XZ 3
D X2Z

[Turn over
BSS/ 2019/ End-of-Year Examination/ 3NA Sc(Chem)

www.KiasuExamPaper.com
100
4

9 What is the definition of relative molecular mass, M r ?



A It is the average mass of an atom compared to the mass of a C-12 atom.
ଵଶ

B It is the average mass of a molecule compared to the mass of a C-12 atom.
ଵଶ

C It is the average weight of an atom compared to the mass of a C-12 atom.
ଵଶ

D It is the average weight of a molecule compared to the mass of a C-12 atom.
ଵଶ

10 What is equivalent to 2 moles?

A 2 g of hydrogen gas, H 2
B 14 g of nitrogen gas, N 2
C 20 g of argon, Ar
D 24 g of carbon, C

11 Which element has a completely-filled outer electron shell?

A Br B K C N D Ne

12 What is a trend of the elements from the left to the right of the Periodic Table?

A The elements become more metallic.


B The elements become darker in colour.
C The melting point of the elements increases.
D The proton number increases.

13 An element has the electronic configuration 2, 8, 8, 1.

What is the group and period number of this element?

group period
A I 1
B I 4
C VII 1
D VII 4

BSS/ 2019/ End-of-Year Examination/ 3NA Sc(Chem)

www.KiasuExamPaper.com
101
5

14 A piece of potassium is placed in a trough of cold water added with a few drops of Universal Indicator.

Which observation will not take place?

A explosion
B potassium darts around the surface of the water
C Universal Indicator turns from green to red
D vigorous effervescence

15 Which list of metals is placed in order of decreasing reactivity?

A calcium, iron, zinc, silver


B sodium, zinc, magnesium, copper
C magnesium, iron, lead, copper
D zinc, iron, copper, lead

16 What are the products of the reaction between magnesium and steam?

A magnesium hydroxide and hydrogen


B magnesium hydroxide and oxygen
C magnesium oxide and hydrogen
D magnesium oxide and oxygen

17 Why is copper used to make electrical wires?

A Copper has a low melting point.


B Copper has high density.
C Copper is a good conductor of electricity.
D Copper is shiny.

18 What is the observation when a piece of silver is added to dilute hydrochloric acid?

A effervescence
B explosion
C no visible change
D solution turns greenish-yellow

[Turn over
BSS/ 2019/ End-of-Year Examination/ 3NA Sc(Chem)

www.KiasuExamPaper.com
102
6

19 Metals can be beaten into thin sheets without breaking.

What is the term to describe this property of metal?

A ductile
B malleable
C soft
D sonorous

20 The observations made when three unknown metals, X, Y and Z are separately added to water, is
shown in the table below.

metal observation when added to water

X explosion

Y no visible change

Z effervescence

Which arrangement puts the metals in order of increasing reactivity?

A X, Y, Z
B X, Z, Y
C Y, X, Z
D Y, Z, X

BSS/ 2019/ End-of-Year Examination/ 3NA Sc(Chem)

www.KiasuExamPaper.com
103
7

Section B (14 marks)

Answer all the questions in this section in the spaces provided.

1 The table shows four elements and some information about them.

proton (atomic) electron electronic


element symbol
number number configuration
carbon 6 6 2,4
oxygen O 8
Mg 2,8,2
Cl 17 17

(a) Complete the table above to describe the different elements. [4]

(b) Draw the ‘dot and cross’ diagram to represent the arrangement of electrons in a
molecule of carbon dioxide, CO 2 .

[2]

[Turn over
BSS/ 2019/ End-of-Year Examination/ 3NA Sc(Chem)

www.KiasuExamPaper.com
104
8

2 Sodium, Na, is a highly reactive metal found in Group I of the Periodic Table. Fluorine, F 2 , is a pale
yellow gas found in Group VII of the Periodic Table.

(a) Explain why sodium is found in Group I of the Periodic Table.

_____________________________________________________________________ [1]

(b) Give a physical property of sodium.

_____________________________________________________________________ [1]

(c) Sodium and fluorine reacts in a highly explosive reaction to produce the salt sodium
fluoride, NaF, as the only product.

11.5 g of excess sodium reacts with fluorine to produce 0.2 moles of sodium fluoride.

(i) What is the number of moles of sodium that is added in this reaction?

number of moles of sodium = _____________ mol [2]

(ii) What is the mass of sodium fluoride produced in this reaction?

mass of sodium fluoride = _________________ g [2]

3 In ancient Egypt, copper was added to gold to form an alloy called red-gold. Besides giving the
gold a nice red hue, it made the gold stronger.

In a particular grade of red-gold discovered, it was found to contain 20 % copper and 80 %


gold.

Draw a diagram to represent the structure of this alloy.

[2]

BSS/ 2019/ End-of-Year Examination/ 3NA Sc(Chem)

www.KiasuExamPaper.com
105
9

Section C (20 marks)

Answer any two questions from this section.

4 (a) A water-soluble ink, X, is made up of several dyes. A student carried out an investigation
to determine the types of dyes found in ink X. The set-up for the experiment is shown
below.

cover

beaker

solvent

start line drawn using


position of spot of ink X at the ink X
start of the experiment

(i) Name the method of separation above.

________________________________________________________________ [1]

(ii) Two mistakes were made in her experimental set-up. Suggest the changes she will
need to make.

change 1: _______________________________________________________

_______________________________________________________

change 2: _______________________________________________________

_______________________________________________________
[2]

(iii) Some sand was found in the liquid ink X. Suggest the separation method that is
required to remove the sand from the mixture.

___________________________________________________________________ [1]

[Turn over
BSS/ 2019/ End-of-Year Examination/ 3NA Sc(Chem)

www.KiasuExamPaper.com
106
10

(b) Suggest an appropriate apparatus to measure the quantities below.

(i) 25.0 cm3 of liquid water _____________________________ [1]

(ii) time taken for a fast reaction to be completed _____________________________ [1]

(iii) 2.0 g of iron powder _____________________________ [1]

(iv) temperature of boiling water _____________________________ [1]

[Turn over
BSS/ 2019/ End-of-Year Examination/ 3NA Sc(Chem)

www.KiasuExamPaper.com
107
11

5 The table below lists a set of elements and their electronic configurations.

element electronic configuration

aluminium 2, 8, 3

calcium 2, 8, 8, 2

fluorine 2, 7

hydrogen 1

lithium 2, 1

Use information from the table to answer the questions below.

(a) Metals and non-metals can react with one another to form compounds with high melting
points.

(i) Name two elements that can combine to form a compound with high melting point.

_______________________________ and ________________________________ [1]

(ii) Give the chemical formula of the compound formed from the elements named in
(a)(i).

___________________________________________________________________ [1]

(iii) State the type of bonding found in the compound formed when metals and nom-
metals react.

___________________________________________________________________ [1]

(iv) Suggest the electrical conductivity of the compound formed in (a)(i).

___________________________________________________________________ [1]

(b) A non-metal can react with another non-metal to form a compound with low melting point.

(i) Name two elements from the table that will react to form a compound with low
melting point.

_______________________________ and ________________________________ [1]

(ii) Write the chemical equation for the formation of the compound from the elements
named in (b)(i).

___________________________________________________________________ [2]

BSS/ 2019/ End-of-Year Examination/ 3NA Sc(Chem)

www.KiasuExamPaper.com
108
12

(iii) Explain why the compound has a low melting point.

___________________________________________________________________

___________________________________________________________________ [1]

6 (a) Fluorine, chlorine, bromine and iodine are elements from Group VII of the Periodic Table.

(i) State the name given to all Group VII elements.

___________________________________________________________________ [1]

(ii) Describe the trend in the colour of Group VII elements down the group.

___________________________________________________________________ [1]

(iii) When fluorine is bubbled into a solution of potassium iodide, a brown solution is
formed. Explain this observation.

___________________________________________________________________

___________________________________________________________________

___________________________________________________________________ [2]

(b) You are provided with three unlabelled metals. It is known that the three metals are
calcium, iron and gold and they can be identified using an acid.

(i) Give an example of an acid that you can use to identify the metals.

___________________________________________________________________ [1]

(ii) Circle the observation for each of the metal with the substance having pH 1.

calcium: vigorous effervescence/ mild effervescence / no visible change

iron: vigorous effervescence/ mild effervescence / no visible change

gold: vigorous effervescence/ mild effervescence / no visible change


[3]

END OF PAPER

BSS/ 2019/ End-of-Year Examination/ 3NA Sc(Chem)

www.KiasuExamPaper.com
109
13

BLANK PAGE

[Turn over

BSS/ 2019/ End-of-Year Examination/ 3NA Sc(Chem)

www.KiasuExamPaper.com
110
14

BSS/ 2019/ End-of-Year Examination/ 3NA Sc(Chem)

www.KiasuExamPaper.com
111
www.KiasuExamPaper.com
112
3N Sc(Chem) End of Year Examinations 2019

Section A
1 B 2 D 3 B 4 A 5 B
6 B 7 C 8 B 9 B 10 D
11 D 12 D 13 B 14 C 15 C
16 C 17 C 18 C 19 B 20 D

Section B
Question 1
(a) Every 2 correct 1 mark

proton (atomic) electron electronic


element symbol
number mber
number configuration

carbon C 6 6 2,4

oxygen O 8 8 2,6

magnesium Mg 12 12
2 2,8,2

Chlorine Cl 17
17 17 2,8,7
2,

(b) Correctt bonded


bondded
d electrons
electro
rons
ro ns (2
(2 pairs);
ppaair
air
irs));
Correct
ct unbonded
Correc unbond
nded
nd d electrons;
nded ele
lecttrroons
ns;

Question
Ques on 2
sttiion
(a) It
It has
h s 1 valence
ha vvaalencce eel
electron;
lec
ectrroonn;
n;
(b)) Soft/low
Sof
offt/lo
tt//lo
low density/low
low ddeens
nsit
ityy//lloow me
melt
melting
ltin
ltingg and bboiling point/good conductor of heat or
in
electricity/malleable/ductile/shiny;
lectr
tric
tr icitty/
ic y/malllea
eabl
ble/
bl e/du
e/ dduuctil
(c) (i) No. of
o moles
oles = 11.5/23;
mol 11
= 0.5 mol;
(ii) Mass = 0.2 x (23 + 19);
= 8.4 g;

Question 3
Correct ratio;
Correct representation of alloy (Min 3 layers, closely-packed);

www.KiasuExamPaper.com
113
Section C
Question 4
(a) (i) Chromatography;
(ii) Place start line above solvent;
Draw start line with pencil;
(iii) Filtration;
(b) (i) pipette;
(ii) electronic stopwatch;
(iii) electronic balance; Reject ‘electric balance’
(iv) thermometer;

Question 5
(a) (i) Any metal (Al, Ca, Li) + Any non-metal
metal (F, H)
H);;
(ii) Correct formula;
(iii) ionic bonding;
(iv) Conducts only
ly in molten/aqueous
mol
olte
lte
ten/
n/aqu ous state;
aque staatte;
st
(b) (i) Fluorine hydrogen;
ne andd hydro
rro
oge
gen;
n;
(ii)
i) H 2 + F 2 ĺ
ĺ+)
+
+)
Correct
Co
C rrreecct formula;
fo
orrm
mul
ula;
a
a;
Balanced
Balanc d eequation;
ceed qquuation;;
(iii)
ii ) Small
(iii
(i Smaalll am
Sm aamount
oun of eenergy
unnt of neerg required
rgy requ
quir
qu ed tto
ired
ir overcome weak intermolecular forces;
o ove

Questionn 6
(a) (i) Halogen;
Halo
l geen;
(ii) Darkens;
arke
k ns;
(iii) Fluorine is more reactive than iodine;
And will displace iodine from a solution of its ions;
(b) (i) Sulfuric/hydrochloric/nitric acid;
(ii) calcium: vigorous effervescence;
iron: mild effervescence;
gold: no visible change;

www.KiasuExamPaper.com
114
www.KiasuExamPaper.com
115
BEATTY SECONDARY SCHOOL
END OF YEAR EXAMINATION 2019

SUBJECT : SCIENCE (CHEMISTRY) LEVEL : Sec 3 Normal Academic

PAPER : 5105/04 DURATION : 1 hour 15 minutes

SETTER : Ms Estella Chin DATE : 7 October 2019

CLASS : NAME : REG NO :

_________________________________________________________________________________

READ THESE INSTRUCTIONS FIRST

Write your name, class and register number on all the work you hand in.
Write in dark blue or black pen.
You may use an HB pencil for any diagrams or graphs.
Do not use staples, paper clips, glue or correction fluid.

Answer all questions in Section A and any two questions in Section B.


The use of an approved scientific calculator is expected, where appropriate.
In calculations, you should show all the steps in your working, giving your answer at each stage.
You are advised to spend no longer than 30 minutes on Paper 3.
You may proceed to answer Paper 4 as soon as you have completed Paper 3.
A copy of the Periodic Table is printed on page 12.

At the end of the examination, hand in your answers to Paper 3 and Paper 4 separately.
The number of marks is given in brackets [ ] at the end of each question or part question.

Section Marks
A / 14

B / 16

Total / 30

_________________________________________________________________________________
This paper consists of 12 printed pages (including this cover page).
[Turn over

www.KiasuExamPaper.com
116
2

Section A

Answer all the questions in the spaces provided.

1 The diagrams below represent the particles in three different substances A, B and C at room
temperature.

A B C

Which diagram(s) best represent(s) a

(a) pure substance,

……….………………………………………………………………………………………………..[1]

(b) mixture.

……….………………………………………………………………………………………………..[1]

2 Using water as the solvent, Amber carried out chromatography on three substances I, II and III.
The chromatogram obtained is shown below.

×I ×II ×III starting line

(a) Explain which substance, II or III, is more soluble in water.

………………….……………………………………………………………………………...…….......

………………….……………………………………………………………………………...……...[1]

(b) Describe two other conclusions that can be made from the chromatogram.

………………….……………………………………………………………………………...…….......

………………….……………………………………………………………………………...…….......

………………….……………………………………………………………………………...……...[2]

www.KiasuExamPaper.com
117
3

3 Five metallic elements, which belong to the same group in the Periodic Table, are given special
symbols. The special symbols and the order of reactivity of these metals are shown below.

decreasing order of chemical reactivity

Aa Bb Cc Dd Ee Ff

It is further given that Aa combines spontaneously with oxygen present in air to form a compound
with the formula Aa2O.

(a) State whether compound Aa2O is an ionic or covalent compound.

.…………………………………..……………..…………………………………..………………… [1]

(b) State the group that these elements belong to in the Periodic Table.

.……...…………………………..……………………………………..……………………………… [1]

(c) Write down the formula of the compound formed between the elements Ee and chlorine.

…..……………………………………………………………………………………..……………… [1]

(d) Suggest the name of the elements given the special symbols Aa and Ff.

special symbols name of element

Aa

Ff
[1]

www.KiasuExamPaper.com
118
4

4 The apparatus shown is used to collect the hydrogen gas produced from the reaction of lithium
metal and cold water.

reactants water

(a) Write down a balanced chemical equation, including state symbols, for the reaction between
lithium metal and cold water.

…..……………….………………………………………………………….………..……………… [2]

(b) Effervescence is observed in the reaction. Describe another observation that you would expect
to see in the reaction flask.

…..……………….………………….………………………………………………..……………… [1]

(c) (i) Identify the method of collection of gas as shown above.

…..………..….…………………………………………………………………..………….…… [1]

(ii) State the physical property of the gas that allows it to be collected by this method.

….………….………………………...…………………………………………..………….…… [1]

www.KiasuExamPaper.com
119
5

Section B

Answer any two questions from this section in the spaces provided.

5 Heptanol is an organic compound with the chemical formula C7H15OH. It has a pleasant smell and
is sometimes used in cosmetics due to its fragrance.

(a) The figure below shows the heating curve of heptanol from –50 °C to 200 °C.

(i) State the melting point and boiling point of heptanol.

Melting point …………..…….…°C

Boiling point ……………………°C [1]

(ii) Explain what is meant by the term melting point.

……………………………………….……………………………………….…………………....

…………………………………………………………………………….………………….....[2]

(iii) In the box below, draw a diagram to represent the arrangement of the heptanol particles
at –100 °C.

[1]

www.KiasuExamPaper.com
120
6

(iv) Describe the changes in the arrangement and movement of the heptanol particles as the
temperature decreases from 200 °C to 0 °C.

…………………………………….…………………………………….…………………...........

.………………………………………………………………………….…………………...........

….……………………………………………………………………….………………….......[2]

(b) When heptanol was heated, it reacted with oxygen in air to form carbon dioxide and water
vapour.

(i) Write a balanced chemical equation for the reaction above.

.………………………………………………………………………….………………….......[1]

(ii) Calculate the relative molecular mass, Mr, of heptanol, C7H15OH.

Mr = …………………… [1]

www.KiasuExamPaper.com
121
7

6 The figure shows the electronic structures of four elements drawn by a student.

element 1 element 2 element 3 element 4

(a) One of the electronic structures in the figure above was drawn wrongly. Explain which element
has a wrong electronic structure.

.……...……………….…………..……………………………………..…………….……………… [1]

(b) State the two elements which belong to the same period, and the period that they belong to.

Elements which belong to the same period: ……… and ………

Period …….. [1]

(c) Element 3 reacts with element 4 to form an ionic compound.

(i) Explain why atoms react to form compounds.

.……...……………….…………..……………………………………………………………… [1]

(ii) Draw a ‘dot and cross’ diagram of the compound formed between element 3 and element
4 showing only the outer shell electrons.

[2]

www.KiasuExamPaper.com
122
8

(d) (i) Determine the type of compound formed between element 1 and 3.

.……...……………….…………..……………………………………..…………….……….… [1]

(ii) Draw a ‘dot and cross’ diagram of the compound formed between element 1 and element
3 showing only the outer shell electrons.

[2]

www.KiasuExamPaper.com
123
9

7 Petrol is a mixture of hydrocarbons (compounds containing carbon and hydrogen atoms only)
containing between 5 and 10 carbon atoms and is used as a fuel for internal combustion engines.

Four of these hydrocarbons, A, B, C and D, are shown below.

A B

C D

(a) (i) Define relative molecular mass.

.……...……………….…………..……………………………………..…………….……….…

.……...……………….…………..……………………………………..…………….……… [2]

(ii) Which structure A, B, C or D, has the highest relative molecular mass?

.……...……………….…………..……………………………………..…………….……… [1]

(b) The table shows the boiling points of some of the hydrocarbons in the petrol fraction.

number of carbon atoms 5 6 7 8 9 10


boiling point / °C
36 69 126 151 174

www.KiasuExamPaper.com
124
10

(i) On the grid, plot a graph to show how the boiling point changes with the number of
carbon atoms in these hydrocarbons. Draw a best-fit curve through the points.
180
boiling point
/ °C

160

140

120

100

80

60

40

number of
carbon atoms

[2]

(ii) Use your graph to deduce the boiling point of the hydrocarbon with 7 carbon atoms.

boiling point …………………………. °C [1]

www.KiasuExamPaper.com
125
11

(c) (i) Suggest a method of separation that can be used to separate the hydrocarbons A, B, C
and D.

...…...……………….…………..…………………………………..………………….……… [1]

(ii) Explain why this method of separation was chosen.

.……...………………...………..…………………………………..………………….……… [1]

www.KiasuExamPaper.com
126
12

www.KiasuExamPaper.com
127
BEATTY SECONDARY SCHOOL
END OF YEAR EXAMINATION 2019

SUBJECT : SCIENCE (CHEMISTRY) LEVEL : Sec 3 Normal Academic

PAPER : 5105/03 DURATION : 1 hour 15 minutes

SETTER : Ms Estella Chin DATE : 7 October 2019

CLASS : NAME : REG NO :

_________________________________________________________________________________

READ THESE INSTRUCTIONS FIRST

Write in soft pencil.


Write your name, class and register number on the Optical Answer Sheet provided.
Do not use staples, paper clips, glue or correction fluid.

There are twenty questions on this paper. Answer all questions. For each question there are four
possible answers, A, B, C and D.
Choose the one you consider correct and record your choice in soft pencil on the separate Answer
Sheet.

Read the instructions on the Answer Sheet very carefully.

Answers to Paper 3 and Paper 4 must be handed in separately.


Each correct answer will score one mark. A mark will not be deducted for a wrong answer.
You are advised to spend no more than 30 minutes on Paper 3.
You may proceed to answer Paper 4 as soon as you have completed Paper 3.
Any rough working should be done in this booklet.
A copy of the Periodic Table is printed on page 8.
The use of an approved scientific calculator is expected, where appropriate.

_________________________________________________________________________________
This paper consists of 8 printed pages (including this cover page).
[Turn over

www.KiasuExamPaper.com
128
2

1 The four pieces of apparatus shown below can be used to measure volumes. Which
apparatus is the most suitable to measure 20.5 cm3 of liquid?

A B C D

2 A student intends to measure the volume of carbon dioxide gas produced from the reaction
between 10 cm3 of hydrochloric acid and excess calcium carbonate. The list of apparatus
available is shown below.

1 gas syringe 2 measuring cylinder


3 stopwatch 4 thermometer

Which piece(s) of apparatus is/are not required?

A 1 and 2
B 3 and 4
C 2 only
D 4 only

3 Calcium nitrate is soluble in water, while calcium carbonate is insoluble in water.


What is the correct order for the processes required to separate a mixture of solid calcium
nitrate from solid calcium carbonate?

first last
A dissolve evaporate crystallise filter
B dissolve filter evaporate crystallise
C filter dissolve evaporate crystallise
D filter crystallise evaporate dissolve

www.KiasuExamPaper.com
129
3

4 A student separated two miscible liquids using the apparatus shown.

thermometer

electric mixture
heater

Which statement correctly describes apparatus R?

A It is made up of cotton wool to absorb excess moisture.


B It is made up of glass beads to prevent the loss of volatile substances.
C It is made up of glass surfaces to provide increased surface area for condensation.
D It is made up of plastic balls to filter the liquid with smaller molecular size.

5 Which statement about the molecules in steam is correct?

A The molecules are diatomic.


B The molecules are weakly attracted to each other.
C The molecules do not possess kinetic energy.
D The molecules vibrate about fixed positions.

6 In which process(es) do particles lose speed?

1 boiling 2 condensation
3 freezing 4 melting

A 1 and 2
B 1 and 4
C 2 and 3
D 2 and 4

www.KiasuExamPaper.com
130
4

7 Which statement is not a difference between a compound and a mixture?

A A compound can be separated into simpler substances by physical means while a


mixture cannot.
B A compound has a fixed composition by mass while a mixture does not.
C A compound has different chemical properties from its constituent elements while a
mixture has similar chemical properties as those of its components.
D A compound is pure while a mixture is impure.

8 The boxes below contain information about substances F, G and H.

F is speckled blue and G is a reddish-brown solid H is colourless and


black. When water is that is a good conductor of has a constant
added, a black both heat and electricity. It composition. It
suspension in a blue cannot be decomposed produces two gases
solution is obtained. into anything simpler. when electrolysed.

Which of the following shows the correct classification of the substances?

F G H
A compound element mixture
B compound compound element
C mixture element compound
D mixture element mixture

9 The nucleon number of an isotope of copper is 60. How many protons, neutrons and
electrons are present in an atom of this isotope?

protons neutrons electrons


A 29 29 31
B 29 31 29
C 29 60 31
D 31 60 29

www.KiasuExamPaper.com
131
5

10 The nuclei of four different atoms W, X, Y and Z, are shown below.

W X Y Z
4 protons 5 protons 5 protons 6 protons
5 neutrons 5 neutrons 6 neutrons 7 neutrons

Which statement correctly describes the four atoms?

A Atom X represents boron-5 while atom Y represents boron-6.


B Atoms W and X are isotopes that have the same number of neutrons, but different
number of protons.
C Atoms X and Y have similar chemical properties.
D Atoms Y and Z have similar physical properties.

11 Why does molten potassium bromide conduct electricity?

A Potassium is a metal which is a good conductor of electricity.


B The orderly arrangement of the ions allows electricity to be easily transferred.
C The strong forces of attraction present allows electrical charges to pass through.
D There are mobile ions which behave as charge carriers.

12 Non-metals can form both covalent bonds and ionic bonds. Element X can be found in
Group VI in the Periodic Table. How many covalent bonds does it form in total and what is
the charge on an ion of X when it forms an ionic bond?

number of covalent bonds charge on ion of X


A 2 2–
B 2 6–
C 6 2+
D 6 6+

13 A metal X and a non-metal Y react together to form an ionic compound X 2Y. Which
statement is correct when this compound is formed?

A Each atom of X gives away two electrons.


B Each atom of X receives two electrons.
C Each atom of Y gives away two electrons.
D Each atom of Y receives two electrons.

www.KiasuExamPaper.com
132
6

14 Aspirin is an oral medication that is used to reduce fever and relieve muscle aches,
toothaches, common cold, and headaches. The structural formula of aspirin is shown
below.

What is the molecular formula of aspirin?

A C9O4H8N2
B CH4COC6H4COH
C CH3CO2C6H4CO2H
D CH3CO2C6H4N2CO2H

15 Which equation is not correctly balanced?

A C3H8 + 5O2 3CO2 + 4H2O


B 2NH3 + O2 N2 + 3H2O
C CH4 + 2O2 CO2 + 2H2O
D 2H2S + CO2 2S + 2H2O + C

16 Three elements X, Y and Z have consecutive, increasing proton numbers. If element Y is a


noble gas, what is the formula of the elements X, Y and Z respectively?

element X element Y element Z


A X Y2 Z
B X2 Y Z
C X2 Y Z+
D X Y2 Z+

17 Which statement is incorrect about the trends of Group VII elements on descending the
group?

A The colour intensity increases.


B The density increases.
C The melting point increases.
D The reactivity increases.

www.KiasuExamPaper.com
133
7

18 Which row shows the correct properties of the Group VII elements?

element state at room colour effect on potassium iodide


temperature solution
A astatine gas black turns brown
B bromine liquid reddish-brown remains colourless
C chlorine gas greenish-yellow turns brown
D iodine solid brown remains colourless

19 What is the molar mass, in g/mol, of magnesium sulfate, MgSO4?

A 60
B 80
C 120
D 216

20 The relative molecular mass (Mr) of a compound with the formula FexO4 is 232. What is
the value of x?

A 2
B 3
C 4
D 6

www.KiasuExamPaper.com
134
8

www.KiasuExamPaper.com
135
www.KiasuExamPaper.com
136
3NA EOY 2019 Answer Key
5105/03

1 2 3 4 5 6 7 8 9 10
A B B C B C A C B C
11 12 13 14 15 16 17 18 19 20
D A D C B B D C C B

5105/04

1 (a) A, C 1
(b) B 1
2 (a) Substance II is more soluble in water as the spot has travelled a further distance on the 1
chromatogram.
(b) Substance I is a mixture made up of three components.
nts. 2
The same substance is present in substances I and II.
Substance II and/or III is/ are pure.
(Any two)

3 (a) ionic compound 1


(b) Group I 1
(c) EeCl 1
(d) 1
special
cial symbols
sy
ymbolls name
na
name o
off element
elemen
nt

Aa
a Fran
an
anci
cium
um
Francium

F
Ff Lith
thiu
thium
Lithium

4 (a) 2Li(s)
(s) + 2H
2H2O (l) Æ 2 2LiOH
LiiOH
L OH (aq(aq)
q) + H2 (g
((g)) 2
1 mark:rk: Balanced
Bala
Ba anceded cchemical
hemica
he ca al e
eqqua
u ti
t on
equation
1 mark: k: St
tat
ate symbol
State sy
s y ol
symbolsols ((pprro
ovided correct balanced chemical equation)
(provided
(b) Li dissolves
ves in wa
water/
ateer/
r/ 1
Li floats onn water
wa er / darts
atte dart on water surface.
(c) (i) Displacement
acem of water 1
(ii) Insoluble in water 1

5 (a) (i) Melting point: –35 °C [1] 2


Boiling point: 176 °C [1]

(ii) The temperature [1] at which a substance changes from the solid state into liquid 1
state [1].

www.KiasuExamPaper.com
137
2

(iii) 1

(iv) At 200 °C, the particles were far apart [1 mk pt] and moved rapidly in all 2
directions [1 mk pt].
At 0 °C, the particles are closely packed [1 mk pt] and slide over one another
[1 mk pt]

Every 2 mk pt: 1 mark

(b) (i) 2C7H15OH + 21O2 Æ 14CO2 + 16H2O 1


(ii) Mr = 12 x 7 + 15 + 16 + 1 1
= 116

6 (a) Element 2. There is only 1 electron in thee first


fir
irst electron
elec
ecctr
tron
on shell.
on sh
hel
ell.. / The
e first
firs
st electron shell
sh 1
should have 2 electrons.
(b) Elements 2 and 3. Periodd 2. 1
(c) (i) To achieve a stable,, noble
no
nob
blle ga
g
gass co
configur
configuration/
rattion/ structure.
structur
uree. 1
(ii) 2
+ –

1 ma
mark:
ark
rkk:: co
ccorrect
orrecct charges
char
ch arge
arg s and number of ions
1 mark: correct
corr
co rrec
rr ectt electronic
ec electro structure
(d) (i) Covalent
vale
ent compound.
compo 1
(ii) 2

1 mark: correct non-bonding electrons


1 mark: correct bonding electrons

www.KiasuExamPaper.com
138
3

7 (a) (i) Relative molecular mass, Mr, is the average mass of an molecule [1] of the 2
1
substance when compared with the mass of of an atom of the carbon-12
12
isotope [1], 126 C.
(ii) B 1
(b) (i) Correctly plotted points [1] 2
Smooth curve [1]
(ii) 98 °C (Accept value from student’s graph) 1
(c) (i) Fractional distillation. 1
(ii) A, B, C and D are miscible liquids / have different boiling points. 1

www.KiasuExamPaper.com
139
www.KiasuExamPaper.com
140
CANBERRA SECONDARY SCHOOL

2019 Semestral Assessment 2


Secondary Three Normal Academic

SCIENCE (CHEMISTRY) 2 October 2019


5105/03 1 hour 15 minutes
5105/04 1105h –1220h

Name: ______________________________________ ( ) Class: ________

READ THESE INSTRUCTIONS FIRST

Do not turn over the paper until you are told to do so.
Write in dark blue or black ink pen on both sides of the paper.
Do not use staples, paper clips, glue or correction fluid.

Answer ALL questions in Paper 3. Indicate your answers on the OTAS provided.
You are advised to spend no more than 30 minutes on Paper 3.

Answer ALL questions in Paper 4 Section A and any TWO questions in Section B.
Write all your answers on the spaces provided.

At the end of the examination, hand in your OTAS and question paper.

The number of marks is given in brackets [ ] at the end of each question or part question.

You may use a calculator for this examination.

A copy of the Periodic Table is printed on Page 17.

FOR MARKER’S USE

Marks Max
Section
Awarded Marks

Paper 3 20
Section A 14
Paper 4
Section B 16
Total 50

This question paper consists of 17 printed pages including the cover page.

Setter: Ms Rebecca Ng

www.KiasuExamPaper.com
141
2

Paper 3 (20 marks)

Answer all questions in this section.

1 The following steps were taken by a student to investigate a reaction between zinc and
dilute hydrochloric acid.

1. Measure and pour 20.0 cm3 of dilute hydrochloric acid into a beaker.
2. Measure the initial temperature of the acid.
3. Weigh 2 g of zinc granules and add it into the acid.
4. Measure the time taken for the reaction to complete.

Which set of apparatus does she need?

electronic stopwatch measuring laboratory


balance cylinder thermometer
A 9 9 9 8
B 9 9 9 9
C 9 8 9 9
D 8 9 8 9

2 Which of the following statements is true about particles in the solid state?

A The particles are arranged closely packed in an orderly manner


B The particles move by sliding over each other.
C When the particles are cooled down, they stop moving.
D When the particles are heated, the particles expand and become larger.

3 Chromatography was used to test for food colourings which have been banned.
The results are shown in the diagram.

Which of the three drinks tested contains both of the banned colourings?

A Fizzo only
B Juicy only
C Fizzo and Sparkle only
D Juicy and Sparkle only
____________________________________________________________________________________________________
Canberra Secondary School Science (Chemistry) 5105/03, 5105/04
2019 Semestral Assessment 2 Secondary 3 Normal (Academic)

www.KiasuExamPaper.com
142
3

4 Which diagram represents a mixture of an element and a compound?

A B C D

5 Which of the following options is correct?

element compound mixture


A air sodium oxide chlorine
B water ammonia air
C chlorine steel sodium oxide
D potassium water steel

6 Which of the following is likely to be a pure compound?

A a white powder which dissolves in water


B a liquid which gives two components when distilled
C blue crystals which melt over the range of 55 oC to 60 oC
D green crystals which melts at 58 oC

7 Three statements about isotopes are given.

1 Isotopes of the same element have the same number of protons.


2 Isotopes of the same element have different number of neutrons.
3 Isotopes of the same element have different atomic masses.

Which of the following is true?

A All statements are correct and statement 1 explains statement 2.


B All statements are correct and statement 2 explains statement 3.
C Statement 1 and 2 is correct but statement 3 is not correct.
D Statement 3 is correct but statement 1 and 2 is not correct.

8 Which of the following options contains only electronic configuration of metal atoms?

A 1 2,2 2,8,3
B 2 2,4 2,8,5
C 2,8 2,8,1 2,8,2
D 2,8,2 2,8,8,1 2,8,8,2

____________________________________________________________________________________________________
Canberra Secondary School Science (Chemistry) 5105/03, 5105/04
2019 Semestral Assessment 2 Secondary 3 Normal (Academic)

www.KiasuExamPaper.com
143
4

9 Which of the following statements about bonding is true?

A Covalent bonding is the intermolecular forces of attraction between molecules.


B Covalent bonding involves sharing of electrons between metals.
C Ionic bonding is the transfer of electrons between non-metals.
D Ionic bonding involves the transfer of electrons from metals to non-metals.

10 Elements W and X have the proton number of 8 and 11 respectively.

Which of the following shows the correct chemical formula and type of compound
formed from the elements?

chemical formula
type of compound
of compound
A XW 2 ionic
B X2W ionic
C WX covalent
D WX 2 covalent

11 Which of the following shows the properties of carbon monoxide?

electrical conductivity
melting point / °C boiling point / °C
in solid state in molten state
A -183 -162 poor poor
B 15 104 poor poor
C 657 997 poor good
D 2040 3654 good good

12 The table shows the results of adding dilute hydrochloric acid and aqueous sodium
hydroxide to an oxide.

Which of the following results indicates that the oxide is acidic?

result of adding
dilute hydrochloric acid dilute sodium hydroxide
A no reaction no reaction
B no reaction reaction
C reaction no reaction
D reaction reaction

13 Which of the following options is correct?

acidic oxide basic oxide amphoteric oxide neutral oxide


A carbon monoxide sodium oxide zinc oxide water
B carbon dioxide lead(II) oxide aluminium oxide nitrogen dioxide
C nitrogen dioxide copper(II) oxide potassium oxide nitrogen monoxide
D sulfur dioxide magnesium oxide aluminium oxide water

____________________________________________________________________________________________________
Canberra Secondary School Science (Chemistry) 5105/03, 5105/04
2019 Semestral Assessment 2 Secondary 3 Normal (Academic)

www.KiasuExamPaper.com
144
5

14 Salts can be prepared using the following methods:

1 metal + acid
2 metal oxide + acid
3 metal carbonate + acid
4 metal hydroxide + acid

Which method(s) will lead to the production of a gas as the other product?

A 1 only
B 1 and 3 only
C 2 and 4 only
D 2, 3 and 4 only

15 Aqueous solution P is added to aqueous ammonium nitrate.

The mixture is heated and ammonia gas is given off.

What is P?

A ammonium sulfate
B hydrochloric acid
C sodium hydroxide
D sodium sulfate

16 Which of the following correctly matches the gas and its identification test?

gas test and result


A ammonia moist red litmus paper remains red
B carbon dioxide lighted splint extinguishes
C hydrogen effervescence observed in limewater
D oxygen glowing splint relights

17 How are the elements arranged in the Periodic Table?

A in order of chemical reactivity


B in order of electron shells
C in order of proton number
D in order of relative atomic mass

____________________________________________________________________________________________________
Canberra Secondary School Science (Chemistry) 5105/03, 5105/04
2019 Semestral Assessment 2 Secondary 3 Normal (Academic)

www.KiasuExamPaper.com
145
6

18 Code letters, T, U, W, X, Y and Z of 6 different elements are placed in the Periodic


Table as shown below.

I II III IV V VI VII 0
T U W
X Y

Which one of the following is a correct statement regarding T, U, W, X, Y and Z?

A T and X are solids with high densities at room temperature.


B X and Y have the same number of electron shells.
C Y is the most reactive element of the 6 elements.
D Z is more reactive than U.

19 Which statement about Group I metals is false?

A Their densities decrease down the group.


B Their melting points decrease down the group.
C Their reactivity increases down the group.
D They react with water to form alkalis.

20 The table lists the halogens and their appearance.

halogen appearance
fluorine pale yellow gas
chlorine yellowish-green gas
bromine red-brown liquid
iodine dark purple solid
astatine ?

Astatine would be expected to be a

A brown gas.
B black solid.
C grey liquid.
D white solid.

End of Paper 3

____________________________________________________________________________________________________
Canberra Secondary School Science (Chemistry) 5105/03, 5105/04
2019 Semestral Assessment 2 Secondary 3 Normal (Academic)

www.KiasuExamPaper.com
146
7

Paper 4
Section A (14 marks)

Answer all the questions in this section.


Write your answers in the spaces provided.

A1 The list shows the formulae of some substances.

AgNO 3 BaSO 4 CaCO 3 CO NaCl SO 2

For each statement, (a) to (c), select from the list the substance described.

Write each formula in the space provided.

A formula may be used once, more than once, or not at all.

(a) This substance can be prepared using the titration method.

…………………………………………………………………………………………… [1]

(b) This substance can be used to reduce excess acidity in soil.

…………………………………………………………………………………………… [1]

(c) This substance when dissolved in water, turns blue litmus paper red.

…………………………………………………………………………………………… [1]

A2 The table below gives information about an atom of hydrogen and an atom of oxygen.

atom number of protons electronic configuration

hydrogen 1

oxygen 2,6

(a) With reference to the Periodic Table, complete the table above. [1]

(b) Hydrogen and oxygen can react to form a compound which boils at 100 oC.

Draw the ‘dot and cross’ diagram to show the bonding in the compound formed.
Show all electrons.

[2]

____________________________________________________________________________________________________
Canberra Secondary School Science (Chemistry) 5105/03, 5105/04
2019 Semestral Assessment 2 Secondary 3 Normal (Academic)

www.KiasuExamPaper.com
147
8

(c) With reference to its structure and bonding, explain why the compound in (b)
has a low boiling point.

……………………………………………………………………………………………

……………………………………………………………………………………………

…………………………………………………………………………………………… [2]

A3 Magnesium chloride, MgCl 2 , is a soluble salt. Crystals of magnesium chloride can be


made by reacting dilute hydrochloric acid, HCl, with an excess of powdered
magnesium carbonate, MgCO 3 .

The diagram below shows the first three steps used to prepare crystals of this salt.

(a) (i) Explain why an excess of powdered magnesium carbonate is used.

……………………………………………..……………………………………..

……………………………………………..…………………………………….. [1]

(ii) Name the processes being used in step 2 and step 3 respectively.

process used in step 2 is ……………………………………………………….

process used in step 3 is ………………………………………………………. [1]

(b) The solution left in step 3 is allowed to cool for some time to allow crystals to
grow. Describe the remaining steps required to obtain a sample of pure and dry
magnesium chloride crystals.

……………………………………………………………………………………………

……………………………………………………………………………………………

…………………………………………………………………………………………… [2]

____________________________________________________________________________________________________
Canberra Secondary School Science (Chemistry) 5105/03, 5105/04
2019 Semestral Assessment 2 Secondary 3 Normal (Academic)

www.KiasuExamPaper.com
148
9

(c) A student plans to prepare the salt, silver chloride, using the same method
above.

Do you expect the student to be successful in preparing the salt? Explain your
answer.

……………………………………………………………………………………………

……………………………………………………………………………………………

…………………………………………………………………………………………… [2]

End of Section A

____________________________________________________________________________________________________
Canberra Secondary School Science (Chemistry) 5105/03, 5105/04
2019 Semestral Assessment 2 Secondary 3 Normal (Academic)

www.KiasuExamPaper.com
149
10

Paper 4
Section B (16 marks)

Answer any TWO questions in this section.


Write your answers in the spaces provided.

B1 A chemist obtained a mixture of three alcohols, ethanol, propanol and butanol.

The table below shows the boiling points of the three alcohols respectively.

alcohol boiling point / oC


ethanol 78
propanol 97
butanol 118

In order to obtain pure samples of each alcohol, the chemist set up the experiment as
shown below.

(a) Complete the table below.

name of process occurring


name of apparatus
in the apparatus

Y
[2]

(b) Suggest an apparatus that could be added to the set up to ensure a more
efficient separation of the three alcohols.

…………………………………………………………………………………………… [1]

____________________________________________________________________________________________________
Canberra Secondary School Science (Chemistry) 5105/03, 5105/04
2019 Semestral Assessment 2 Secondary 3 Normal (Academic)

www.KiasuExamPaper.com
150
11

(c) Of the three alcohols present in the mixture, which alcohol will be collected last?
Explain your answer.

……………………………………………………………………………………………

…………………………………………………………………………………………… [2]

(d) Explain the purpose of the boiling stones.

……………………………………………………………………………………………

…………………………………………………………………………………………… [1]

(e) (i) In the box below, draw a diagram to show the arrangement of particles in
P.

[1]

(ii) Describe the movement of particles in P.

……………………………………………..……………………………………..

……………………………………………..…………………………………….. [1]

____________________________________________________________________________________________________
Canberra Secondary School Science (Chemistry) 5105/03, 5105/04
2019 Semestral Assessment 2 Secondary 3 Normal (Academic)

www.KiasuExamPaper.com
151
12

B2 (a) John was washing his hair when some shampoo accidentally entered his mouth.
He rinsed his mouth immediately as it left a bitter taste in his mouth. After his
shower, John checked the label of the shampoo bottle. Part of the shampoo’s
ingredients list is shown below.

(i) The chemical that caused the bitter taste in John’s mouth is an alkali that
is also responsible for the soapy feel of the shampoo.

From the ingredient list above, state the ingredient that has caused the
bitter taste in John’s mouth.

……………………………………………..…………………………………….. [1]

(ii) John brought a small sample of the shampoo to the science laboratory
and diluted it with water. With the permission of his science teacher, he
added one drop of Universal Indicator into the diluted shampoo.

State the colour change that the student will expect to see in the drop of
Universal Indicator.

……………………………………………..…………………………………….. [1]

(iii) Under supervision, John did further tests and found out that a typical
1 litre bottle of shampoo contains 30 g of the salt, sodium chloride, NaCl.

Calculate the number of moles of sodium chloride present in a typical


1 litre bottle of shampoo. Round off your answer to 3 decimal places.

number of moles of sodium chloride = …………………………. mol [2]

____________________________________________________________________________________________________
Canberra Secondary School Science (Chemistry) 5105/03, 5105/04
2019 Semestral Assessment 2 Secondary 3 Normal (Academic)

www.KiasuExamPaper.com
152
13

(b) 20.0 cm3 of potassium hydroxide solution was placed in a beaker.

The temperature of this solution is measured and recorded.

To the beaker of potassium hydroxide solution, a 2.0 cm3 portion of dilute


hydrochloric acid is added and the mixture is stirred. The highest temperature
reached is measured and recorded.

Seven further 2.0 cm3 portions of hydrochloric acid are added. Each time, the
mixture is stirred and the highest temperature reached is measured and
recorded.

The results for the experiment are shown in the table below.

volume of
acid added / 0.0 2.0 4.0 6.0 8.0 10.0 12.0 14.0
cm3
temperature
of the mixture 21.0 24.2 27.4 30.7 32.9 31.2 29.4 27.6
/ oC

Four points from the results above have been plotted on the grid below and a
best-fit line for the four points has been drawn.

(i) On the grid, plot the remaining four points using the results above. Mark
each point with a cross (x). [1]

(ii) Draw a best-fit line for these four points in (b)(i). Extend this line to
intersect (cross) the line already drawn. [1]

____________________________________________________________________________________________________
Canberra Secondary School Science (Chemistry) 5105/03, 5105/04
2019 Semestral Assessment 2 Secondary 3 Normal (Academic)

www.KiasuExamPaper.com
153
14

(iii) The acid and the alkali exactly neutralise each other at the point where
the two graph lines intersect (cross).

From your graph, determine the volume of hydrochloric acid required to


exactly neutralise the potassium hydroxide solution.

volume = ……………………………… cm3 [1]

(iv) Write an ionic equation to show what is meant by the term neutralisation.
Include state symbols.

……………………………………………..…………………………………….. [1]

____________________________________________________________________________________________________
Canberra Secondary School Science (Chemistry) 5105/03, 5105/04
2019 Semestral Assessment 2 Secondary 3 Normal (Academic)

www.KiasuExamPaper.com
154
15

B3 (a) The table shows the relative atomic masses, A r , and the melting points of some
Group I metals.

metal relative atomic mass, A r melting point / oC


lithium 7 181
sodium 23 98
potassium 39
rubidium 85 39
caesium 133 28

(i) Plot a graph of melting point against relative atomic mass, marking each
point with a (x). [1]

(ii) Draw a curved line of best fit, taking into account all your plotted points. [1]

____________________________________________________________________________________________________
Canberra Secondary School Science (Chemistry) 5105/03, 5105/04
2019 Semestral Assessment 2 Secondary 3 Normal (Academic)

www.KiasuExamPaper.com
155
16

(iii) Use your graph to predict the melting point of potassium.

………………….………….. oC [1]

(iv) Describe two observations made when sodium is added to water


containing Universal Indicator.

……………………………………………..……………………………………..

……………………………………………..……………………………………..

……………………………………………..…………………………………….. [2]

(b) Halogens exist as diatomic molecules and are all found in Group VII of the
Periodic Table.

A teacher bubbles chlorine gas through a colourless potassium iodide solution.

(i) Write down a balanced chemical equation for the reaction that occurs
between chlorine and the potassium iodide solution.

……………………………………………..…………………………………….. [1]

(ii) Describe an observation that you would make in this chemical reaction.
Give an explanation for the observation.

……………………………………………..……………………………………..

……………………………………………..…………………………………….. [2]

End of Paper 4

____________________________________________________________________________________________________
Canberra Secondary School Science (Chemistry) 5105/03, 5105/04
2019 Semestral Assessment 2 Secondary 3 Normal (Academic)

www.KiasuExamPaper.com
156
17

The Periodic Table of the Elements


Group
I II III IV V VI VII 0
1 2
H He
hydrogen helium
key 1 4
3 4 proton (atomic) number 5 6 7 8 9 10
Li Be atomic symbol B C N O F Ne
lithium beryllium name boron carbon nitrogen oxygen fluorine neon
7 9 relative atomic mass 11 12 14 16 19 20
11 12 13 14 15 16 17 18
Na Mg Al Si P S Cl Ar
sodium magnesium aluminium silicon phosphorus sulfur chlorine argon
23 24 27 28 31 32 35.5 40
19 20 21 22 23 24 25 26 27 28 29 30 31 32 33 34 35 36
K Ca Sc Ti V Cr Mn Fe Co Ni Cu Zn Ga Ge As Se Br Kr
potassium calcium scandium titanium vanadium chromium manganese iron cobalt nickel copper zinc gallium germanium arsenic selenium bromine krypton
39 40 45 48 51 52 55 56 59 59 64 65 70 73 75 79 80 84
37 38 39 40 41 42 43 44 45 46 47 48 49 50 51 52 53 54
Rb Sr Y Zr Nb Mo Tc Ru Rh Pd Ag Cd In Sn Sb Te I Xe
rubidium strontium yitrium zirconium niobium molybdenum technetium ruthenium rhodium palladium silver cadmium indium tin antimony tellurium iodine xenon

157
85 88 89 91 93 96 - 101 103 106 108 112 115 119 122 128 127 131
55 56 57 – 71 72 73 74 75 76 77 78 79 80 81 82 83 84 85 86
Cs Ba lanthanoids
Hf Ta W Re Os Ir Pt Au Hg Tl Pb Bi Po At Rn
caesium barium hafnium tantalum tungsten rhenium osmium iridium platinum gold mercury thallium lead bismuth polonium astatine radon
133 137 178 181 184 186 190 192 195 197 201 204 207 209 - - -
87 88 89 - 103 104 105 106 107 108 109 110 111 112 114 116

www.KiasuExamPaper.com
Fr Ra Rf Db Sg Bh Hs Mt Ds Rg Cn Fl Lv
francium radium actinoids Rutherfordium dubnium seaborgium bohrium hassium meitnerium darmstadtium roentgenium copernicium flerovium livermorium
- - - - - - - - - - - - -

57 58 59 60 61 62 63 64 65 66 67 68 69 70 71
lanthanoids La Ce Pr Nd Pm Sm Eu Gd Tb Dy Ho Er Tm Yb Lu
lanthanum cerium praseodymium neodymium promethium samarium europium gadolinium terbium dysprosium holmium erbium thulium ytterbium lutetium
139 140 141 144 - 150 152 157 159 163 165 167 169 173 175
89 90 91 92 93 94 95 96 97 98 99 100 101 102 103
actinoids Ac Th Pa U Np Pu Am Cm Bk Cf Es Fm Md No Lr
actinium thorium protactinium uranium neptunium plutonium americium curium berkelium californium einsteinium fermium mendelevium nobelium lawrencium
- 232 231 238 - - - - - - - - - - -
3
The volume of one mole of any gas is 24 dm at room temperature and pressure (r.t.p.)
www.KiasuExamPaper.com
158
CANBERRA SECONDARY SCHOOL

2019 Semestral Assessment 2


Secondary Three Normal Academic

SCIENCE (CHEMISTRY) 2 October 2019


5105/03 1 hour 15 minutes
5105/04 1105h –1220h

Name: Mark Sheme__________________________ ( ) Class: ________

READ THESE INSTRUCTIONS FIRST

Do not turn over the paper until you are told to do so.
o.
es of the paper.
Write in dark blue or black ink pen on both sides pap
perr.
Do not use staples, paper clips, glue or correction
io fluid.
orrectio
ion
io d..

Answer ALL questions in Paper


er 3. Indica
Indicate
cate
ca te yourr answers
n errs on
answer
an on the
e OTAS
the OTA provided.
T S pr
prov
ovided.
ov
mo
ore
re than
You are advised to spend no more tha
han 30 minutes
minuteses on
es on Paper 3.
3.

estions in
Answer ALL questions in Paper
Pa
P ape
p r 4 Se
S ctio
on A and
Section da
anny TWO
any TW
T WO questions
ques
estion
ns in Section
Sectio B.
Se
Write all your answers
ans wers on
nsswe n the
tth spaces
he sp
pac e provided.
aces pro
ro v
viided.
d.
d.

At the
he end
en
e nd off the
the examination,
exa
ami
m na
ati o , hand
tion nd in
nd in your
yo
y urr OTAS
ou TAS and
OTA nd question
an estion paper.
ques
qu es

The number
mbe
be off mar
er o marks
arks
ar given
ks is gi
g iv
veen in
nbbrackets
rackets [ ] at the
ack the end of each question or part question.

You may use


se a calculator
callcu
c la tor for
lato or this
ffo hiss examination.
th exa
xami
m natio

A copy of the Periodic


c Table
Tabl
Ta ble
bl e is printed
pri on Page 17.

FOR MARKER’S USE

Marks Max
Section
Awarded Marks

Paper 3 20
Section A 14
Paper 4
Section B 16
Total 50

This question paper consists of 17 printed pages including the cover page.

Setter: Ms Rebecca Ng

www.KiasuExamPaper.com
159
2

Paper 3 (20 marks)

Answer all questions in this section.

1 The following steps were taken by a student to investigate a reaction between zinc and
dilute hydrochloric acid.

1. Measure and pour 20.0 cm3 of dilute hydrochloric acid into a beaker.
2. Measure the initial temperature of the acid.
3. Weigh 2 g of zinc granules and add it into the acid.
4. Measure the time taken for the reaction to complete.

Which set of apparatus does she need?

electronic stopwatch measuring ry


laboratory
balance cylinder ometer
thermometer
A 9 9 9 8
B 9 9 9 9
C 9 8 9 9
D 8 9 8 9

2 Which of the following statements


ments iss true ab
a
about
bou
out p
pa
particles
art
rtic
icle
les in
in the
the solid
sol
olid
id state?
sta
tate?

A The particles are arranged


arrrra
ang
nged
ed closely
clo
lose
lo sely packed
se pac
acke
ac k d in an orderly
orrde
derl
rly manner
rl mann
ma nner
nn
B The particles
icles move e byby sliding
slid
slid ng over
diin ov
o er each
ver ea
acch other.
C When
en the pparticles
arti
ar
arti c es are
ticl re ccooled
are oole
oo down,
led do wn tthey
own
wn, heyy stop
s op moving.
st mov
ovin
ov ing.
in
D Whenn tthe particles
he p arrttiicl
arti es are
cles re heated,
re h ated
he ed, the particles
atted pa
art
rtiic es expand and
clle and become
becom larger.

3 C
Ch
hroma
ro
oma
mato
t graphy
Chromatography hy was
was
as used to
to ttest
esst fo
or ffo
for ood
d ccolourings
food olou
ol ouri
ou ring
rings whi
ng which have been banned.
The
e results
re
essu
ult
lts are
arre shown
a sh
how
o n in
n the
the
he diagram.
diia
agr
g am.

Which of the three drinks tested contains both of the banned colourings?

A Fizzo only
B Juicy only
C Fizzo and Sparkle only
D Juicy and Sparkle only
____________________________________________________________________________________________________
Canberra Secondary School Science (Chemistry) 5105/03, 5105/04
2019 Semestral Assessment Secondary 3 Normal (Academic)

www.KiasuExamPaper.com
160
3

4 Which diagram represents a mixture of an element and a compound?

A B C D

5 Which of the following options is correct?

element compound mixture


A air sodium oxide chlorine
B water ammonia air
C chlorine steel sodium oxide
de
D potassium water steel
eel

6 Which of the following is likely to be a pure


re compound?
compound
d?

A a white powder which dissolves


ssolves in n water
wate
wa er
B a liquid which gives two components
com
om
mpop nentntts when
when
wh en distilled
dis
isti
tillled
till ed
C blue crystals which
hich melt
melltt over
me ver the
ove th range
rangge of 55
ng 55 C to
o o660
0 oC
D green crystals
tals which
ch melts
ch mellttss att 58
58 C
o

7 e statem
Three em
ments a
statements bo
b out
aboutut iisotopes
soto
so oppe
es are
are give
ar ven.
ve n
given.

1 Issotopes
Isotopes ess of
of the
th
he same e element
elleemme
ent
nt have
haavve the
t e same
th sa
ame number
num
mbe of protons.
2 Isotopes
Is otopes of
sot of the
he same
th e element
me ele
lemen have
ntt h avee different
diff
di ffer
ff ent number
eren
er en num of neutrons.
3 Isotopes
IIs
sot
ot o
op es of
pes of the
t e same
th sa
s a
amme
meeelement
lle
ement havhaveve different
diff
di fferent atomic masses.

Which
ich of
of the
he following
th ffo
olllllo
owiin
ow ng is
is true?
true?
e?
?

A All statements
sttat
s ateem
ments s are
are correct
correct and statement 1 explains statement 2.
B All statements
statem
st men
ents
ts are
are correct
co and statement 2 explains statement 3.
C Statement
atem menentt 1 and 2 is correct but statement 3 is not correct.
D Statement
tement 3 is correct but statement 1 and 2 is not correct.

8 Which of the following options contains only electronic configuration of metal atoms?

A 1 2,2 2,8,3
B 2 2,4 2,8,5
C 2,8 2,8,1 2,8,2
D 2,8,2 2,8,8,1 2,8,8,2

____________________________________________________________________________________________________
Canberra Secondary School Science (Chemistry) 5105/03, 5105/04
2019 Semestral Assessment Secondary 3 Normal (Academic)

www.KiasuExamPaper.com
161
4

9 Which of the following statements about bonding is true?

A Covalent bonding is the intermolecular forces of attraction between molecules.


B Covalent bonding involves sharing of electrons between metals.
C Ionic bonding is the transfer of electrons between non-metals.
D Ionic bonding involves the transfer of electrons from metals to non-metals.

10 Elements W and X have the proton number of 8 and 11 respectively.

Which of the following shows the correct chemical formula and type of compound
formed from the elements?

chemical formula
type of compound
of compound
A XW 2 ionic
B X2W ionic
C WX covalent
D WX 2 covalent

11 Which of the following shows the properties


properrti
ties
e of ca
carbon
arb
rbon
on m
monoxide?
on
o noxide
e?

electrical
el
lec triicall cconductivity
ectr
tr ond
on ductivity
y
melting point / °C
C boiling
bo
b ing point
oililin point / °C
po C
in solid
d state
sta
tate in molten
mol
olte
ten
te n state
A -183
83 -16
-162
16
1 62 poor
pooor
or poor
B 15 104
104
10 poor
pooor poor
p
C 657 997
99 7
99 poor good
D 2040
20
20 3654
3654
36 4 good
good good

12 The
e ta
ttable
ab
blle shows
sh
show
o s th
the
he re
results
es
suulltts off adding di
dilu
dilute
lute
lu te h
hydrochloric
ydroc acid and aqueous sodium
hydroxide
ydrroxidde to
oxiid
ox o an
an oxide.
oxid
de.

Which
ch of
of the
he following
the fol
olllo
o
owi
wiiing
w ng
n ts indicates
g results ndiicates that the oxide is acidic?
ind

result of adding
dilute
dilu
di lute
lu te hydrochloric
hydr acid dilute sodium hydroxide
A no reaction no reaction
B no reaction reaction
C reaction no reaction
D reaction reaction

13 Which of the following options is correct?

acidic oxide basic oxide amphoteric oxide neutral oxide


A carbon monoxide sodium oxide zinc oxide water
B carbon dioxide lead(II) oxide aluminium oxide nitrogen dioxide
C nitrogen dioxide copper(II) oxide potassium oxide nitrogen monoxide
D sulfur dioxide magnesium oxide aluminium oxide water

____________________________________________________________________________________________________
Canberra Secondary School Science (Chemistry) 5105/03, 5105/04
2019 Semestral Assessment Secondary 3 Normal (Academic)

www.KiasuExamPaper.com
162
5

14 Salts can be prepared using the following methods:

1 metal + acid
2 metal oxide + acid
3 metal carbonate + acid
4 metal hydroxide + acid

Which method(s) will lead to the production of a gas as the other product?

A 1 only
B 1 and 3 only
C 2 and 4 only
D 2, 3 and 4 only

15 Aqueous solution P is added to aqueous ammonium nitrate.

The mixture is heated and ammonia gas is given off.

What is P?

A ammonium sulfate
B hydrochloric acid
C sodium hydroxide
D sodium sulfate

16 Which of the following


followi ng correctly
w ng cor
o rect ly matches
cttly mat
atches
at ch
h the
eggas
as
a s and
and its identification
ident
ntif
ntific
if i at
ic atio test?
i n te

gas
gaas test
test and result
re
A am
a
ammonia
mmom n niia mo
moist
oisst red
red litmus
litmus paper remains red
B carbon
carb
bon
on dioxide
dio
i xidede
de lighted
liighted splint extinguishes
C hydrogen
hyydr
d ogge
en n bubble
bubb
bble
bb le gas
gas
a into
int limewater, effervescence observed
D oxygen
oxxyyg
geenn glowing splint relights

17 How are the el


elem
elements
eme
ements arranged
arr in the Periodic Table?

A in order of chemical reactivity


der o
B in order of electron shells
C in order of proton number
D in order of relative atomic mass

____________________________________________________________________________________________________
Canberra Secondary School Science (Chemistry) 5105/03, 5105/04
2019 Semestral Assessment Secondary 3 Normal (Academic)

www.KiasuExamPaper.com
163
6

18 Code letters, T, U, W, X, Y and Z of 6 different elements are placed in the Periodic


Table as shown below.

I II III IV V VI VII 0
T U W
X Y

Which one of the following is a correct statement regarding T, U, W, X, Y and Z?

A T and X are solids with high densities at room temperature.


B X and Y have the same number of electron shells.
C Y is the most reactive element of the 6 elements.
D Z is more reactive than U.

19 Which statement about Group I metals is false?

A Their densities decrease down the group.


B Their melting points decrease down the group.grooup
p.
C Their reactivity increases down thehe group.
th g ou
gr up.
p.
D They react with waterr to form alkalis.
forrm alkali
liis
s..

20 The table lists the halogens


ha
alogens
ns
n s and
nd their
and the ir appearance.
heir ap
pppe
eaarra
ance.
e.

halogen
h
haalo
logen
n appearance
ap
ppe
pe a
arran
an c
cee
fluorine
fluo
uo
ori
rine
n pale
pale
pa e yellow
yellow gas
ga
as
chlorine
chlori
rine
ri ne
e yellowish-green
yellllow
ye o issh-gr
gree
een
ee n gas
bromine
b
brrom
min
ine re
red-brown
ed-
d-br
brrow
b o n liq
liquid
iiodine
io dine
ne
ne dark
da k purple
da pu solid
astatine
as
statiin
nee ?

Astatine would be
be expected
exp
ex pected to be a

A brown gas.
wn gas
B black solid.
C grey liquid.
D white solid.

End of Paper 3

1 2 3 4 5 6 7 8 9 10
B A B C D D B C D B
11 12 13 14 15 16 17 18 19 20
A B D B C D C B A B

____________________________________________________________________________________________________
Canberra Secondary School Science (Chemistry) 5105/03, 5105/04
2019 Semestral Assessment Secondary 3 Normal (Academic)

www.KiasuExamPaper.com
164
7

Paper 4
Section A (14 marks)

Answer all the questions in this section.


Write your answers in the spaces provided.

A1 The list shows the formulae of some substances.

AgNO 3 BaSO 4 CaCO 3 CO NaCl SO 2

For each statement, (a) to (c), select from the list the substance described.

Write each formula in the space provided.

A formula may be used once, more than once, or not at all.

(a) metho
od.
This substance can be prepared using the titration method.

NaCl [1]

(b) This substance can be used to reduce


duce excess acidity
acid
ac itty in
idit n soil.
soi
oiil..

CaCO
Ca 3 [1]

(c) n dissolved
This substance when disso
iis
sso
solvve
edd in water,
wa
ate
t r, turns blue
e llitmus
itm
tmus p
tm aper red..
ap
paper

SO 2 [1]

A2 ta
abblle below
The table be
elo
low gives
giiv
vees information
info
inform
fo rmat
a ion about
ab
a bou
out an
n atom
atto
om of hydrogen
hyd
ydro
r gen and
a an atom of oxygen.

attom
atomm n mb
nu ber o
number off pr
prot
oton
ot o s
protons electronic configuration

hydrogen
hy
yddrrog
og e
enn 1 1

oxygen
o
oxxygen
n 8 2,6

(a) With
h reference
ref
efer
erence to the Periodic Table, complete the table above. [1]

(b) Hydrogen and oxygen can react to form a compound which boils at 100 oC.

Draw the ‘dot and cross’ diagram to show the bonding in the compound formed.
Show all electrons.

Correct sharing [1]


Correct no. of electrons [1]

[2]

____________________________________________________________________________________________________
Canberra Secondary School Science (Chemistry) 5105/03, 5105/04
2019 Semestral Assessment Secondary 3 Normal (Academic)

www.KiasuExamPaper.com
165
8

(c) With reference to its structure and bonding, explain why the compound in (b)
has a low boiling point.

It is a covalent compound [1]

A low amount of energy is required to overcome the weak intermolecular

forces of attraction between the molecules. [1] [2]

A3 Magnesium chloride, MgCl 2 , is a soluble salt. Crystals of magnesium chloride can be


made by reacting dilute hydrochloric acid, HCl, with an excess of powdered
magnesium carbonate, MgCO 3 .

The diagram below shows the first three steps used to prepare crystals
ys of this salt.

(a) (i)
(i
i) Explain
E
Ex p aiin why
xpl why a
wh an excess
ne s of
xcess of powdered
powde
ow
o wde
derre magnesium
ed ma
agn esiium carbonate
gne ca is used.

ensure
To e
ennsure
su
s that
ur th
hat all
at a
alll th
ll tthe
eaacid
cid iis
ac s used
us
u sed up/fully reacted.
ed up/fu

……………………………………………..……………………………………..
……
………
………
………
………
… ……
…………
…………
…… [1]

(ii)
i)) Name
N me
Na me tthe processes
he p roce
ro sses being
cess
ce ss be used in step 2 and step 3 respectively.

process
proc
oces s used
ess used in
i step 2 is filtration

process used in step 3 is crystallisation


proce [1]

(b) The solution left in step 3 is allowed to cool for some time to allow crystals to
grow. Describe the remaining steps required to obtain a sample of pure and dry
magnesium chloride crystals.

Filter the mixture to obtain the crystals as the residue.

Rinse crystals with a little cold distilled water.

Dry crystals between sheets of filter paper. [2]


3 pts [2], 2 pts [1]. 1 pt [0]

____________________________________________________________________________________________________
Canberra Secondary School Science (Chemistry) 5105/03, 5105/04
2019 Semestral Assessment Secondary 3 Normal (Academic)

www.KiasuExamPaper.com
166
9

(c) A student plans to prepare the salt, silver chloride, using the same method
above.

Do you expect the student to be successful in preparing the salt? Explain your
answer.

No silver chloride is an insoluble salt.[1]

It should be prepared via precipitation reaction.[1]

[2]

End of Section A

____________________________________________________________________________________________________
Canberra Secondary School Science (Chemistry) 5105/03, 5105/04
2019 Semestral Assessment Secondary 3 Normal (Academic)

www.KiasuExamPaper.com
167
10

Paper 4
Section B (16 marks)

Answer any TWO questions in this section.


Write your answers in the spaces provided.

B1 A chemist obtained a mixture of three alcohols, ethanol, propanol and butanol.

The table below shows the boiling points of the three alcohols respectively.

alcohol boiling point / oC


ethanol 78
propanol 97
butanol 118

In order to obtain pure samples of each alcohol, the chemist


st sett up
up the experiment as
shown below.

(a) Co
omp
mple
lete
Completete tthe
he ttable
he able bel
elow
el ow.
ow
below.

name of process occurring


name of apparatus
in the apparatus
Boiling
X distillation flask
(A: evaporation)

Y condenser condensation
[2]
4 correct [2], 2-3 correct [1]. 0-1 correct [0]
(b) Suggest an apparatus that could be added to the set up to ensure a more
efficient separation of the three alcohols.

Fractionating column [1]

____________________________________________________________________________________________________
Canberra Secondary School Science (Chemistry) 5105/03, 5105/04
2019 Semestral Assessment Secondary 3 Normal (Academic)

www.KiasuExamPaper.com
168
11

(c) Of the three alcohols present in the mixture, which alcohol will be collected last?
Explain your answer.

Butanol [1]

It has the highest boiling point [1] [2]

(d) Explain the purpose of the boiling stones.

To ensure smooth and even boiling. [1]

…………………………………………………………………………………………… [1]

(e) (i) In the box below, draw a diagram to show the arrangement of particles in
P.

[1]

(ii) cribe the movement


Describe movem
mentt of parti
icclles in P.
particles

Partic
cle
es sl
Particles s ide over o
slide ne
onee anoth
her
er th
another hroughout tthe
throughout he lliquid.[1]
he iquid.[1]
iq

……………………………………………..……………………………………..

……………
… ……
… …………
……
… … ………
……
………
……
……………..…
.………
.…… … [1]

____________________________________________________________________________________________________
Canberra Secondary School Science (Chemistry) 5105/03, 5105/04
2019 Semestral Assessment Secondary 3 Normal (Academic)

www.KiasuExamPaper.com
169
12

B2 (a) John was washing his hair when some shampoo accidentally entered his mouth.
He rinsed his mouth immediately as it left a bitter taste in his mouth. After his
shower, John checked the label of the shampoo bottle. Part of the shampoo’s
ingredients list is shown below.

(i) The chemical that caused the bitter taste in John’s mouth is an alkali that
is also responsible for the soapy feel of the shampoo.

redient
nt tthat
From the ingredient list above, state the ingredient hat ha
ha has caused the
bitter taste in John’s mouth.

Sodium hydroxide [1]

(ii) samp
John brought a small sample ple
le of the e shampoo
sham
sh amp pooo to the he sscience
cience laboratory
ci labo
Wiitth the
and diluted it with water. With th
he permission
pe
p ermmisssi
sion
sion
on of
of his
hiis science
scie
sc ience teacher,
ie teache he
added one dropop of Universal
Un
niv
iver
e sal Indicator
I dica
In atto
or into
into
in to the
th
hee diluted
dililut
uted
ut ed shampoo.
sha
hampoo.

e the colo
State our
u cha
colour hange
ha e th
change hat the
that he
e sstudent
tudent willl expect
ex ctt to
to see
se
ee in tthe
h drop
he dro of
Univer
errs
sa
Universalal Indi
dica
di cato
tor .
Indicator.

Green
n tto
Green ob lu
ue/
e pu
purp
rple
blue/purplele [1]

( iii)
(i
(iii) Unde
Un
Under de su
der
de upervissio
on,
supervision, n John
Joh
ohn di
d
didd further
furthe
er te
test
sts and found out that a typical
st
tests
1 lilitre
itre
tre bottle
tr bo
b ottle of
of shampoo
s a
sh am
mpo
oo contains
c nttai
co a ns 3
300 g of th
the salt, sodium chloride, NaCl.

Ca
C lccul
Calculate ulat
atte
e thethe number
th n mb
nu berr o
off mo
m les of sodium chloride present in a typical
moles
1 litre
lilittrre
re bottle
bottttlle
bo e of shampoo.
sham
sh ampo
ampoo. Round
po Ro off your answer to 3 decimal places.

M r of N
NaCl
aCl
aC
Cl = 2
233+3
35.5 = 58.5 [1]

Molle
Mole
Mo e = 30 / 58.5 = 0.513 mol

No 3 dp, max [1]


N

number of moles of sodium chloride = 0.513 mol [2]

____________________________________________________________________________________________________
Canberra Secondary School Science (Chemistry) 5105/03, 5105/04
2019 Semestral Assessment Secondary 3 Normal (Academic)

www.KiasuExamPaper.com
170
13

(b) 20.0 cm3 of potassium hydroxide solution was placed in a beaker.

The temperature of this solution is measured and recorded.

To the beaker of potassium hydroxide solution, a 2.0 cm3 portion of dilute


hydrochloric acid is added and the mixture is stirred. The highest temperature
reached is measured and recorded.

Seven further 2.0 cm3 portions of hydrochloric acid are added. Each time, the
mixture is stirred and the highest temperature reached is measured and
recorded.

The results for the experiment are shown in the table below.

volume of
acid added / 0.0 2.0 4.0 6.0 8.0 10.0 12.0 14.0
cm3
temperature
of the mixture 21.0 24.2 27.4 30.7 32.9
32.9 31
31.2 2
29.4 27.6
/ oC

Four points from the results above e have


have been
beeen
en plotted
plo
lott
tted
ed on the
th
he grid
grid below and
a a
best-fit line for the four points has been
bee
eenn drawn.
drra
d aww
wn.
n.
n.

(i) On the grid, p


plotllo
ot the
tth
h
hee remaining
re
ema
m ining
g four
fo
our points
poiints using
ussin
ing
g the
e results
res
re sults above.
abov
ab ove.
ov e Ma
Mark
h point w
each iitth a cr
with ross (x).
cross (x).
). [1]

i)
(ii) Draw w a b es
e st-fit lilline
best-fit ine ffor or these ffour
or ou
o ur po
p ints in (b
points (b)(
)(i)). E
(b)(i). xtend this line to
Extend
in
nte
t rsec
ect (cross)
ect
ec
intersect (cro
(c
(cro
oss
sss)) the
tthhe line
liliine
ne already
ne alr
lrrea
lrea
eady
dy drawn.
drra
aw
wnn. [1]

____________________________________________________________________________________________________
Canberra Secondary School Science (Chemistry) 5105/03, 5105/04
2019 Semestral Assessment Secondary 3 Normal (Academic)

www.KiasuExamPaper.com
171
14

p (iii) The acid and the alkali exactly neutralise each other at the point where
the two graph lines intersect (cross).

From your graph, determine the volume of hydrochloric acid required to


exactly neutralise the potassium hydroxide solution.

volume = 7.6 (+/-0.2) cm3 [1]

(iv) Write an ionic equation to show what is meant by the term neutralisation.
Include state symbols.

H+ (aq) + OH- (aq) ї H 2 O (l) [1]

____________________________________________________________________________________________________
Canberra Secondary School Science (Chemistry) 5105/03, 5105/04
2019 Semestral Assessment Secondary 3 Normal (Academic)

www.KiasuExamPaper.com
172
15

B3 (a) The table shows the relative atomic masses, A r , and the melting points of some
Group I metals.

metal relative atomic mass, A r melting point / oC


lithium 7 181
sodium 23 98
potassium 39
rubidium 85 39
caesium 133 28

(i) Plot a graph of melting point against relative atomic mass, marking each
point with a (x). [1]

(ii) Draw a curved line of best fit, taking into account all your plotted points. [1]

____________________________________________________________________________________________________
Canberra Secondary School Science (Chemistry) 5105/03, 5105/04
2019 Semestral Assessment Secondary 3 Normal (Academic)

www.KiasuExamPaper.com
173
16

(iii) Use your graph to predict the melting point of potassium.

76 (+/- 2) oC [1]

(iv) Describe two observations made when sodium is added to water


containing Universal Indicator.

Effervescence observed
Sodium darts on the surface of water and melts away
Mixture turns from green to blue/purple
Yellow flame observed
[2]
Any 2 of the 4
(b) Halogens exist as diatomic molecules and are all found in Group VII of the
Periodic Table.

A teacher bubbles chlorine gas through a colourlesss potassium


pota
ass
ssiu
ium iodide solution.
m io

(i) Write down


d wn a b
do balanced
alan
al ance cchemical
ced chhemic
em
e mic
ical equation
a equ
q ationn for
for the rereaction
reac
acti
ac on that occurs
tion
ti
between
betwee
ee
e e
enn chlorine
c lorine and the
ch e potassium
the pottassi um iodide
siium
s ium iodide solution.
so
olution
on..
on

Cl
Cl 2 + 2KI
2K
2 KI ї 2KCl
2K
2 KCl + I 2 [1]

((i
ii)
i)
(ii) De
es sccri
rb
Describe be
e an observation
obse
ob serrv
vat
a io
on tth
hatt yyou
that ou would
would make
m in this chemical reaction.
G
Gi
ivve
Givee an
an explanation
exxp
plla
annat
a io
on for thee observation.
obse
ob servatio
se

Co
Colo
lourle
lour
urle
urles
Colourlessss
ss olut
uttio
u on ttu
solution urns b
turns brown. [1]

C
Ch
hlorine
lo n
lo
Chlorinenee be
bein
ein
beinging mo
more reactive than iodine, displaces iodine from the [2]
solution.
solu
uttiio
onn. [1
[[1]]

End of Paper 4

____________________________________________________________________________________________________
Canberra Secondary School Science (Chemistry) 5105/03, 5105/04
2019 Semestral Assessment Secondary 3 Normal (Academic)

www.KiasuExamPaper.com
174
17

The Periodic Table of the Elements


Group
I II III IV V VI VII 0
1 2
H He
hydrogen helium
key 1 4
3 4 proton (atomic) number 5 6 7 8 9 10
Li Be atomic symbol B C N O F Ne
lithium beryllium name boron carbon
carb on
n nitrogen
nitroge oxygen fluorine neon
7 9 relative atomic mass 11 12 14 16 19 20
11 12 13 14 15 16 17 18
Na Mg A
All Si P S Cl Ar
sodium magnesium aluminium silicon
silic
silicon phosphorus sulfur chlorine argon
23 24 27 28 31 32 35.5 40
19 20 21 22 23 24 25 26 27 28
8 29
29 30
0 31 32
32 33 34
3 35 36
K Ca Sc Ti V Cr Mn Fe
Fe Co
Co N
Ni C
Cu Zn
n Ga
a Ge
G e As Se Br Kr
potassium calcium scandium titanium vanadium chromium
m manganese
manganes
nes
ese
es iron
iro
rron
ro
o cobalt
coba
ba
ba
balt nickel
nick
ckkel copper zinc
zin
nc gallium
galliu
um germanium arsenic
arse
rsennic selenium
seleniu bromine krypton
39 40 45 48 51 52 55
5 56
6 59
9 59
5 9 64 65
65 70 73 75 79 80 84
37 38 39 40 41 42 43
3 44
44 45
45 46
46 47 48
48 49 50 51 52 53 54
Rb Sr Y Zr Nb M
Mo Tc Ru
Ru Rh
Rh Pd
Pd A
Ag Cd n
In S
Sn Sb
S Te I Xe
rubidium strontium yitrium zirconium niobium molybdenum
moly
ybden
num
u ttechnetium
technetium ruthenium rhodium
rhod
ho
o ium palladium
palladiu
diu
ium
iu
um ssilver
silv er cadmium indium
indi
n um
nd m tin antimony tellurium iodine xenon

175
85 88 89 91 93
3 96
96 - 101
1 103
103 106
1 06
06 108
10 112 115
1 119 122 128 127 131
55 56 57 – 71 72 73
73 74
74 75
5 76
6 77 78
78 79
79 80 81
81 82 83 84 85 86
Cs Ba lanthanoids
noids
Hff Ta W Re
Re Os
O s Irr Ptt
P Au Hg Tl Pb Bi Po At Rn
caesium barium hafnium
um
u m tantalum
um
m tungsten
st
st rhenium
m osmium
osmi
smi
ssm
mium
iu iridium
iirid
ir
rrid
diium
iu
u platinum
plat
pla
p la
atinum
a gold
d mercury
m
merc ury thallium lead bismuth polonium astatine radon
133 137 1788 181 184
84
4 186 19
190
190
9 0 192
19
192
9 2 195
195 197 201 204 207 209 - - -
87 88 89 03
3
9 - 103 104
04
04 105 106
6 107 08
0
108 8 109
09
09 110 11
111 112
11 114 116

www.KiasuExamPaper.com
Fr Ra R
Rf D
Db Sg
Sg Bh
h Hs
Hs M
Mt Ds Rg Cn Fl Lv
francium radium actinoids
oids Rutherfordium
Ruthe
herford
rford
diium
um
m dubnium
dubn
ubn
bniu
bn ium
ium
um seaborgium
se
s
seab
eab
eaborgi
o um
or bohrium
boh
bohr
bo
b oh
ohr
o h ium
ium
m hassium
ha
hass
asssium
a i meitnerium
m darmstadtium
darmstadti
darms tadtium
ta
tadti roentgenium
roentge copernicium flerovium livermorium
- - - - - - - - - - - - -

57
7 58
58 9
59 60
60 61 62 63 64 65 66 67 68 69 70 71
lanthanoids La Ce
e Prr
P Nd Pm Sm Eu Gd Tb Dy Ho Er Tm Yb Lu
lanthanum
hanum cerium
ceriu
ceri
cer um praseodymium
prase
prase
eod
o
odymi
ody um neodymium
neod
dymiu
ymiumm promethium samarium europium gadolinium terbium dysprosium holmium erbium thulium ytterbium lutetium
139
39 140
0 141 144 - 150 152 157 159 163 165 167 169 173 175
89 90
90 91 92 93 94 95 96 97 98 99 100 101 102 103
actinoids Ac Th
T h Pa U Np Pu Am Cm Bk Cf Es Fm Md No Lr
actinium thorium protactinium
protacti
protac nium uranium
ur neptunium plutonium americium curium berkelium californium einsteinium fermium mendelevium nobelium lawrencium
- 232
32
2 231 238 - - - - - - - - - - -
3
The volume of one mole of any gas is 24 dm
d at room temperature and pressure (r.t.p.)
www.KiasuExamPaper.com
176
East Spring Secondary School
Towards Excellence and Success

Name: …………………………………………………………………………………… ( )

Class: ……………………

Second Semestral Examinations 2019


Secondary 3 Normal Academic

Science (Chemistry) 5105/03


Paper 3 Multiple Choice

TUESDAY Paper 3 and 4: 1 hour 15 minutes


8 OCTOBER 2019 0800 – 0915

Additional Material: OTAS

INSTRUCTIONS TO CANDIDATES

Write in soft pencil.


Do not use staples, paper clips, highlighters, glue or correction fluid.
Write your name, class and register number in the spaces provided on the paper and on your
OTAS.

There are 20 questions on this paper. Answer all questions. For each question there are 4
possible answers A, B, C and D.
Choose the one you consider correct and record your choice in soft pencil on the OTAS.
Calculators and mathematical sets are allowed.

You are advised not to spend more than 30 minutes on Paper 3.


You may proceed to answer Paper 4 as soon as you have completed Paper 3.

Answers to Paper 3 and Paper 4 must be handed in separately.

A copy of the Periodic Table is on page 8.

20
This question paper consists of 8 printed pages including the cover page.

www.KiasuExamPaper.com
177
2

Answer all questions by shading on the OTAS.

1 Which of the following laboratory apparatus should be used to measure 27.20 cm3
of dilute hydrochloric acid?

A pipette B burette

C beaker D measuring cylinder

2 A gas is denser than air and insoluble in water.


Which method cannot be used to collect the gas?
A B

C D

3 Chlorobenzene, C 6 H 5 Cl, is a commonly used solvent.

What is the number of atoms present in chlorobenzene?

A 2 B 3 C 11 D 12

www.KiasuExamPaper.com
178
3

4 Which of the following is most likely a mixture?

A an odourless, colourless liquid


B a shiny solid that can conduct electricity
C a dye that gives two spots on the chromatogram
D a pale-green gas that cannot be broken down to simpler substances

5 Seawater is heated using the apparatus set-up shown below.

At which point is the temperature 100°C?

6 A student wishes to extract a coloured solution from some berries to make an


indicator solution.
1 crush the berries
2 add acid
3 add a solvent
4 filter the mixture
5 distil the filtrate

Which of these instructions should the student follow?


A 1, 2 and 4 B 1, 3 and 4 C 3, 4 and 5 D 2, 5 and 4

www.KiasuExamPaper.com
179
4

7 What happens to the molecules in a liquid when it turns into a solid at its freezing
point?

A They cool down.


B They gain energy.
C They lose energy.
D They move faster.

8 The table shows the melting points of four different substances.

Which substance is a pure solid at a temperature of 20°C?

melting point / °C

A 32

B 25 to 28

C -20 to -30

D -35

96
9 An atom of molybdenum is represented as 42 Mo.

How many electrons does one atom of molybdenum contain?


A 42 B 54 C 96 D 138

10 One isotope of hydrogen is represented by the symbol 11 H.

Which diagram represents another isotope of hydrogen?

A B C D

key
2p 2p 1p 1p = electron
2n 1n 2n 1n p = proton
n = neutron

www.KiasuExamPaper.com
180
5

11 Which of the following correctly describes the formation of a positive ion?


A An atom gains electrons.

B An atom gains protons.

C An atom loses electrons.

D An atom loses protons.

12 Covalent bonds are formed when electrons are ……1…… .

Most covalent componds have …..2….. electrical conductivity.

Which words correctly complete gaps 1 and 2?

1 2

A shared high

B shared low

C transferred high

D transferred low

13 Hydrogen reacts with nitrogen to form ammonia as shown by the chemical equation
below:
x H 2 (g) + y N 2 (g) Æ z NH 3 (g)

What are the values for x, y and z needed to balance the equation?

x y z

A 1 2 2
B 2 2 1
C 3 1 2
D 3 2 2

www.KiasuExamPaper.com
181
6

14 Aqueous solution X is added to aqueous ammonium chloride.


The mixture is heated and ammonia gas is given off.
What is X?
A ammonium sulfate B hydrochloric acid
C sodium chloride D sodium hydroxide

15 When aluminium, sulfur and calcium burn in oxygen, oxides are formed.

Which row identifies the type of oxides that is formed by each one of them?

aluminium oxide sulfur dioxide calcium oxide


A acidic basic amphoteric
B amphoteric acidic basic
C amphoteric basic acidic
D basic acidic amphoteric

16 A student was given dilute hydrochloric acid and the four solids listed below.

1 magnesium
2 magnesium carbonate
3 magnesium sulfate
4 magnesium oxide

Which of these solids can be used to prepare magnesium chloride?

A 1, 2 and 3 only B 1, 2 and 4 only


C 1, 3 and 4 only D 2, 3 and 4 only

17 Potassium sulfate is a soluble salt that is used as a food additive.


Which of the following chemicals are suitable in the preparation of potassium sulfate?
A potassium hydroxide and sulfuric acid
B sodium nitrate and potassium sulfate
C potassium nitrate and sulfuric acid
D sodium hydroxide and potassium sulfate

www.KiasuExamPaper.com
182
7

18 Element X, Y and Z are in Group VII of the Periodic Table.


X is a gas.

Y is less reactive than Z.

Z is a red liquid.

X, Y and Z are placed in order of increasing proton number.

Which order is correct?


A ;ĺ<ĺ= B ;ĺ=ĺ<
C <ĺ;ĺ= D YĺZ ĺX

19 Lithium, sodium and potassium are all in the same group of the Periodic Table.

Which statement about these elements are correct?


A They all have the same number of electrons in their atoms.

B They all have the same number of electrons in their outer shell.

C They all have the same number of protons in their atoms.

D They all have the same total number of protons plus neutrons in the nuclei of
their atoms.

20 What is the approximate volume of oxygen in 200cm3 of air?

A 10cm3

B 20cm3

C 40cm3

D 200cm3

End of Paper 3

www.KiasuExamPaper.com
183
8

www.KiasuExamPaper.com
184
East Spring Secondary School
Towards Excellence and Success

Name: …………………………………………………………………………………… ( )

Class: ……………………

Second Semestral Examinations 2019


Secondary 3 Normal Academic

Science (Chemistry) 5105/04


Paper 4

TUESDAY Paper 3 and 4: 1 hour 15 minutes


08 OCTOBER 2019 0800 – 0915

Candidate answer on the Question Paper.


No additional materials are required.

INSTRUCTIONS TO CANDIDATES

Write your name, class and register number in the spaces provided above, and on all the
work you hand in.
Write in dark blue or black pen.
You may use a soft pencil for any diagrams, graphs, tables or rough working.
The use of an approved scientific calculator is expected, where appropriate
Do not use staples, paper clips, highlighters, glue or correction fluid.

Section A
Answer all questions.
Write your answers in the space provided on the question paper.

Section B
Answer any two questions.
Write your answers in the space provided on the question paper.

Answer all questions in the spaces provided.


The number of marks is given in brackets [ ]
at the end of each question or part question.
A copy of the Periodic Table is printed on page 11.
30
This question paper consists of 11 printed pages including the cover page.

www.KiasuExamPaper.com
185
2

Section A [14 Marks]


Answer all questions in the spaces provided.
1 Sulfur dioxide is one of the pollutants that is carefully monitored by the National Environment
Agency (NEA) in Singapore.
It can react with raindrops to produce acid rain.

(a) Suggest one major source of sulfur dioxide production in Singapore.


………………………………………………………………………………………….……….[1]

(b) State the effect of acid rain on the environment.


…………………………………………………………………………………………...……..….

……………………………………………………………………………………………..……[1]

(c) Name another gas pollutant that has the same effect as sulfur dioxide on the
environment.
…………………………………………………………………………………….……….……[1]

(d) Acid rain can cause the soil in community gardens to be too acidic for plant growth.

(i) Name the chemical that can be added to the soil to make it suitable for plant
growth.
………………………………………………………………………..…………………[1]

(ii) Explain, in terms of the chemistry involved, the effect the chemical in (d)(i) has on
the soil.
………………………………………………………………………………….………….

……………………………………………………………………………….…….……[1]

www.KiasuExamPaper.com
186
3

2 Figure 2.1 shows the arrangement of particles in substances A to F.

Figure 2.1

Which of the diagrams A, B, C, D, E and F best represents

(a) a solid element,


……………………………………………………………………………………….………[1]

(b) a compound,
……………………………………………………………………………………………….[1]

(c) a mixture of element and compound.


…………………………………………………………………………………………….…[1]

www.KiasuExamPaper.com
187
4

3 (a) A teacher uses the apparatus in Figure 3.1 to bubble chlorine gas through colourless
lithium bromide solution.

chlorine gas

lithium bromide
solution

Figure 3.1
(i) Describe an observation that can be made during this chemical reaction.
…………………………………………………………………………………....….…[1]

(ii) Explain why a reaction can occur between chlorine gas and lithium bromide
solution.
………………………………………………………………………….…….…..….……
………………………………………………………………………….…….…..….…[1]

(b) Table 3.2 shows the observation of three Group I metals when they were placed in a
beaker of water.

Metal observation

P reacts vigorously with water with a yellow flame

Q reacts quickly with water

R reacts violently with water with a lilac flame

Table 3.2
(i) With reference to Table 3.2, arrange the metals in the order of increasing
reactivity.
………………………………………………………………………………..…..……[1]

(ii) One of the metals that was added to water was sodium. Sodium react with water
to form sodium hydroxide and hydrogen gas.
Write a balanced chemical reaction, with state symbols, for the reaction between
sodium and water.
…………………………………………………………………………….…...…….…[2]

www.KiasuExamPaper.com
188
5

(iii) Describe what you would observe when a few drops of Universal Indicator is
added to the reaction mixture.
………………………………………….……………………………………………....[1]

www.KiasuExamPaper.com
189
6

Section B [16 marks]


Answer two out of three questions in the spaces provided.

4 A student carried out an experiment to determine how fast magnesium carbonate reacts with
hydrochloric acid by measuring the volume of gas X formed at regular intervals.
The equation for the reaction is as follows:
hydrochloric acid + magnesium carbonate ĺmagnesium chloride + water + gas X

(a) Describe the test and observation for gas X that is formed.
test: ………………………………………..………………………………………………...…..
observation: …………………………………………………………………………………….
……………………………………………………………………………………………...…[2]

(b) Magnesium hydroxide is also able to react with hydrochloric acid to form
magnesium chloride and water.
Write an ionic equation, with state symbols for the reaction between
magnesium hydroxide and hydrochloric acid.
…………………………………………………………………………………………………[1]
(c) When preparing salts by this method, just enough hydrochloric acid is added to react
exactly with all the magnesium hydroxide.
What is the pH of the solution formed at the end of the reaction to produce
magnesium chloride?

pH = …………………………….[1]
(d) Draw a ‘dot-and-cross’ diagram to show the arrangement in magnesium chloride.
Show the outer electrons only.

[2]

www.KiasuExamPaper.com
190
7

(e) State and explain whether magnesium chloride has a high or low melting point.
……………………………………………………………………………………………………

……………………………………………………………………………………………………

…………………………………………………………………………………………………[2]

5 Table 5.1 gives information about a fluorine atom and an atom of element Y.

fluorine atom atom of element Y

number of protons 6

number of electrons 9 6

arrangement of electrons 2.7


Table 5.1

(a) Complete Table 5.1. [1]

(b) Name element Y.


…………………………………………………………………………………………………[1]

(c) Draw a diagram to show the electronic configuration of the fluorine atom.

[1]
(d) Describe, in terms of electrons, how a fluorine atom becomes a fluoride ion.
…………………………………………………………………………………………………....
…………………………………………………………………………………………………[1]

www.KiasuExamPaper.com
191
8

(e) Compound Z is formed when element Y reacts with fluorine.


Z has a melting point of -184°C and a boiling point of -128°C.

A sample of compound Z was heated from -200°C to -100°C.

(i) Sketched a graph of temperature/°C against time/min to show the changes in


temperature as compound Z is heated from -200°C to -100°C.

temperature/°C

time/min
[2]

(ii) Describe how the arrangement and movement of the particles in compound Y
changes as it is heated from -200°C to -150°C.
………………………………………………………………………………………………..

……………………………………………………………………………………………….

……………………………………………………………………………………………….

………………………………………………………………………………….………...[2]

www.KiasuExamPaper.com
192
9

6 Table 6.1 shows the solubility of some substances in cold water.

substances solubility in cold water

lead (II) chloride insoluble

calcium chloride soluble

calcium sulfate insoluble

silver carbonate insoluble

sodium nitrate soluble

silver nitrate soluble

silver chloride insoluble

Table 6.1

(a) State the method used to prepare a sample of silver chloride.


………………………………………………………………………………………………....[1]
(b) With reference to Table 6.1, name a substance that can react with silver nitrate to
produce silver chloride.
……………………………………………………………………………………………….…[1]
(c) Describe how a pure sample of silver chloride can be obtained after mixing the two
substances in (b) together.
……………………………………………………………………………………………………..
……………………………………………………………………………………………………..
…………………………………………………………………………………………………….
………………………………………………………………………………………………....[3]

www.KiasuExamPaper.com
193
10

(d) Silver nitrate can also be used as an additive in chromatography to separate organic
compounds. Figure 6.2 shows a chromatogram comparing a sample with five known
substances after a silver nitrate chromatography.

sample A B C D E

Figure 6.2
(i) State the substance(s) that are present in the sample.
…….…………………………………………………………………………………….[1]

(ii) State one conclusion that can be made about substances A, B, C and D.
…………..………………………………………………………………………………….
………………….……………………………………………………………………….[1]

(iii) Explain why no spot is observed for substance E.


………………………..…………………………………………………………………….
……………………………….………………………………………………………….[1]

End of paper 4

www.KiasuExamPaper.com
194
11

www.KiasuExamPaper.com
195
www.KiasuExamPaper.com
196
ESSS 2019 3NA Science Chemistry SA2
Paper 3

1 2 3 4 5 6 7 8 9 10
B B D C B B C A A D
11 12 13 14 15 16 17 18 19 20
C B C D B B A B B C

Paper 4

1a Burning of sulfur-containing coal/fossil fuel in power stations. 1


Reject volcano
1b It can increase acidity/lower pH in water bodies causing g aquatic
aqua
uatitic life to Any 1 - 1
die. /
It can corrode limestone/marble building.
1c Nitrogen dioxide / oxides of nitrogen 1
1di Calcium hydroxide/Slaked lime 1
Accept calcium oxide/quicklime
1dii Calcium hydroxide neutralizes the exce
excess
cess
ce s ac
acid
ciid
d pr
p
pres
present
res
esen
ent in
n the ssoil
oill
oi 1
/increases pH of the soil.

2a B 1
2b A 1
2c E 1

3ai The solution


so
s olutiion
on turn n from
from
fr m co
c
colourless
lourrlle
lo ess to o reddish-brown.
redd
re ddisish-brown n. 1
3aii
ii Chlorine
C lo
Ch orriine is
s more
m re
mo e reactive
reactivve th
tthan
ha
ann bromine
broommiinne
e ssoo chlorine
chlo
lori
rine
ne ccan
an d
displace 1
bromine
brom
br om
o mini e from m li
llithium
th
hiuum brom
bromide
ommid de so
ssolution.
olu
luti
tionn.
3bi Q,, P
Q P,, R 1
3bii Na
a ((s)
s) + H 2 O (l ĺ
(l ĺ1D2+ DT +
ĺ 1D D2
2++ DT
DT + + 2 (g
(g) eqn -1
ss - 1
3biii The
he Universal
Uniiv
Un ver
ersa
al IIn
Indicator
nddiica
c tor turns
turn
rns
rn s (from
(fr
from green
g to) violet. 1

4a Test: bu
bubble
ubb
b le th
the
he ga
gass into li
limewater. 1
Observation:
vatio
on: A wh
w
white
ite p
precipitate is formed. 1
4b H+ (aq) + O
OHH- DT
DT ĺ+ 2 O (l) 1
4c 7 1
4d Mg – 1
both Cl –
- xx 2+ - 1
Cl x x Mg x Cl x
x x
xx
- electron of Cl
X – electron of Mg

www.KiasuExamPaper.com
197
4e Magnesium chloride has a high melting point. 1
Large amount of energy is required to overcome strong electrostatic 1
forces of attraction between oppositely-charged ions/strong ionic
bonds between ions.

5a Fluorine number of proton: 9 1


arrangement of electrons for X: 2.4
5b carbon 1
5c 1

5d Fluorine has an electronic configurationn of 2.7/7 va


valence
alelenc
nce
nc e electrons.
elec
ctr
tron
ons.
on s. 1
It gains one electron to achieve stable
ble noble gas
s electronic
ele
elle
eccctr
trro
onnic
ic
configuration.
5ei Label
Labe -1
temperature / °C
Shape
S ape -1
Sh

-100

-128
8

-18
18
84
-184

-200
-20
00
time / min

5eii The arrangement


anggem
e ent cchanges from very closely packed in orderly manner 4c-2
at -200°C
C to cl
closely packed in disorderly/random manner at -150°C. 2-3c-1
The movement of the particles changes from vibrating about fixed 0-1c-0
positions at -200°C to sliding past one another randomly at -150°C.

www.KiasuExamPaper.com
198
6a Precipitation 1
6b Calcium chloride 1
6c Filter the mixture to obtain silver nitrate as residue. 1

Wash/Rinse the residue with distilled water. 1

Dry residue between filter paper. 1

[Order of steps must be correct]


6di C and D 1
6dii They are pure substances as they have (only one spot above the start 1
line).
6diii Substance E is not soluble in the solvent. 1

www.KiasuExamPaper.com
199
www.KiasuExamPaper.com
200
HILLGROVE SECONDARY SCHOOL
END-OF-YEAR EXAMINATION 2019
SECONDARY 3 (NORMAL ACADEMIC)

CANDIDATE ( ) CLASS
NAME 3 -

CENTRE INDEX
NUMBER S NUMBER

SCIENCE (CHEMISTRY) 5015


7 October 2019

Candidates answer on the Question Paper. 1 hour

Additional Materials: OTAS sheet 08:00 AM to 09:00 AM

READ THESE INSTRUCTIONS FIRST


Write your class, index number and name on the work you hand in.
You may use an HB pencil for any diagrams, graphs, tables or rough working.
Write in dark blue or black pen.
Do not use staples, paper clips, glue or correction fluid.

The use of an approved scientific calculator is expected, where appropriate.


You may lose marks if you do not show your working or if you do not use appropriate units.

Section A
There are fifteen questions in this section. Answer all questions. For each question there are four
possible answers A, B, C and D.
Choose the one you consider correct and record your choice in soft pencil on the separate answer
sheet (OTAS).

Section B and C
Answer all questions.
Write your answers in the space provided on the question paper.

The number of marks is given in brackets [ ] at the end of each question or part question.
A copy of the Periodic Table is printed on page 11.

For Examiner’s Use


Section Marks

A 15
B 11
C 14
Parent’s/ Guardian’s Signature: ___________________ TOTAL 40

This document consists of 11 printed pages.

www.KiasuExamPaper.com
201
2

Section A: Multiple Choice Questions


Answer all questions. Record your answers in the OTAS provided.

1 Which method is used to obtain glass pieces from aqueous sodium chloride?

A chromatography
B crystallisation
C distillation
D filtration

2 Which piece of apparatus is used to measure only a fixed quantity of liquid?

3 Which is neither an element nor a compound?

A air
B ammonia
C carbon monoxide
D chlorine

4 What decides the order of arrangement of the elements in the Periodic Table?

A density
B number of neutrons
C number of protons
D relative atomic mass

5 Which is/are the main particle(s) that contribute to the mass of an atom?

A electron and neutron


B electron and proton
C neutron and proton
D proton only

www.KiasuExamPaper.com
HGV EOY 2019 3NA Science (Chemistry) 5105
202
3

6 The diagram shows the arrangement of electrons in the atoms of four different elements.
Which element is the least reactive?

7 The experiment shown is used to test potassium bromide crystals.

The lamp does not light up.

Distilled water is then added to the beaker and the lamp lights up.

Which statement best explains these observations?

A Crystals are free to move in water.


B Electrons are free to move in water.
C Ions are free to move in water.
D The lamp is unreliable.

8 Which statement about hydrogen is not correct?

A Each hydrogen atom has a noble gas electronic structure.


B Each hydrogen molecule has one single covalent bond.
C Hydrogen exist as diatomic molecules.
D Hydrogen exist as simple covalent molecules.

www.KiasuExamPaper.com
HGV EOY 2019 3NA Science (Chemistry) 5105
203
4

9 Covalent bonds are formed when electrons are ……1…… .


Most covalent compounds have ……2…… electrical conductivity.

Which words correctly complete gaps 1 and 2?

10 Which equation for the reaction between sodium carbonate and dilute hydrochloric acid is correct?

A Na2CO3 + 2HCl → 2NaCl + CO2 + H2O


B Na2CO3 + HCl → NaCl + CO2 + H2O
C Na2CO3 + 2HCl → NaCl + CO2 + H2O
D Na2CO3 + HCl → Na2Cl + CO2 + H2O

11 Substance Q has the following properties:

1. it conducts electricity when molten


2. it has a high melting point
3. it dissolves in water

What can Q be?

A carbon
B gold
C oil
D sodium chloride

12 The chart shows the colour of Universal Indicator at different pH values.

A solution of lemon juice is only slightly acidic.


Which colour does Universal Indicator give with this solution?

A blue
B orange
C red
D violet

www.KiasuExamPaper.com
HGV EOY 2019 3NA Science (Chemistry) 5105
204
5

13 The diagram shows the soil pH range over which a vegetable grows well.

The pH of the soil initially provided is 5.5.

Why is slaked lime (calcium hydroxide) added to this soil before planting the vegetable?

A The lime adjusts the soil acidity.


B The lime is a fertiliser.
C The lime is a pH indicator.
D The lime speed up reactions.

14 The oxide of element X forms a solution with pH 4.

The oxide of element Y forms a solution that turns Universal Indicator blue.

Which row correctly classifies elements X and Y?

element X element Y
A metal metal
B metal non-metal
C non-metal metal
D non-metal non-metal

15 Calcium carbonate is the main chemical component of chalk.

When vinegar is added to powdered chalk, bubbling of a colourless gas is observed. This gas forms
a white precipitate with limewater.

What is the identity of the gas?

A carbon dioxide
B hydrogen
C nitrogen
D oxygen

www.KiasuExamPaper.com
HGV EOY 2019 3NA Science (Chemistry) 5105
205
6

Section B
Answer all questions in the space provided.

1 Fig. 1.1 shows the structures of some substances containing carbon.

Fig. 1.1

Answer the following questions about these substances.


Each substance may be used once, more than once or not at all.

(a) Which substance(s), A, B or C,

(i) contains strong electrostatic forces of attraction, …………………

(ii) has atoms that share two pairs of electrons with each other, …………………

(iii) contains three elements? …………………


[3]

(b) With reference to its structure, explain why substance B has a low melting point.

[1]

[Total: 4]

2 The incomplete diagram shows the outer shell of electrons in a chlorine molecule, Cl2.

Complete the electronic structure with dots and crosses. Show only the outer shell electrons.

[Total: 2]

www.KiasuExamPaper.com
HGV EOY 2019 3NA Science (Chemistry) 5105
206
7

3 Acids react with bases to form salts.

Table 3.1 shows some reactions of acids and bases and the salts formed. The first row has
been completed for you.
Table 3.1

acid base salt formed

sulfuric acid sodium hydroxide sodium sulfate

……………. acid zinc oxide zinc nitrate

hydrochloric acid magnesium chloride

Complete the table by filling in the two empty boxes.

[Total: 2]

4 Table 4.1 shows the concentration of some ions present in a sample of seawater.

Table 4.1

(a) Which positive ion in Table 4.1 has the lowest concentration?

[1]

(b) Give the name of the ion with the formula I–.

[1]

(c) Which two ions in Table 4.1 are formed from elements in Group II of the Periodic Table?

[1]

[Total: 3]

www.KiasuExamPaper.com
HGV EOY 2019 3NA Science (Chemistry) 5105
207
8

Section C
Answer all questions in the space provided.

5 Iron is a commonly used metal. It can react with many types of substances.

(a) An isotope of iron has 58 nucleons.

Complete Table 5.1 to show


• the number of electrons and neutrons in this isotope of iron,
• the relative charges on each particle.

Table 5.1
number of particle relative charge
particle present on the particle

electron

neutron 0

proton 26
[2]

(b) Iron reacts with hydrochloric acid to form iron(II) chloride and a gas which ‘pops’ with a
lighted splint.

Complete the chemical equation with appropriate numbers and chemical formulae for this
reaction.

Fe + ....... HCl ЍFeCl2 + ....... .............


[2]

(c) Fig. 5.1 shows a measuring cylinder containing FeCl2 solution, as shown below.

Fig. 5.1

(i) What is the volume of the solution?

[1]

(ii) Suggest another measuring instrument that can take more accurate volume
measurement in the laboratory.

[1]

[Total: 6]

www.KiasuExamPaper.com
HGV EOY 2019 3NA Science (Chemistry) 5105
208
9

6 A student wants to find out if flower petals contain one coloured substance or two. He follows
the procedure shown in Fig. 6.1.

Fig. 6.1

(a) When he puts the chromatography paper in ethanol, suggest and explain whether the
starting line should be above or below the surface of the ethanol.

[2]

(b) The student concludes that there are two substances in the petal colour.
On Fig. 6.2, show what the chromatography paper looks like after the experiment.

Fig. 6.2

[2]

[Total: 4]

www.KiasuExamPaper.com
HGV EOY 2019 3NA Science (Chemistry) 5105
209
10

7 Fig. 7.1 shows apparatus used to distil seawater.

Fig. 7.1

(a) At which point(s), X, Y or Z, is/are the temperature exactly 100 °C?

[1]

(b) Use arrows to indicate on the diagram where water should enter and exit the condenser.
[1]

(c) On Fig. 7.1, draw and label what should be added to the setup to make boiling smooth.
[1]

(d) Suggest the name of the apparatus that must be added to the setup to separate a mixture
of water and ethanol.

[1]

[Total: 4]

www.KiasuExamPaper.com
HGV EOY 2019 3NA Science (Chemistry) 5105
210
11

www.KiasuExamPaper.com
HGV EOY 2019 3NA Science (Chemistry) 5105
211
www.KiasuExamPaper.com
212
3NA CHEM EOY 2019 MARK SCHEME
1 2 3 4 5 6 7 8 9 10
D D A C C B C A B A
11 12 13 14 15
D B A C A

1 a I A;
Ii B;
iii A;
b Weak intermolecular forces of attraction / van der Waals forces + low energy required to
overcome ;
2 2 electrons in the shared space ;
6 electrons in the unshared space in the shell of the chlorine atom
om on
o the right ;

3 Nitric ;
Magnesium
esium oxide
oxxide / magnesium
o magn
ma gnes
esiuium hydroxide
hydr
hy drox de ;
oxiid
2++
2
4 a Calcium ion / Ca
Ca ;
b Io
Iodide
odi
d de
de ;
c Caa2+ an
C and M 2+
nd Mg 2
(A
(A:
A: name
names
mes o
me off ion
ions)
ns)
s) ;
5 A number
nu
n u
umb
mber o
mb off particle
part
pa rtic
rticlle pre
ic present relative charge on the
paarrtticlee
p
particle particle

elec
eelectron
el
leeccttrro
onn 26 -1
neutron
ne
n euuttron 32 0
proton on 26 +1
Mark
Mar rk by
rk by column
colu
lumn
lu mn ;;
b H 2 in the
the llast
ast blank ;
as
fficiients: 2 and 1 (A: no number in front of H 2 . R: if wrong or no formula is filled instead
Coefficients:
of H 2 an
and the equation is not balanced with the coefficients) ;
c i 55 cm3 ;
ii Burette ;
6 a Line above ethanol ;
Prevent spot of colour from dissolving in ethanol and not travel up the chromatography
paper ;
b Original spot of colour shown in the same position ;
Two spots shown (vertically) above the spot of colour ; R: two spots horizontally aligned
7 a Y;
B Arrow indicating water in at the bottom inlet + water out at the upper outlet (words not
necessary, reject if words conflict with arrows) ;
c Drawing + label of boiling chips at the bottom of the flask ;
d Fractionating column ; R: other spellings like fractional

1
www.KiasuExamPaper.com
213
www.KiasuExamPaper.com
214
NAME: ( ) CLASS: 3A

HOUGANG SECONDARY SCHOOL


SEMESTRAL ASSESSMENT 2 / 2019

SCIENCE (CHEMISTRY) 5105/03


PAPER 3

SECONDARY THREE NORMAL (ACADEMIC)

Tuesday, 08 October 2019 Total duration for Papers 3 and 4:


1 hour 30 minutes
MAKE THE DIFFERENCE RESPECT OURSELVES RESPECT OTHERS MAKE THE DIFFERENCE RESPECT OURSELVES RESPECT OTHERS MAKE THE DIFFERENCE
MAKE THE DIFFERENCE RESPECT OURSELVES RESPECT OTHERS MAKE THE DIFFERENCE RESPECT OURSELVES RESPECT OTHERS MAKE THE DIFFERENCE
MAKE THE DIFFERENCE RESPECT OURSELVES RESPECT OTHERS MAKE THE DIFFERENCE RESPECT OURSELVES RESPECT OTHERS MAKE THE DIFFERENCE
MAKE THE DIFFERENCE RESPECT OURSELVES RESPECT OTHERS MAKE THE DIFFERENCE RESPECT OURSELVES RESPECT OTHERS MAKE THE DIFFERENCE
MAKE THE DIFFERENCE RESPECT OURSELVES RESPECT OTHERS MAKE THE DIFFERENCE RESPECT OURSELVES RESPECT OTHERS MAKE THE DIFFERENCE
MAKE THE DIFFERENCE RESPECT OURSELVES RESPECT OTHERS MAKE THE DIFFERENCE RESPECT OURSELVES RESPECT OTHERS MAKE THE DIFFERENCE

READ THESE INSTRUCTIONS FIRST

Write in soft pencil.


Do not use staples, paper clips, glue or correction fluid.
Write your name, register number and class on the Answer sheet (OTAS) and Question papers in
the spaces provided.

There are twenty questions on this paper. Answer all questions. For each question there are four
possible answers, A, B, C and D.
Choose the one you consider correct and record your choice in soft pencil on the OTAS provided.

Answers to Paper 3 and Paper 4 must be handed in separately.


Each correct answer will score one mark. A mark will not be deducted for a wrong answer.
You are advised to spend no more than 30 minutes on Paper 3.
You may proceed to answer Paper 4 as soon as you have completed Paper 3.
Any rough working should be done in this booklet.
A copy of the Periodic Table is printed on page 8.
The use of an approved scientific calculator is expected, where appropriate.

This document consists of 8 printed pages (including this cover page).


[Turn over

www.KiasuExamPaper.com
215
2

1 Which statement about the behaviour of the particles in a gas is incorrect?

A There are large spaces between the particles.


B They are able to move at high speeds.
C They are arranged in regular patterns.
D They spread throughout the container.

2 Ganymede is the largest moon of the planet Jupiter and has a surface temperature of -179 qC.
The table shows the substances found on the surface of Ganymede.

substance melting point / qC boiling point / qC


hydrogen 259 252
ethane 183 89
nitrogen 210 196
methane 182 162
argon 189 186
water (trace amount) 0 100

What substance(s) is/are likely to exist in liquid state on the surface of Ganymede?

A argon, hydrogen and nitrogen only


B ethane and methane only
C hydrogen and nitrogen only
D water only

3 Two correct methods of collecting a gas X are shown.

X water

Which properties of gas X are shown by these collection methods?

density of X water solubility of X


A less than air high
B less than air low
C more than air high
D more than air low

www.KiasuExamPaper.com
216
3

o
4 Which technique can be used to separate a mixture of ethanol (boiling point 79 C) and methyl
o
benzene (boiling point 111 C)?

A crystallisation
B evaporation
C filtration
D fractional distillation

5 Which of the following facts about kerosene shows that it is a mixture?

A It boils over the range 170 oC to 250 oC.


B It burns in air to produce carbon dioxide and water.
C It consists of two elements, carbon and hydrogen.
D It is obtained from crude oil.

6 A white solid, Z, is produced when magnesium reacts with oxygen.


Which of the following best describes Z, magnesium and oxygen respectively?

Z magnesium oxygen

A compound element element

B element compound compound

C element mixture mixture

D mixture compound compound

7 The atoms of element X have the electronic configuration 2.8.7.


Which statement about element X is correct?

A It does not form isotopes.


B It forms an ion with a charge of 1+.
C It has 17 protons in the nucleus of its atom.
D It is a metallic element.

www.KiasuExamPaper.com
217
4

8 The diagram shows the nuclei of four different atoms, P, Q, R and S.

P Q R S

Which two atoms are isotopes of each other?

A P and Q
B P and S
C Q and R
D Q and S

9 Which of the following statements about sodium atoms and sodium ions is true?

A They have the same number of electrons.


B They have the same number of protons.
C They have the same number of protons and electrons.
D They have the same number of valence electrons.

10 The ‘dot and cross’ diagram shows a molecule of hydrogen chloride.

Cl

How do the atoms bond together?

A The atoms shared a pair of electrons.


B The atoms shared a pair of protons.
C The hydrogen atom gives the chlorine atom a proton.
D The hydrogen atom gives the chlorine atom an electron.

11 Which of the following pair is not likely to form an ionic compound?

A aluminium and nitrogen


B magnesium and oxygen
C nitrogen and oxygen
D sodium and chlorine

www.KiasuExamPaper.com
218
5

12 Two statements were made on the properties of ionic compounds.

Statement 1
Ionic compounds can conduct electricity in the molten state but not in the solid state.

Statement 2
Ionic compounds contain free-moving ions when molten or dissolved in water but not in
the solid state.

Which of the following is true?

A Only statement 1 is correct.


B Both statements are correct.
C Both statements are correct and statement 1 explains statement 2.
D Both statements are correct and statement 2 explains statement 1.

13 The diagram shows models of covalent molecules.

X Y Z

What is the chemical name of each of these molecules?

X Y Z
A hydrogen ammonia water
B hydrogen water ammonia
C ammonia hydrogen water
D water ammonia hydrogen

14 The equation shows the reaction that occurs when ethene is burnt in air.

C 2 H 4 + xO 2 ĺyCO 2 + zH 2 O

What are the values of x, y and z?

x y z
A 3 1 1
B 3 2 2
C 4 1 2
D 4 2 1

www.KiasuExamPaper.com
219
6

15 Which of the following equations is balanced?

A 2Al + 3H 2 O Æ Al 2 O 3 + 2H 2
B Mg + O 2 Æ MgO 2
C 4Na + O 2 Æ 2Na 2 O
D S + O 2 Æ SO3

16 What is the relative formula mass of Na2SO4?

A 70
B 71
C 124
D 142

17 A 18 g sample of pure carbon is completely burned in oxygen, as shown in the equation.


C + O2 Æ CO2

What volume of carbon dioxide gas is produced at room temperature and pressure?

A 12 dm3
B 24 dm3
C 36 dm3
D 48 dm3

18 Citric acid is a white solid.

Which statement shows that it must be an acid?

A It changes the colour of indicators.


B It contains oxygen atoms.
C It gives hydrogen ions in water.
D It is soluble in water.

19 Ammonium chloride is heated with substance Q and the reaction produces ammonia gas.
What type of substance is Q?

A acid
B base
C metal
D salt

www.KiasuExamPaper.com
220
7

20 The diagram shows the pH values of the soil in two parts of the garden of a house, X and Y.
The house owner wishes to use lime to neutralise the soil in one part of the garden.

To which part should the lime be added, and why?

part of garden reason


A X Lime is acidic.
B X Lime is basic.
C Y Lime is acidic.
D Y Lime is basic

www.KiasuExamPaper.com
221
8

www.KiasuExamPaper.com
222
1

NAME: ( ) CLASS: 3A

HOUGANG SECONDARY SCHOOL


SEMESTRAL ASSESSMENT 2 / 2019

SCIENCE (CHEMISTRY) 5105/04


PAPER 4

SECONDARY THREE NORMAL (ACADEMIC)

Tuesday, 08 October 2019 Total duration for Papers 3 and 4:


1 hour 30 minutes
MAKE THE DIFFERENCE RESPECT OURSELVES RESPECT OTHERS MAKE THE DIFFERENCE RESPECT OURSELVES RESPECT OTHERS MAKE THE DIFFERENCE
MAKE THE DIFFERENCE RESPECT OURSELVES RESPECT OTHERS MAKE THE DIFFERENCE RESPECT OURSELVES RESPECT OTHERS MAKE THE DIFFERENCE
MAKE THE DIFFERENCE RESPECT OURSELVES RESPECT OTHERS MAKE THE DIFFERENCE RESPECT OURSELVES RESPECT OTHERS MAKE THE DIFFERENCE
MAKE THE DIFFERENCE RESPECT OURSELVES RESPECT OTHERS MAKE THE DIFFERENCE RESPECT OURSELVES RESPECT OTHERS MAKE THE DIFFERENCE
MAKE THE DIFFERENCE RESPECT OURSELVES RESPECT OTHERS MAKE THE DIFFERENCE RESPECT OURSELVES RESPECT OTHERS MAKE THE DIFFERENCE
MAKE THE DIFFERENCE RESPECT OURSELVES RESPECT OTHERS MAKE THE DIFFERENCE RESPECT OURSELVES RESPECT OTHERS MAKE THE DIFFERENCE

READ THESE INSTRUCTIONS FIRST

Write your name, register number and class on all the work you hand in.
Write in dark blue or black pen on both sides of the paper.
You may use an HB pencil for any diagrams or graphs.
Do not use staples, paper clips, glue or correction fluid.

Answer all questions in Section A and any two questions in Section B.


The use of an approved scientific calculator is expected, where appropriate.
In calculations, you should show all the steps in your working, giving your answer at each stage.
You are advised to spend no longer than 30 minutes on Paper 3.
You may proceed to answer Paper 4 as soon as you have completed Paper 3.
A copy of the Periodic Table is printed on page 11.

At the end of the examination hand in your answers to Paper 3 and Paper 4 separately.
The number of marks is given in brackets [ ] at the end of each question or part question.

FOR EXAMINER’S USE


PAPER 3 PAPER 4
TOTAL
Section A Section B

20 24 16 60

This document consists of 11 printed pages (including this cover page).


[Turn over

www.KiasuExamPaper.com
223
2

Section A

Answer all the questions in the spaces provided.

1 The following is a list of separation techniques used to separate mixtures.

chromatography fractional distillation evaporation


filtration crystallisation

Which of these separation techniques is used to separate each of the underlined substance
from the stated mixture?

(a) copper(II) sulfate crystals from copper(II) sulfate solution

.…………………………………………….. [1]

(b) red dye from a black ink

................…………………………………. [1]

(c) nitrogen from liquid air

…………………………………………….. [1]

2 The set of apparatus is used in the preparation of crystals of the salt, copper(II) chloride,
CuCl 2 .

Y
Z

X
gentle heat

(a) Name apparatus X, Y and Z.

X …………………………..………

Y …………………………………..

Z ………………………………….. [2]

(b) What is the name of an apparatus that can be used to measure accurately the
volume of a liquid?

……………………………………………………………………………………………….. [1]

www.KiasuExamPaper.com
224
3

3 (a) The following shows a list of substances.

air carbon chlorine steel


water hydrogen oxygen zinc

From the list given, identify

(i) a gaseous mixture, …………………………………. [1]

(ii) a metallic element, …………………………………. [1]

(iii) a pure compound. …………………………………. [1]

(b) A student claims that an impure sample of an element melted at a fixed temperature of
65 oC during an experiment.

Explain why the student’s claim is wrong.

..……………………………………………………………………………………………….

………………………………………………………………………………………………… [1]

4 Balance the following equations.

(a) ....... H 2 O 2 (l) Æ ……. H 2 O (l) + ……. O 2 (g) [1]

(b) …… Cl 2 (g) + …… KBr (aq) Æ …….KCl (aq) + …… Br 2 (aq) [1]

(c) …… C 3 H 8 (g) + …… O 2 (g) Æ ……. CO 2 (g) + ....... H 2 O (g) [1]

www.KiasuExamPaper.com
225
4

5 The table shows information about some elements.

element proton number arrangement of electrons

lithium
2, 1
neon
10
potassium
19
oxygen
2, 6

(a) Complete the table by filling in the blank spaces. [2]

(b) Which elements belong to the same group? Explain your answer.

…………………..…………………………………………………………………………….

………………………………………………………………………………………………… [1]

(c) Draw a diagram to show the electronic structure of an oxygen atom.

[1]

(d) Unlike the other three elements, neon does not bond with other atoms. Explain why.

……………………………………………………………………………………………….. [1]

www.KiasuExamPaper.com
226
5

6 X is an unknown element. It has been determined that 0.2 mole of XO has a mass of 8 g.

(a) Define relative atomic mass.

…………………………………………………………………………………………………

………………………………………………………………………………………………… [1]

(b) Calculate the relative atomic mass of element X and suggest its identity.

Identity of element X is ………………….. [2]

7 Mrs Kay wants to grow cucumbers in her garden. Cucumbers require soil that is slightly
alkaline.

The soil in the garden has a pH of 5.0.

(a) Explain why the soil in this garden is not suitable for growing cucumbers.

……………………………………………………………………………………………… [1]

(b) Name a chemical that she can add to the soil to make it suitable for planting
cucumbers.

……………………………………………………………………………………………… [1]

(c) Describe a test to check the change of pH after adding the chemical stated in (b).

……………………………………………………………………………………………… [1]

End of Section A

www.KiasuExamPaper.com
227
6

Section B

Answer any two questions from this section in the spaces provided.

8 Sodium chloride, NaCl can be made by burning sodium in chlorine gas.

(a) Describe, in terms of electrons, how a chlorine atom becomes a chloride ion.

…………………………………………………………………………………………………

………………………………………………………………………………………………… [1]

(b) The diagram shows the arrangement of sodium ions and chloride ions in a crystal
of sodium chloride.

Cl

Na

(i) Explain why sodium chloride has a high melting point.

…………………………………………………………………………………………

………………………………………………………………………………………… [2]

(ii) Give another physical property of sodium chloride other than the one
mentioned in b(i).

…………………………………………………………………………………………. [1]

(iii) Draw a ‘dot and cross’ diagram in the space below to show the arrangement
of the electrons in NaCl. You are required to show only the valence electrons.

[2]

www.KiasuExamPaper.com
228
7

(c) Chlorine, Cl 2 , is a poisonous gas if it is inhaled in large amounts. However, it is


used in industries in small amounts due to its useful properties.

Draw a ‘dot and cross’ diagram in the space below to show the arrangement of the
electrons in Cl 2 . You are required to show only the valence electrons.

[2]

9 (a) The salt, magnesium chloride, is commonly used in fire extinguishers and can be
obtained by the reaction of magnesium hydroxide and hydrochloric acid.

(i) Name the type of reaction between magnesium hydroxide and hydrochloric
acid.

………………………………………………………………………………………… [1]

(ii) Magnesium chloride can also be made from the reaction of magnesium
carbonate and hydrochloric acid.

Name the gas produced in this reaction and describe a test to identify this
gas.

name of gas …………………………………………………………………………

test ……………………………………………………………………………………

…………………………………………………………………………………………

observation ………………………………………………………………………….

………………………………………………………………………………………… [3]

www.KiasuExamPaper.com
229
8

(b) The following is part of a label found on a bottle of mineral water.

Typical analysis
ions mg/dm3

bicarbonate (HCO 3 -) 225.0


calcium (Ca2+) 55.0
chloride (Cl-) 9.0
magnesium (Mg2+) 15.0
nitrate (NO 3 -) 3.5
potassium (K+) 0.5
sodium (Na+) 5.0
sulfate (SO 4 2-) 9.0

dry residue at 180 oC 210.0 mg/dm3


pH 8.0

(i) How many diffferent metal ions are present in the mineral water?

………………………………………………………………………………………… [1]

(ii) Iron(II) sulfate is an essential body mineral. Iron can be obtained from the
food that we eat.

Deduce the chemical formula of iron(II) sulfate.

………………………………………………………………………………………… [1]

(iii) The mineral water is tested with Universal indicator.

What colour would you expect to see and what does the pH tell you about the
mineral water?

colour ….. ………………………………………………………………………….

mineral water is ……………………………………………………………………. [2]

www.KiasuExamPaper.com
230
9

10 (a) The diagram shows particles in four different substances.

A B C D

(i) Suggest an identity for substances A and C.

A …………………………………

C ………………………………… [2]

(ii) Substance A represents a compound whereas substance B represents a


mixture.

State two differences between compounds and mixtures.

1……………………………………………………………………………………….

………………………………………………………………………………………..

2……………………………………………………………………………………….

………………………………………………………………………………………… [2]

www.KiasuExamPaper.com
231
10

(b) Benzene, C 6 H 6 , is an organic compound.


A sealed container filled with solid benzene was gently heated until only benzene
vapour was present. The graph shows the temperature changes against time.

temperature/ oC
D
C

time / min

(i) Benzene has a melting point of 5 qC and a boiling point of 80 qC. Predict the
temperature at region A.

………………………………………………………………………………………… [1]

(ii) Draw a diagram to show the particles in benzene at region B.

[1]

(iii) In terms of the kinetic particle theory, explain the changes that happened to
the benzene particles, in terms of the arrangement and movement of the
particles as benzene is heated from 0 °C to 15 °C.

…………………………………………………………………………………………

…………………………………………………………………………………………

………………………………………………………………………………………… [2]

End of Paper 4

www.KiasuExamPaper.com
232
11

www.KiasuExamPaper.com
233
www.KiasuExamPaper.com
234
SA2_2019
Sec 3NA Science (Chemistry) 5105
Answer Key
Paper 3 [20 marks]

Qn Ans E/M/H KwU/ A Qn Ans E/M/H KwU / A


A1 C E KwU A11 C E KwU
A2 B H A A12 D H KwU
A3 B M kWU A13 B M A
A4 D E A A14 B M KwU
A5 A E A A15 C M KwU
A6 A E kWU A16 D E KwU
A7 C M KwU A17
7 C M KwU
A8 A E KwU A18 C E A
A9 B E kWU A19
A1
19 B M KwU
Kw
A10 A E KwU A2
A
A20
20 D E A

HOT Questions = 2/20


20 = 10
1
10%% L
LO
LOT
T Questi
Questions
ion
ons
s = 11/2
11/20
/20
/2 0 = 55
55%

er 4 (40
Paper (40
40 marks)
marks)
ma

Se
Sectio
on A [24
Section [2
24 ma
marks] Marks E/M/H

1 (a) Crystallisation
Crry
yssta
talllissa
attiio
on 1 E
(b) Ch
Chromatography
hrro
oma
matograp aphy
ap hy 1 E
(c) Fractional
Fraction nal distillation
disisti
t llatio 1 E
[3]
2 (a) X – beaker
beake 1 m – one/two
Y – filter
t funnel correct E
Z – evaporating dish 2 m - all correct
(b) Pipette or burette 1 E
[3]
3 (a) i air 1 M
ii zinc 1 M
iii water 1 E
(b) An impure sample of the element will not melt at a fixed 1 M
temperature/ An impure sample of the element will melt at a
lower temperature / An impure sample of the element will
melt within a range of temperature.
[4]

www.KiasuExamPaper.com
235
4 (a) 2 H 2 O 2 (l) Æ 2 H 2 O (l) + 1 O 2 (g) 1 E
(b) 1 Cl 2 (g) + 2 KBr (aq) Æ 2 KCl (aq) + 1 Br 2 (g) 1 E
(c) 1 C 3 H 8 (g) + 5 O 2 (g) Æ 3 CO 2 (g) + 4 H 2 O (g) 1 M
[3]
5 (a) element proton number arrangement of electrons
E
lithium 3 2, 1

2,8 1 –each
neon 10
correct pair
potassium 19 2, 8, 8, 1
Tot:2
oxygen 8 2, 6

(b) Lithium and potassium.


Both atoms have the same number of valence 1 E
electron/have one electron in its outer shell. Hence
nce they
ey
are in Group I

(c)
1 E
Oxygen atom
Correct no of
O shells &
correct
ectt number
correc
ec numbe
of electrons
ele
lectron

(d)
d) N
Ne
Neon
eon nh has
as a completely
co
omple
mp ple
lete
tely
l filled
fil
illed
d outer
oute
ou t rs sh
shell
heelll of electrons/
ele
lect
le ctro
ct ons/ ha
has a 1 M
full
ull valence
fu val
alle
enncee shell
shell
sh
[Reject:
[Rej
[R e ect: Neone n iis
Neo s a noble
nob
ob
o le gas/unreactive/has
ble gas//u
unnrreacctt ve/
c
ctiv e/ha
has
ha s a stable
stab
electronic
elle
e eccctr
tron
tr onicc shell]
shell]]
[5]
6 (a) Relative
Re
R elaati
t ve
ve atomic
ato
ommiic m
ma
mass
ass iis
s the
the average
averag mass of one atom of
av
an
an element
eleeme nt compared
ent co
c omppar ed to
ared to 1/12 the
t mass of a carbon-12 1 E
atom
atto
omm
(b) Mola
Molar
lla
lar
a mass ss of XO
XO = 88/0.2 = 40 1 H

X + 166 = 40
X = 24
1
X is Magnesium
[3]
7 (a) The soil is acidic whereas cucumbers need a slightly 1 M
alkaline condition to grow.
(b) Calcium hydroxide / calcium oxide/calcium carbonate 1 E
Lime/slaked lime/quicklime
[Reject: limewater]
(c) Dissolve some soil in water /make a soil solution and 1 H
use a pH meter or Universal indicator to test the soil
condition

www.KiasuExamPaper.com
236
[Reject: use universal indicator/litmus paper even if the
observations are mentioned in the answer]
[3]
24

HOT Questions = 3/24 = 12.5%

LOT Questions (E) = 15/24 = 62.5 %

Section B (16 marks) Marks E/M/H

8 (a) A chlorine atom gains an electron to form a chloride ion. 1 E


(b) i A lot of energy is required to overcome the strong 1
electrostatic forces of attraction between the oppositely
oppositeely 1 H
charged ions/ Na+ and Cl- ions
ii M
1 – ccorrect
+ - ch
charge on the ions

1 – correc
correct
Cl number of
of valen
valence
electrons
elec
electr
tro
ons

iii Good
Go
G oodd cconductor
on
o n ucttor
ndu o off e
el
electricity
lec
ectr
tric
icitty when
wh
when
en molten
mol
oltten orr in
in aqueous
aq 1 E
state/
s ate/
st (any one)
Soluble
luble in
Solu
So n water
wat
at er
(c)) 1 – correct M
number of bonded
electrons

1 – correct
number of non-
bonding electrons

[Minus
us mark if all the electrons are not drawn in dots or cross]
[8]
9 (a) i neutralisation 1 E
ii Carbon dioxide 1 E
Bubble the gas into limewater 1 M
White precipitate is formed in limewater 1 H

(b) i 4 1 M
ii FeSO 4 1 H
iii Bluu- green /blue 1 M
Slightly alkaline /weakly alkaline/ alkaline 1
[8]

www.KiasuExamPaper.com
237
10 (a) i A: Water / carbon dioxide (or any appropriate answer) 2 E
C: any diatomic molecule
ii Compounds can be separated into its components by M
chemical methods but mixtures are separated by physical 2
methods. Any two

Chemical properties of the mixture is the same as its


components while the physical and chemical properties of a
compound is different from those of its constituent elements.

No energy change when a mixture is formed while an energy


change takes place when a compound is formed.

The components of a mixture can be mixed in any ratio while


the elements in a compound are always combined chemically
ally
in a fixed ratio.
(b) i 5 oC 1 E
ii

1
E

liquid
d

iii Movement: x arrangement


arra
ar rang
ngem
e ent H
At 0OC, the p particles
articles es
e s are
arre
e vibrating
viib
b
bra
rati
ra ting
ng ab
a
about
bou
o t th
their
heir fixe
fixed
ed x movement
moveme
mo
position.
osition. When
Wh
W hen
heenn heated
heatted to 15 5 C,
O C tthe
he sub
substance
ub
u bs
sttan
a ce melted
mel
elte
el ted and
te and
its particles
pa
p
part
art
rt
rticles
s st
taarrtte
started ed too move
mo ov
ve a awwayy, sliding
away, s id
sl idin
ng past
pa
p ast each
eacch otothe
h r
other 2
tthroughout
th roug hout tthe
gho e lliquid
he iqui
iqui
iq ui d

A
Ar rangememmen
Arrangement: en t :
At 0OC
At C,, th
the
he pparticle
artic
iccle
le in
in the
the solid state
th stat
ate
at e are
are closely
close packed in
an
a orderly
orrd
deerly mamanner.
m nnerr. When
ann Wh
W hen heated
hea
eaate
tedd to 15 C
OC, the particles are
now
no
n ow slslightly
s ig
gh y far
httly fa apart
arr a p rt in
pa disorderly
n a di s rder manner.
diso

[8]
total [24]

HOT Questions = 5/ 24 = 20.8 %


LOT (Easy) Questions = 8/24 = 33.3 %

www.KiasuExamPaper.com
238
www.KiasuExamPaper.com
239
KENT RIDGE SECONDARY SCHOOL
END-OF-YEAR EXAMINATION 2019

SCIENCE (CHEMISTRY) 5105/3

PAPER 3

SECONDARY 3 NORMAL (ACADEMIC)

Monday 07 October 2019 1 hour 15 minutes


KENT RIDGE SECONDARY SCHOOL KENT RIDGE SECONDARY SCHOOL KENT RIDGE SECONDARY SCHOOL KENT RIDGE SECONDARY SCHOOL KENT RIDGE SECONDARY SCHOOL KENT RIDGE
KENT RIDGE SECONDARY SCHOOL KENT RIDGE SECONDARY SCHOOL KENT RIDGE SECONDARY SCHOOL KENT RIDGE SECONDARY SCHOOL KENT RIDGE SECONDARY SCHOOL KENT RIDGE
KENT RIDGE SECONDARY SCHOOL KENT RIDGE SECONDARY SCHOOL KENT RIDGE SECONDARY SCHOOL KENT RIDGE SECONDARY SCHOOL KENT RIDGE SECONDARY SCHOOL KENT RIDGE
KENT RIDGE SECONDARY SCHOOL KENT RIDGE SECONDARY SCHOOL KENT RIDGE SECONDARY SCHOOL KENT RIDGE SECONDARY SCHOOL KENT RIDGE SECONDARY SCHOOL KENT RIDGE

Name: ___________________________( ) Class: Sec __________

ADDITIONAL MATERIALS
Multiple Choice Answer Sheet

READ THESE INSTRUCTIONS FIRST


Do not open this question paper until you are told to do so.

Write in soft pencil.


Do not use staples, paper clips, highlighters, glue or correction fluid.
Write your name, class and index number on the Answer Sheet and in the spaces provided at
the top of this page.

There are twenty questions on this paper. Answer all questions. For each question, there are
four possible answers A, B, C and D.
Choose the one you consider correct and record your choice in soft pencil on the separate
Answer Sheet.

Read the instructions on the Answer Sheet very carefully.

Each correct answer will score one mark. A mark will not be deducted for a wrong answer. Any
rough working should be done in this booklet.

Answers to Paper 3 and Paper 4 must be handed in separately.

You are advised to spend no more than 30 minutes on Paper 3.

You may proceed to answer Paper 4 as soon as you have completed Paper 3.

_____________________________________________________________________________________
This Question Paper consists of 8 printed pages, including this page.

Setter: Ms Siti Fadzilah [Turn over]

www.KiasuExamPaper.com
240
Section A
The total mark for this section is 20.

1 A student wishes to add exactly 20.15 cm3 of acid to exactly 25.0 cm3 of an alkali in a titration
experiment.

Which apparatus would be most suitable to measure these volumes?

acid alkali
A pipette measuring cylinder
B pipette burette
C measuring cylinder burette
D burette pipette

2 The graph shows the change in temperature with time when ice at -20oC is heated to 120oC?

Which row correctly describes the change taking place when ice is heated?

points change
A P to Q average energy of particles remains constant
B Q to R ice melting
C R to S the volume of steam is increasing
D T to U water boiling

Secondary 3 Normal (Academic) 2 5105/03 Science (Chemistry)


Kent Ridge Secondary School End-of-Year Examination 2019

www.KiasuExamPaper.com
241
3 A piece of dry ice becomes smaller very quickly when left in the open air without any trace
of liquid unlike normal ice.

Which diagram shows the correct change in the arrangement of the carbon dioxide
molecules in dry ice when left in the open air?

4 Which substance is a liquid at room temperature, 25 oC?

melting point /°C boiling point /°C


A -77 -33
B -1 58
C 78 329
D 180 1330

5 Which of the following substances is likely to be a pure compound?

A A blue substance that melts exactly at 300 oC.


B A yellow powder that is insoluble in water.
C A colourless compound.
D A suspension that separates into two portions when allowed to settle.

Secondary 3 Normal (Academic) 3 5105/03 Science (Chemistry)


Kent Ridge Secondary School End-of-Year Examination 2019

www.KiasuExamPaper.com
242
6 A student placed some substances into three different boxes as shown below.

I II III

What is the correct identity of the molecular representations of the substances?

I II III
A air water vapour oxygen
B oxygen air water vapour
C oxygen water vapour air
D water vapour air oxygen

7 Which substance has been wrongly classified as element, compound or mixture?

property classification
A black powder burns in air to form a colourless gas element
B colourless solution produces two colourless gases when an element
electric current is passed through it

C green powder on heating leaves a black residue and a compound


colourless gas is evolved

D white solid melts over 56 °C – 58 °C mixture

8 Benzoic acid has the molecular formula of C 7 H 6 O 2 . How many atoms are present in a
molecule of benzoic acid?

A 3
B 9
C 10
D 15

Secondary 3 Normal (Academic) 4 5105/03 Science (Chemistry)


Kent Ridge Secondary School End-of-Year Examination 2019

www.KiasuExamPaper.com
243
9 What is the correct relative mass of each particle in an atom?

proton electron neutron


1
A 1 1
1840
1
B 1 1
1840
1
C 1 1
1840
1 1
D 1
1840 1840

10 The diagram shows a particle of an element in Period 2.

e- e-

e- - e-
e- e e- e-

e- e-

What could the particle be?

A an atom of carbon
B an atom of neon
C an ion of boron
D an ion of nitrogen

11 The electronic configuration of some atoms are shown below.

Which atom forms an ion with a positive charge?

A 2, 5
B 2, 8, 2
C 2, 8, 6
D 2, 8, 8

Secondary 3 Normal (Academic) 5 5105/03 Science (Chemistry)


Kent Ridge Secondary School End-of-Year Examination 2019

www.KiasuExamPaper.com
244
12 The diagram shows the molecule of a compound.
Which pair of elements could Y and Z be?

Z Y Z

X – electron of Y
– electron of Z

Y Z
A calcium chlorine
B carbon oxygen
C sodium chlorine
D sulfur chlorine

13 The table shows the properties of some substances.

Which substance could be sodium chloride?

conducts electricity
melting point / °C
when liquid in aqueous solution
A -114 none none
B -114 none good
C 180 none insoluble
D 808 good good

14 Aluminium sulfate reacts with calcium hydroxide according to the chemical equation below.

w Al 2 (SO 4 ) 3 + x Ca(OH) 2 ĺy Al(OH) 3 + z CaSO 4

What values of w, x, y, and z can balance the above equation?

w x y z
A 1 1 1 2
B 1 3 2 3
C 2 3 2 3
D 2 3 3 1

Secondary 3 Normal (Academic) 6 5105/03 Science (Chemistry)


Kent Ridge Secondary School End-of-Year Examination 2019

www.KiasuExamPaper.com
245
15 The graph below shows the colour ranges of the acid-base indicators methyl orange,
bromothymol blue and phenolphthalein.

A few drops of different indicators were added into a solution separately. The colour of the
solution turned yellow in methyl orange, yellow in bromothymol blue and colourless in
phenolphthalein.

What is the pH range of the solution?

A 2.5 to 3.5
B 4.5 to 5.5
C 7.5 to 8.5
D 9.5 to 10.5

16 Workers in a factory accidently spilled some sulfuric acid on the floor. Which of the following
would leave an alkaline mixture when added in excess?

A calcium hydroxide
B copper(II) nitrate
C magnesium carbonate
D silver powder

17 Which statement about the Periodic Table is correct?

A All groups contain both metals and non-metals.


B Atoms of elements in the same group have the same number of electron shells.
C On Group 0, reactivity increases with increasing proton number.
D The number of valence electrons in an atom increases when moving across the period
from left to right.

Secondary 3 Normal (Academic) 7 5105/03 Science (Chemistry)


Kent Ridge Secondary School End-of-Year Examination 2019

www.KiasuExamPaper.com
246
18 A noble gas Z is used to fill weather balloons.

Which descriptions of Z are correct?

number of outer electrons


in an atom of Z structure of gas Z

A 1 diatomic molecule
B 1 single atoms
C 2 single atoms
D 2 diatomic molecules

19 The chart below shows the melting points of the elements in Group I of the Periodic Table.
The melting point for potassium is missing.

temperature/°C

200
Li
150

100
Na
50
Rb
Cs
0

What is the likely melting point of potassium?

A 0 °C
B 63 °C
C 98 °C
D 180 °C

20 Which of these gases cause acid rain?

A carbon dioxide and methane


B carbon dioxide and nitrogen dioxide
C nitrogen dioxide and sulfur dioxide
D sulfur dioxide and methane

End of Paper 3

Secondary 3 Normal (Academic) 8 5105/03 Science (Chemistry)


Kent Ridge Secondary School End-of-Year Examination 2019

www.KiasuExamPaper.com
247
KENT RIDGE SECONDARY SCHOOL
END-OF-YEAR EXAMINATION 2019

SCIENCE (CHEMISTRY) 5105/4

PAPER 4

SECONDARY 3 NORMAL (ACADEMIC)

Monday 07 October 2019 1 hour 15 minutes


KENT RIDGE SECONDARY SCHOOL KENT RIDGE SECONDARY SCHOOL KENT RIDGE SECONDARY SCHOOL KENT RIDGE SECONDARY SCHOOL KENT RIDGE SECONDARY SCHOOL KENT RIDGE
KENT RIDGE SECONDARY SCHOOL KENT RIDGE SECONDARY SCHOOL KENT RIDGE SECONDARY SCHOOL KENT RIDGE SECONDARY SCHOOL KENT RIDGE SECONDARY SCHOOL KENT RIDGE
KENT RIDGE SECONDARY SCHOOL KENT RIDGE SECONDARY SCHOOL KENT RIDGE SECONDARY SCHOOL KENT RIDGE SECONDARY SCHOOL KENT RIDGE SECONDARY SCHOOL KENT RIDGE
KENT RIDGE SECONDARY SCHOOL KENT RIDGE SECONDARY SCHOOL KENT RIDGE SECONDARY SCHOOL KENT RIDGE SECONDARY SCHOOL KENT RIDGE SECONDARY SCHOOL KENT RIDGE

Name: ___________________________( ) Class: Sec __________

READ THESE INSTRUCTIONS FIRST


Do not open this question paper until you are told to do so.

Write your name, class and index number in the spaces provided at the top of this page.
Write in dark blue or black pen on both sides of the paper.
You may use a soft pencil for any diagrams or graphs.
Do not use staples, paper clips, highlighters, glue or correction fluid.

Section A
Answer all questions. FOR EXAMINER’S USE

Section B PAPER 3
Answer any two questions. 20
Write your answers in the spaces provided on the question
paper. PAPER 4
Section A 14
In calculations, you should show all the steps in your working,
giving your answer at each stage.
PAPER 4
Enter the numbers of the Section B questions you have Section B 16
answered on the dotted lines in the grid. A copy of the Periodic
Table can be found on page 11. Q………

Q………

TOTAL
50

_____________________________________________________________________________________
This Question Paper consists of 11 printed pages, including this page.

Setter: Ms Siti Fadzilah [Turn over

www.KiasuExamPaper.com
248
Section A
Answer all the questions.
The total mark for this section is 14.

1 Part of the Periodic Table is shown in Fig. 1.1. The letters A to G are not real symbols
of the elements.

Fig. 1.1

(a) Using letters A to G, answer (i) and (ii)

(i) Which two elements exist as gas at room temperature?

(ii) Which element forms an amphoteric oxide?

[2]

(b) State whether each of the following statements is True or False.

(i) Element B is more reactive than G.

(ii) Element G has a higher melting point than B.

[2]

Secondary 3 Normal (Academic) 2 5105/04 Science (Chemistry)


Kent Ridge Secondary School End-of-Year Examination 2019

www.KiasuExamPaper.com
249
2 The formulae of ionic compounds can be deduced from the charges on the ions
present. The following list shows the symbols of some common ions.

‫ܪ‬ା ‫ܽܥ‬ଶା ‫݃ܯ‬ଶା ‫ ݈ܣ‬ଷା

‫ି ݈ܥ‬ ܱଶି ܱܵସଶି ܱܰଷି

Write the chemical formula for each of the following compounds.

(a) calcium oxide

(b) magnesium chloride

(c) calcium sulfate

(d) aluminium nitrate [2]

Secondary 3 Normal (Academic) 3 5105/04 Science (Chemistry)


Kent Ridge Secondary School End-of-Year Examination 2019

www.KiasuExamPaper.com
250
3 Plants make a variety of coloured pigments. A student extracted blue colouring from
four different plants, R, S, T and U. The student put a spot of each colouring on a
piece of filter paper. The filter paper was dipped into a solvent and left for 30 minutes.

The results are shown in Fig. 3.1.

Fig. 3.1

(a) Suggest a suitable solvent for the experiment.

[1]

(b) Which plant contained the greatest number of different pigments?

[1]

(c) Which two plants contained the same pigments?

[1]

Secondary 3 Normal (Academic) 4 5105/04 Science (Chemistry)


Kent Ridge Secondary School End-of-Year Examination 2019

www.KiasuExamPaper.com
251
4 Phosphorus is a non-metal. Fig. 4.1 shows a molecule of phosphorus oxide.

Fig. 4.1

(a) Complete Table 4.1 to show the number of electrons, neutrons and protons in a
phosphorus atom.

Table 4.1

symbol number of electrons number of neutrons number of protons


ଷଵ
ଵହܲ [1]

(b) With reference to Fig. 4.1, what is the molecular formula of phosphorus oxide?

[1]

(c) Is the bonding on phosphorus oxide ionic or covalent?

[1]

(d) Phosphorus reacts with hydrogen to form phosphorus hydride, PH 3 .

Draw a ‘dot and cross’ diagram of phosphorus hydride. Only show the outer
shell electrons.

[2]

Secondary 3 Normal (Academic) 5 5105/04 Science (Chemistry)


Kent Ridge Secondary School End-of-Year Examination 2019

www.KiasuExamPaper.com
252
Section B

The total mark for this section is 16.


Answer any two questions for this section in the spaces provided.

5 (a) Magnesium metal was reacted with dilute hydrochloric acid.

Bubbles of gas was observed, as seen in Fig. 5.1

Fig. 5.1

(i) Complete the equation, by stating the name of the salt and gas produced.

magnesium hydrochloric
+ Æ +
metal acid

(ii) Describe a positive test to identify the gas in (a)(i).

test:

result: [4]

Secondary 3 Normal (Academic) 6 5105/04 Science (Chemistry)


Kent Ridge Secondary School End-of-Year Examination 2019

www.KiasuExamPaper.com
253
(b) A farmer wanted to plant a new crop which only grows well on neutral soil. He
took a sample of the soil from his land and varied the pH of the soil by adding
different amount of lime to it.

Fig 5.2 shows the graph of how the pH of the sample of soil changes as lime is
added.

Fig. 5.2

Use the graph to answer the following questions.

(i) Is the soil acidic or alkaline before adding lime?

Give a reason for your answer.

(ii) What is the pH of soil required by the farmer to plant the new crop?

(iii) What mass of lime must then be added to the soil so that it is suitable for
growing the new crop?

(iv) After adding 4.0 g of lime to the soil sample, a small amount of the soil was
removed and added to water. A few drops of Universal indicator were then
added to the mixture.

What will be the colour of Universal indicator in this mixture?

[4]

Secondary 3 Normal (Academic) 7 5105/04 Science (Chemistry)


Kent Ridge Secondary School End-of-Year Examination 2019

www.KiasuExamPaper.com
254
6 (a) Fluorine, chlorine, bromine, iodine and astatine are elements in Group VII.

(i) State the name given to Group VII elements.

(ii) Going down the group, from fluorine to astatine, describe two trends in their
properties.

(ii) In an experiment, chlorine was added to aqueous potassium fluoride.


However, no visible change was observed. Briefly explain why.

[4]

(b) Table 6.1 shows the information of elements in Group I.

Table 6.1

element symbol atomic number electronic structure


lithium Li

sodium Na 11

potassium K 2,8,8,1

(i) Complete the table with the atomic number and electronic structure for the
elements in Group I.

(ii) Using data from Table 6.1, suggest the reason why lithium, sodium and
potassium are found in the same group?

(iii) Caesium is another element in Group I. Write the formula of a caesium ion.

[4]

Secondary 3 Normal (Academic) 8 5105/04 Science (Chemistry)


Kent Ridge Secondary School End-of-Year Examination 2019

www.KiasuExamPaper.com
255
7 (a) The pie chart in Fig. 7.1 shows the composition of air.

other gases

gas B

gas A

Fig. 7.1

(i) Identify gas A.

[1]

(ii) Name two gases that are classified under ‘other gases’.

[1]

(b) The graph in Fig. 7.2 shows the amount of carbon monoxide in the air of a large
city.

Fig. 7.2

(i) At what times is the level of carbon monoxide highest?

(ii) The peak period in a city is typically between 7 to 8 am and 6.30 to 7.30 pm.

Suggest why the level of carbon monoxide is the highest at these times.

Secondary 3 Normal (Academic) 9 5105/04 Science (Chemistry)


Kent Ridge Secondary School End-of-Year Examination 2019

www.KiasuExamPaper.com
256
(iii) Carbon monoxide is an air pollutant. What is the effect of carbon monoxide
on human health?

(iv) Name another pollutant that can commonly be found in the air sample of the
city.

[4]

(c) Carbon monoxide is a covalent compound with the formula CO.

(i) Calculate the relative molecular mass of carbon monoxide.


[relative atomic masses, A r : C, 12; O, 16]

relative molecular mass =

(ii) Hence, calculate the number of moles in 9.8 g of carbon monoxide.

number of moles = [2]

End of Paper

Secondary 3 Normal (Academic) 10 5105/04 Science (Chemistry)


Kent Ridge Secondary School End-of-Year Examination 2019

www.KiasuExamPaper.com
257
Secondary 3 Normal (Academic) 11 5105/04 Science (Chemistry)
Kent Ridge Secondary School End-of-Year Examination 2019

www.KiasuExamPaper.com
258
www.KiasuExamPaper.com
259
CONFIDENTIAL

Kent Ridge Secondary School


End-of-Year Examination 2019
Secondary 3 Normal Academic
Science Chemistry
Marking Scheme
Setter: Ms Siti Fadzilah

Paper 3 (20 marks)

1 2 3 4 5 6 7 8 9 10
D B B B A C B D A D

11 12 13 14 15 16 17 18 19 20
B B D B B A D C B C

Paper 4 Section A (14 marks)

Qn no Remarks
1 (a)(i) B and C 1
(a)(ii) D 1
(b)(i) True 1
(b)(ii) True 1
Q1: [4]
Q
2 (a) CaOO 2 correct:
(b) MgCl 2 1m
(c)) CaSO
Ca
CaSO 4
(d) Al(NO
Al(N
NO 3 ) 3 Max: 2m
Q2: [2]
3 (a)
(a
a) water
wa
w ate
t r/e
et
ethanol
th
haanol 1
(b) S 1
(c)
c) R an
a
anddT 1
Q3: [3]
4 (a) 3 correct:
number
n umbe of number of number of 1m
symbol
symb
bol
electrons
el neutrons protons
ଷଵ
Max: 1m
ଵହܲ 15 16 15

(b) P4O6 1
(c) Covalent 1
(d) 1m:
shared
electrons
H P H
1m:
valence
electrons
H
Max: 2m
Q4: [5]

Secondary 3 Normal (Academic) 1 5105/03 and 04 Science (Chemistry)


Kent Ridge Secondary School End-of-Year Examination 2019

www.KiasuExamPaper.com
260
CONFIDENTIAL

Paper 4 Section B (16 marks)

Qn no Remarks
5 (a)(i) Salt: magnesium chloride 1
1
Gas: hydrogen
(a)(ii) Test: Lighted splint 1
Result: Flame extinguished with a ‘pop’ sound. 1
(b)(i) Acidic, pH less than 7. 1
(b)(ii) pH 7 1
(b)(iii) 2.0 g 1
Accept: 2 g
Reject: if no unit
(b)(iv) Green to blue 1
Reject: purple
Q5: [8]
6 (a)(i) halogen 1

(a)(ii) Down the group, 1m each


- Melting and boiling point
oint increasess Max: 2m
Allow: state changes
nges from
fro s,, liquid
om gas, liq
qui to ssolid
uid to o id
ol
- Colour darkens ns
- Density increasess

Allow iff students s did


diid not
d no ot write
w itte the
wr tth
he phrase
ph
p hra
r se ‘down the
thee group’
(a)(iii) Chlorine
hlorine iiss lelless
ess reactive
rea
ea c
ea cttiivve than
tth
haan
n fluorine.
flu
l uo
orrin
ine. 1
Chlorine
Ch
hlo
lo
lorine iiss unable
un
u n
nab
ab
a ble to
to displace
d sp
di pla
lace
c fluorine
fluorrin
ine from
fo
fr om
m potassium
potassi
sium
si um fluoride.
flu
luoride

Any
y one
on
o ne point
p iin
po nt
(b)(i)
((b
(b)(
b)(
)(i)
(i 2 correct:
element
ellem
emen
e t symbol
sy
s yymb
mb
mbol
bo atomic
atom number
micc n um
umber
um electronic structure 1m
lithium
lith
tth
hiu
i um Li
L 3 2,1 Max: 2m
sodium
so
s od
odi
diium Na 11 2,8,1
pota
potassiu
ass
ssiu
iu
K 19 2,8,8,1
m

(b)(ii) All of the elements have one electron in their outer shell. 1

Allow: same number of electron in their outer shell.


(b)(iii) Cs+ 1
Q6: [8]
7 (a)(i) Nitrogen 1

(a)(ii) Argon 2 correct:


Carbon dioxide 1m

Max: 1m
(b)(i) 8 am and 7.30 pm 1

Accept: 0800 and 1930


Secondary 3 Normal (Academic) 2 5105/03 and 04 Science (Chemistry)
Kent Ridge Secondary School End-of-Year Examination 2019

www.KiasuExamPaper.com
261
CONFIDENTIAL

Qn no Remarks
(b)(ii) These are the times where many there are much incomplete 1
combustion of carbon containing fuels in cars/transport

Allow:
- More transportation for people to go to work and return
home
- There are more cars on the road at that time.

(b)(iii) - Poisonous gas 1


- Poisonous gas causing headaches and brain damage
- Poisonous gas which causes death over prolonged exposure
- Combines with haemoglobin in blood, preventing oxygen from
being transported to other parts of the body.

Reject: toxic gas

Any one point

(b)(iv) Sulphur dioxide / oxides of nitrogen


rogen / nitroge
nitrogen
ge
en di
dioxide
iox
o id
ide
d 1

de
Reject: carbon dioxide
(c)(i) 28 1
(c)(ii) 9.8/28 = 0.35 mol
mo
ol 1

Reject: iff no
no unit
un
u nit
Q6: [8]

Secondary 3 Normal (Academic) 3 5105/03 and 04 Science (Chemistry)


Kent Ridge Secondary School End-of-Year Examination 2019

www.KiasuExamPaper.com
262
www.KiasuExamPaper.com
263
2019 Sec 3 NA Science Chemistry SA2
Kranji Secondary School
Section A: Multiple Choice Questions (20 marks)

For each question, there are four possible answers, A, B, C and D. Choose the one you consider
correct and record your choice in soft pencil on the OMR sheet provided.

1 Which of the following apparatus can be used to measure 24.3 cm3 of solution most
accurately?

A burette
B conical flask
C measuring cylinder
D pipette

2 Heating a liquid causes it to become a vapour.

What happens to the molecules of the liquid during this process?

the molecules get bigger the molecules move further apart


A 9 9
B 9 x
C x 9
D x x

3 A solid metal is heated until it turns to vapour.

The graph shows the temperature of the metal during this process.

Which part of the graph shows the melting of the metal?

4 What can be used to test the purity of a compound?

A melting point
B colour of crystals
C size of crystals
D solubility

www.KiasuExamPaper.com
264
5 Which stages occur in distillation?

A condensation then evaporation


B condensation then filtration
C evaporation then condensation
D filtration then evaporation

6 A mixture of two sugars was compared with four different sugars using chromatography.
The results are shown in the diagram.

Which two sugars does this mixture contain?

A 1 and 2
B 1 and 4
C 2 and 3
D 2 and 4

7 Mixture 1 contains sand and water.

Mixture 2 contains salt and water.

Which method of separation could be used to obtain each of the required products from each
mixture?

mixture 1 mixture 2
to obtain sand to obtain water to obtain salt to obtain water
A crystallisation distillation filtration filtration
B crystallisation filtration filtration distillation
C filtration distillation crystallisation filtration
D filtration filtration crystallisation distillation

3
www.KiasuExamPaper.com
265
8 The diagrams show the nuclei of four different atoms.

Which two atoms are isotopes of each other?

A Q and R
B Q and T
C R and S
D S and T

9 Which change takes place when an atom becomes a positive ion?

A An electron is added.
B An electron is removed.
C A proton is added.
D A proton is removed.

10 Which particle is an ion?

number of protons number of neutrons number of electrons


A 1 0 1
B 3 4 3
C 6 6 6
D 11 12 10

11 Carbon and chlorine form a chloride.

What is the formula of this chloride?

A CCl2
B CCl4
C CaCl2
D CaCl4

12 What are charges on the ions in Al2(SO4)3?

Al SO4
A -2 +3
B +2 -3
C -3 +2
D +3 -2

4
www.KiasuExamPaper.com
266
13 The diagram shows a molecule of hydrogen fluoride.

In the molecule hydrogen fluoride, HF,

A the hydrogen and fluorine share a pair of electrons.


B the hydrogen and fluorine share a pair of protons.
C the hydrogen gives the fluorine an electron.
D the hydrogen gives fluorine a proton.

14 A compound has a molecular formula of X2O and a relative molecular mass of 44.

What is the relative atomic mass of X?

A 9
B 14
C 18
D 28

15 The colour of Universal Indicator in each of four solutions is shown in the table.

Which solution is likely to be an acid of pH 1-2?

solution colour of the Universal Indicator


A blue
B green
C red
D violet

16 Which of the following does not react with dilute sulfuric acid?

A magnesium hydroxide
B magnesium metal
C magnesium nitrate
D magnesium carbonate

17 Many crops will not grow well in an acidic soil.

Which type of chemical reaction takes place when farmers add calcium hydroxide to the soil?

A decomposition
B neutralisation
C precipitation
D saturation

5
www.KiasuExamPaper.com
267
18 The following gases are present in car exhaust fumes.

• carbon dioxide • nitrogen dioxide

• carbon monoxide • water vapour

• nitrogen

Which of these gas(es) is/are also present in unpolluted air?

A nitrogen only
B nitrogen and water vapour only
C nitrogen, carbon dioxide and water vapour only
D nitrogen, carbon monoxide, carbon dioxide and water vapour only

19 A steel works and a chemical works are built near to a city. The limestone (calcium carbonate)
buildings in the city begin to crumble.

Which gas is most likely to cause this damage?

A carbon dioxide
B carbon monoxide
C oxygen
D nitrogen dioxide

20 The diagram shows steel wool inside a test-tube. The test-tube is inverted in water, trapping
air inside. Steel requires oxygen and water to rust.

What will be the water level after several days?

6
www.KiasuExamPaper.com
268
Section B: Structured Questions (14 marks)

Answer ALL questions in the spaces provided.

1 The diagrams A, B, C, D, E and F in Fig. 1.1 represent the particles in different substances.

Fig.1.1

(a) Write in the table, which of the diagrams A, B, C, D, E and F, best represents the
descriptor in the table. Each diagram can only be used once.

descriptor diagram
liquid element
gaseous compound
solid mixture
liquid mixture
[2]

(b) Diagram D in Fig.1.1 represents argon.

Argon contains sub-atomic particles like protons, neutrons and electrons.

(i) Complete the table to show the relative charges and relative masses of a
proton, a neutron and an electron.

particle relative charge relative mass


proton 1
neutron 0
electron
[2]

7
www.KiasuExamPaper.com
269
(ii) Using information from the Periodic Table on page 13. State the number of
protons, electrons and neutrons on one atom of argon.

particle number
proton
neutron
electron
[2]

(iii) Explain why argon exists as atoms instead of as molecules.

……………………………….…………………………………………………………

……………………………….………………………………………...……………[1]

2 The table below shows some information about potassium chloride.

melting point/°C 770


boiling point/°C 1420
electrical conductivity at 25°C does not conduct electricity
solublility in water soluble

(a) (i) State if potassium chloride is a solid, liquid or gas at 25°C.

……………………………….……..………………………………...……………[1]

(ii) Explain why potassium chloride does not conduct electricity at 25°C.

……………………………….……..………………………………...……………[1]

(iii) Draw the dot and cross diagram to show the bonding in potassium chloride.
Show only valence electrons.

[2]

(b) (i) Potassium chloride is a salt that can be prepared by an acid-alkali method
of preparation.

State the method used to prepare potassium chloride.

…………………………………..…………………………………...……………[1]

(ii) Name the acid and the alkali used to prepare potassium chloride.

Acid : …………………………………....……………

Alkali : ……………………………………………….. [2]

8
www.KiasuExamPaper.com
270
Section C: Free Response Questions (16 marks)

Answer ALL questions in the spaces provided.

3 A student used the apparatus in Fig. 3.1 to burn sulfur in air.

sulfur

Fig.3.1

As a result of the reaction, the piece of sulfur changed into an acidic gas, sulfur dioxide.

The student breathe in and suffered irritation to the eyes and lungs and he remembered that
sulfur dioxide is an air pollutant.

(a) (i) State a source of sulfur dioxide

....................................................................................................................................

.............................................................................................................................. [1]

(ii) Other than irritation to the eyes and lungs, state another problem that sulfur
dioxide causes.

....................................................................................................................................

.............................................................................................................................. [1]

The student decided to replace sulfur with zinc and heated since in air. Zinc reacts with
oxygen in the air to produce zinc oxide.

(b) (i) Write a balanced chemical equation of the reaction between zinc with oxygen.

.............................................................................................................................. [1]

(ii) The zinc oxide produced is able to react and dissolve in both acid and alkali. What
type of oxide is zinc oxide?

.............................................................................................................................. [1]

9
www.KiasuExamPaper.com
271
The student then added some zinc to some dilute sulfuric acid contained in test-tube A. He
also added some of the zinc oxide produced by the reaction earlier to some dilute sulfuric
acid in test-tube B as shown in Fig. 3.2.

zinc
zinc oxide

dilute sulfuric acid

Fig.3.2

(c) (i) One of the reactions in Fig. 3.2 produced a gas.

In which test-tube, A or B, were gas bubbles observed? Explain your answer.

test-tube : …………………………………………….

explanation : ………………….……………………………………………………………

....................................................................................................................................

.............................................................................................................................. [2]

(ii) Describe the test for the gas produced in (c)(i).

test : ……………………………………………………………………………..………….

result : ………………….……………………………...……………………………………

.............................................................................................................................. [2]

10
www.KiasuExamPaper.com
272
4 Lead(II) chloride is used to make white pigment in white paint. It has been banned as lead
compounds are toxic.

Lead(II) chloride is prepared in the laboratory by mixing aqueous lead(II) nitrate and aqueous
sodium chloride together.

(a) (i) State if lead(II) chloride is a soluble or insoluble salt.

.............................................................................................................................. [1]

(ii) State the method used to prepare lead(II) chloride in the laboratory

.............................................................................................................................. [1]

(b) The reaction between aqueous lead(II) nitrate and aqueous sodium chloride is shown
below.

Pb(NO3)2 + 2NaCl Æ PbCl2 + 2NaNO3

The lead(II) chloride formed was then filtered from the mixture and dried with filter paper
and weighed.

(i) Calculate the relative molecular mass of lead(II) chloride.

relative molecular mass of lead(II) chloride .................................... [1]

(ii) Calculate the number of moles of lead(II) chloride used if 5.56 g of lead(II) chloride
was used to make the solution for the reaction.

moles of lead(II) chloride .................................... [1]

11
www.KiasuExamPaper.com
273
(iii) Calculate the moles of sodium chloride used.

moles of sodium chloride .................................... [1]

(iv) Calculate the mass of sodium chloride used.

mass of sodium chloride ....................................g [1]

(c) Lead(II) chloride has a high melting point. Explain, in terms of bonding, why it has a high
melting point.

.............................................................................................................................................

.............................................................................................................................................

....................................................................................................................................... [2]

END OF PAPER
12
www.KiasuExamPaper.com
274
The Periodic Table of Elements
Group
I II III IV V VI VII 0
1 2
H He
hydrogen helium
Key 1 4
3 4 proton (atomic) number 5 6 7 8 9 10
Li Be atomic symbol B C N O F Ne
lithium beryllium name boron carbon nitrogen oxygen fluorine neon
7 9 relative atomic mass 11 12 14 16 19 20
11 12 13 14 15 16 17 18
Na Mg Al Si P S Cl Ar
sodium magnesium aluminium silicon phosphorus sulfur chlorine argon
23 24 27 28 31 32 35.5 40
19 20 21 22 23 24 25 26 27 28 29 30 31 32 33 34 35 36
K Ca Sc Ti V Cr Mn Fe Co Ni Cu Zn Ga Ge As Se Br Kr
potassium calcium scandium titanium vanadium chromium manganese iron cobalt nickel copper zinc gallium germanium arsenic selenium bromine krypton
39 40 45 48 51 52 55 56 59 59 64 65 70 73 75 79 80 84
37 38 39 40 41 42 43 44 45 46 47 48 49 50 51 52 53 54
Rb Sr Y Zr Nb Mo Tc Ru Rh Pd Ag Cd In Sn Sb Te I Xe
rubidium strontium yttrium zirconium niobium molybdenum technetium ruthenium rhodium palladium silver cadmium indium tin antimony tellurium iodine xenon
85 88 89 91 93 96 - 101 103 106 108 112 115 119 122 128 127 131

275
55 56 57 – 71 72 73 74 75 76 77 78 79 80 81 82 83 84 85 86
Cs Ba lanthanoids Hf Ta W Re Os Ir Pt Au Hg Tl Pb Bi Po At Rn
caesium barium hafnium tantalum tungsten rhenium osmium iridium platinum gold mercury thallium lead bismuth polonium astatine radon
133 137 178 181 184 186 190 192 195 197 201 204 207 209 – – –
87 88 89 – 103 104 105 106 107 108 109 110 111 112 114 116
actinoids

www.KiasuExamPaper.com
Fr Ra Rf Db Sg Bh Hs Mt Ds Rg Cn Fl Lv
francium radium Rutherfordium dubnium seaborgium bohrium hassium meitnerium darmstadtium roentgenium copernicium flerovium livermorium
– – – – – – – – – – – – –

57 58 59 60 61 62 63 64 65 66 67 68 69 70 71
lanthanoids La Ce Pr Nd Pm Sm Eu Gd Tb Dy Ho Er Tm Yb Lu
lanthanum cerium praseodymium neodymium promethium samarium europium gadolinium terbium dysprosium holmium erbium thulium ytterbium lutetium
139 140 141 144 – 150 152 157 159 163 165 167 169 173 175
89 90 91 92 93 94 95 96 97 98 99 100 101 102 103
actinoids Ac Th Pa U Np Pu Am Cm Bk Cf Es Fm Md No Lr
actinium thorium protactinium uranium neptunium plutonium americium curium berkelium californium einsteinium fermium mendelevium nobelium lawrencium
– 232 231 238 – – – – – – – – – – –
3
The volume of one mole of any gas is 24 dm at room temperature and pressure (r.t.p)

13
14

www.KiasuExamPaper.com
276
KRANJI SECONDARY SCHOOL
Sec 3NA 5105 EYE Answer Key / 2019

Section A: MCQ (20 marks)

1 A 6 D 11 B 16 C
2 C 7 D 12 D 17 B
3 B 8 A 13 A 18 C
4 A 9 B 14 B 19 D
5 C 10 D 15 C 20 D

Section B: Structured Questions (14 marks)

1 (a) F, A, C, E (Any 2 for 1M) 2

(b)(i) +1 (first row), 1 (second row), -1, 1/1840 (third row)) [1M for any
an
ny 2]] 2

(b)(ii) P – 18, N – 22, E – 18 (1 correct – 1M, all 3 correct for


fo
or 2M)
2M)
2M 2

(b)(iii) To obtain the stable electronic structure


tructure
re with
re with full
fu
ull electron
ele
ect
ctrro
ron shells.
ron shellss. 1

2 (a) (i) Solid 1

(ii) No free mobile ions


ons to car
carry
rry
ry e
electrical
le
eccttri
rica
ica
cal ch
charges.
har
arg
gees.
s. 1

(iii)

Legend:
nd:: X – el
e
electron
leecctrron of K
O – electron
ellectron of Cl
e

(b)(i) Titration 1

(b)(ii) Acid : Hydrochloric acid 1


Alkali : Potassium hydroxide 1

Section C: Free Response Questions (16 marks)

3 (a)(i) Volcanic activities/Combustion of fossil fuels such as coals from power stations 1

(a) (ii) Forms acid rains that corrode marble buildings, kills aquatic wildlife as ponds and 1
lakes become too acidic and causes poor plant growth

(b) (i) 2Zn + O2 Æ 2ZnO 1

www.KiasuExamPaper.com
277
(b) (ii) Amphoteric oxide 1

(c) (i) Test-tube A, 1


zinc is a metal which reacts with acid to produce hydrogen gas. 1

(c) (ii) Test : Pushed lighted splint into gas (Allow ECF of 2 marks from (c)(i)) 1
Results : Gas extinguishes lighted splint with a pop sound. 1

4 (a) (i) Insoluble 1

(a) (ii) Precipitation 1

(b) (i) 207 + 2(35.5) = 278 1

(b) (ii) No. of mol of lead(II) chloride (5.56/278) = 0.02 mol 1

(b) (iii) Comparing mol ratio (PbCl2 : NaCl, 1:2)


1
No. of mol of sodium chloride is (0.02 x 2) = 0.04 mol

(b) (iv) Mass of sodium chloride is (0.04 x 58.5


58.5)
.5
5) = 1.1
1.17
17 g 1

(c) Lead(II) chloride has strong


ng elect
electrostatic
trro
ost
s atic fforces
orce
or c s of
of a
attraction
ttrrra
tt act
ctio
ion
n be
between
betw
twee
een (oppositely 1
charged/lead and chloride)
e) ionss

a lot of energy is needed


neede
ed to
t overcome
ove
errc
com
me these
thes
these forces
forrc
forc
fo ce of
ces of attraction.
a traction
at on..
on 1

www.KiasuExamPaper.com
278
www.KiasuExamPaper.com
279

You might also like